Jaankari Rakho & : Geography https://m.jaankarirakho.com/rss/category/geography Jaankari Rakho & : Geography hin Copyright 2022 & 24. Jaankari Rakho& All Rights Reserved. General Competition | Geography | राज्य तथा केन्द्रशासित प्रदेश और पड़ोसी देश https://m.jaankarirakho.com/965 https://m.jaankarirakho.com/965 General Competition | Geography | राज्य तथा केन्द्रशासित प्रदेश और पड़ोसी देश

वस्तुनिष्ठ प्रश्न एवं उत्तर

1. निम्नांकित में से कौन-सा एक सुमेलित नहीं है ?
(a) आंशी राष्ट्रीय उद्यान - कर्नाटक 
(b) बालफक्रम राष्ट्रीय उद्यान - मेघालय
(c) चंदोली राष्ट्रीय उद्यान -- गुजरात
(d) हेमिस राष्ट्रीय उद्यान - लद्दाख
2. मिजोरम में स्थित फावंगपूई राष्ट्रीय उद्यान को किस अन्य नाम से भी जाना जाता है ?
(a) काला पर्वत उद्यान
(b) नीला पर्वत उद्यान
(c) पीला पर्वत उद्यान
(d) मिजो हिल्स उद्यान
3. भारत के किस राज्य में वन्यजीव अभ्यारण्य सबसे अधिक संख्या में हैं ?
(a) कर्नाटक
(b) तमिलनाडु
(c) महाराष्ट्र
(d) मध्य प्रदेश
 
4. निम्नलिखित राज्यों में से किस एक पाखुई वन्यजीव अभ्यारण्य अवस्थित है ?
(a) अरुणाचल प्रदेश 
(b) मणिपुर
(c) मेघालय
(d) नागालैंड
5. हाल ही में हमारे वैज्ञानिकों ने केले के पौधे की एक नई और भिन्न जाति की खोज की है, जिसकी ऊँचाई लगभग 11 मीटर तक जाती है और उसके फल का गूदा नारंगी रंग का है। यह भारत के किस भाग में खोजी गई है? 
(a) अंडमान द्वीप
(b) अन्नामलाई वन
(c) मैकाल पहाड़ियाँ
(d) पूर्वोत्तर उष्णकटिबंधीय वर्षा वन
6. भारत के एक विशेषर्धन क्षेत्र में, स्थानीय लोग जीवित वृक्षों की जड़ों का अनुवर्धन कर इन्हें जलधारा के आर-पार सुदृढ़ पुलों में रूपांतरित कर देते हैं। जैसे-जैसे समय गुजरता है, ये पुल अधिक मजबूत होते जाते हैं। ये अनोखे 'जीवित जड़ पुल' कहाँ पाए जाते हैं ?
(a) मेघालय
(b) हिमाचल प्रदेश
(c) झारखण्ड
(d) तमिलनाडु
7. भारत के राज्यों का निम्नलिखित में से कौन-सा एक युग्म, सबसे पूर्वी और सबसे पश्चिमी राज्य को इंगित करता है ?
(a) असम और राजस्थान 
(b) अरुणाचल प्रदेश और राजस्थान 
(c) असम और गुजरात
(d) अरुणाचल प्रदेश और गुजरात
8. निम्नलिखित में से कौन-सा एक नेशनल पार्क इसलिये अनूठा है कि वह एक प्लवमान (फ्लोटिंग) वनस्पति से युक्त अनूप ( स्कैंप ) होने के कारण समृद्ध जैव विविधता को बढ़ावा देता है ?
(a) भीतरकणिका नेशनल पार्क
(b) केइबुल लामजाओ नेशनल पार्क
(c) केवलादेव घाना नेशनल पार्क
(d) सुल्तानपुर नेशनल पार्क
9. निम्नलिखित राज्यों में से किसमें / किनमें सिंहपुच्छी वानर ( मेकॉक) अपने प्राकृतिक आवास में पाया जाता है ? 
1. तमिलनाडु 
2. केरल
3. कर्नाटक
4. आन्ध्र प्रदेश
नीचे दिए गए कूट का प्रयोग कर सही उत्तर चुनिये
(a) केवल 1, 2 और 3 
(b) केवल 2
(c) केवल 1, 3 और 4
(d) 1, 2, 3 और 4
10. निम्नलिखित राज्यों में से किस राज्य में राजीव गाँधी उड़ान संस्थान (RGNFI ) स्थापित किया जा रहा है ? 
(a) कर्नाटक 
(b) महाराष्ट्र
(c) केरल
(d) ओडिशा
11. छत्तीसगढ़ की सीमा उभयनिष्ठ नहीं है 
(a) महाराष्ट्र के साथ 
(b) बिहार के साथ
(c) ओडिशा के साथ
(d) आंध्र प्रदेश के साथ
12. एशिया का पहला जलविद्युत संयंत्र शिवसमुद्रम में स्थापित किया गया है। यह निम्नलिखित में से किस राज्य में अवस्थित है ? 
(a) केरल
(b) कर्नाटक
(c) आंध्र प्रदेश
(d) तमिलनाडु
13. निम्न में से कौन-सा राज्य क्षेत्रफल में सबसे छोटा है?
(a) उत्तर प्रदेश
(b) मध्य प्रदेश
(c) राजस्थान
(d) उत्तराखण्ड
14. लातूर किस प्रांत में है ? 
(a) आंध्र प्रदेश 
(b) महाराष्ट्र
(c) गुजरात 
(d) मध्य प्रदेश
15. 'विदर्भ' एक प्रादेशिक नाम है भारत में, और यह 
(a) गुजरात का अंग है
(b) महाराष्ट्र का अंग है
(c) मध्य प्रदेश का अंग है
(d) आडिशा का अंग है
16. 'पाट' अंचल (Pat Region ) अवस्थित है 
(a) बिहार में 
(b) झारखण्ड में
(c) मध्य प्रदेश में
(d) मेघालय में में
17. झुमरी तलैया (रेडियो पर गीतों की फरमाइश के लिए प्रसिद्ध ) किस राज्य में स्थित है ? 
(a) बिहार
(b) झारखंड
(c) ओडिशा
(d) पश्चिम बंगाल
18. भारत के निम्नलिखित राज्यों में से किसे 'भारत का कोहिनूर' कहा जाता है ? 
(a) आंध्र प्रदेश
(b) केरल
(c) मध्य प्रदेश
(d) राजस्थान
19. मणिपुर का अधिकांश धरातल है 
(a) मैदानी
(b) पठारी
(c) दलदली
(d) पर्वतीय
20. मणिपुर में कुछ लोग लटकी हुई गाद ( Silt ) से बंधे अपतृण (Weeds) और सड़ती वनस्पति के तैरते हुए द्वीपों (Floating Island) पर बने हुए मकानों में रहते हैं, इन द्वीपों को कहते हैं
(a) तिपिस
(b) बरखान्स
(c) फूमडि
(d) इजबा
21. भारत के किस राज्य को 'सिलिकॉन स्टेट' के नाम से जाना जाता है ?
(a) गोवा
(b) आंध्र प्रदेश
(c) कर्नाटक
(d) केरल
22. कहा जाता है कि भारत में सिलिकॉन वैली स्थित है
(a) बंगलुरू में 
(b) चेन्नई में
(c) कोलकाता में
(d) हैदराबाद में
23. निम्नलिखित में कौन-सा सही सुमेलित है ?
(a) औरंगाबाद - आंध्र प्रदेश
(b) पालनपुर - गुजरात
(c) हुबली - महाराष्ट्र
(d) गुन्टूर - ओडिशा
24. निम्नलिखित में से कौन सुमेलित नहीं है ?
(a) छत्तीसगढ़ - छत्तीसगढ़ मैदान
(b) झारखंड - छोटानागपुर पठार
(C) महाराष्ट्र - वृष्टिछाया प्रदेश
(d) आंध्र प्रदेश - मालनड
25. निम्न वाक्यों में कौन-सा सही है ?
(a) मध्य प्रदेश की सीमा सात राज्यों से लगी है
(b) भोपाल कर्क रेखा के उत्तर में स्थित है
(c) पंजाब राज्य की सीमा कहीं भी जम्मू-कश्मीर से नहीं मिलता
(d) अरुणाचल प्रदेश में कोई राष्ट्रीय पार्क नहीं है
26. निम्न राज्यों में से किसका क्षेत्रफल सबसे कम है ? 
(a) आंध्र प्रदेश 
(b) गुजरात
(c) कर्नाटक
(d) तमिलनाडु
27. भारत के समस्त राज्यों में क्षेत्रफलानुसार, उत्तर प्रदेश का क्या स्थान है ?
(a) पहला
(b) दूसरा
(c) तीसरा
(d) चौथा
28. भारत का लगभग 30 प्रतिशत क्षेत्र तीन राज्यों में समाहित है। ये तीन राज्य है 
(a) राजस्थान, उत्तर प्रदेश एवं आंध्र प्रदेश
(b) मध्य प्रदेश, आंध्र प्रदेश एवं गुजरात
(c) राजस्थान, मध्य प्रदेश एवं महाराष्ट्र
(d) महाराष्ट्र, आंध्र प्रदेश एवं उत्तर प्रदेश
29. क्षेत्रफल के क्रम में भारत के तीन बड़े राज्य हैं
(a) राजस्थान, मध्य प्रदेश, महाराष्ट्र
(b) मध्य प्रदेश, राजस्थान, महाराष्ट्र
(c) महाराष्ट्र, राजस्थान, मध्य प्रदेश
(d) मध्य प्रदेश, महाराष्ट्र, राजस्थान
30. निमें से कौन-सा राज्य, क्षेत्रफल में सबसे छोटा है ?
(a) उत्तर प्रदेश
(b) मध्य प्रदेश
(c) राजस्थान
(d) उत्तराखंड
31. भारत में जनसंख्या के अनुसार, तीसरा एवं क्षेत्रफल में बारहवाँ राज्य है
(a) महाराष्ट्र
(b) मध्य प्रदेश
(c) कर्नाटक
(d) बिहार
32. असम कुल मिलाकर कितने राज्यों एवं केंद्रशासित क्षेत्रों से घिरा हुआ है ?
(a) 6
(b) 7
(c) 8
(d) 9
33. निम्न नगरों में से माध्य समुद्र तल से किसकी ऊँचाई अधिकतम है ? 
(a) बंगलुरू
(b) दिल्ला 
(c) जोधपुर
(d) नागपुर
34. कौन-सा राज्य उत्तर-पूर्वी राज्यों की 'सात बहनों' का भाग नहीं है ? 
(a) मेघालय
(b) पश्चिम बंगाल
(c) अरुणाचल प्रदेश
(d) त्रिपुरा
35. देश के किस प्रदेश में सबसे अधिक जिले हैं ? 
(a) मध्य प्रदेश 
(b) महाराष्ट्र
(c) तमिलनाडु
(d) उत्तर प्रदेश
36. भौगोलिक क्षेत्र की दृष्टि से भारत के निम्न जिलों में से कौन - सा जिला सबसे बड़ा है ? 
(a) लेह
(b) कच्छ
(c) जैसलमेर
(d) बाड़मेर
37. सोनभद्र जिले को स्पर्श करती हैं
(a) दो राज्यों की सीमाएँ
(b) तीन राज्यों की सीमाएँ
(c) चार राज्यों की सीमाएँ
(d) पाँच राज्यों की सीमाएँ
38. चार दक्षिणी राज्य आंध्र प्रदेश, कर्नाटक, केरल और तमिलनाडु में से कौन-सा सबसे अधिक भारतीय राज्यों के साथ सीमावर्ती है ? 
(a) केवल आंध्र प्रदेश
(b) केवल कर्नाटक
(c) आंध्र प्रदेश और कर्नाटक में से प्रत्येक
(d) तमिलनाडु और केरल में से प्रत्येक
39. निम्नलिखित राज्यों के समूहों में से कौन एक तेलंगाना राज्य की सीमा बनाता है ?
(a) आंध्र प्रदेश, कर्नाटक, महाराष्ट्र, छत्तीसगढ़
(b) कर्नाटक, महाराष्ट्र, म० प्र०, आंध्र प्रदेश
(c) आंध्र प्रदेश, छत्तीसगढ़, कर्नाटक, झारखंड
(d) छत्तीसगढ़, कर्नाटक, महाराष्ट्र, म॰ प्र॰
40. निम्न में कौन सुमेल नहीं है ? 
(a) त्रिपुरा - अगरतल्ला
(b) मणिपुर – आइजोल 
(c) नागालैंड - कोहिमा
(d) अरुणाचल प्रदेश – ईटानगर
41. गुजरात की राजधानी कौन-सी है ?
(a) गोधरा
(b) बड़ौदा
(c) गाँधीनगर
(d) अहमदाबाद
42. भारत का कौन-सा राज्य अधिकतम राज्य सीमाओं को छूता है ?
(a) मध्य प्रदेश
(b) कर्नाटक
(c) आंध्र प्रदेश
(d) उत्तर प्रदेश
43. मध्य प्रदेश से किस राज्य की सीमा नहीं छूती है ?
(a) राजस्थान
(b) गुजरात
(c) झारखंड
(d) महाराष्ट्र
44. भारत का सबसे बड़ा संघ राज्य है
(a) दमन और दीव 
(b) पुडुचेरी
(c) दिल्ली
(d) चंडीगढ़
45. भारत का सबसे छोटा केंद्रशासित क्षेत्र कौन-सा है ?
(a) चंडीगढ
(b) अंडमान और निकोबार
(c) लक्षद्वीप
(d) दादरा तथा नगर हवेली
46. निम्न में से एक केन्द्रशासित क्षेत्र नहीं है
(a) त्रिपुरा 
(b) दमन एवं दीव
(c) लक्षद्वीप
(d) पुडुचेरी
हमसे जुड़ें, हमें फॉलो करे ..
  • Telegram ग्रुप ज्वाइन करे – Click Here
  • Facebook पर फॉलो करे – Click Here
  • Facebook ग्रुप ज्वाइन करे – Click Here
  • Google News ज्वाइन करे – Click Here
]]>
Sat, 13 Apr 2024 05:43:31 +0530 Jaankari Rakho
General Competition | Geography | आपदा प्रबंधन https://m.jaankarirakho.com/964 https://m.jaankarirakho.com/964 General Competition | Geography | आपदा प्रबंधन
  • प्राकृतिक आपदा के अंतर्गत बाढ़, सुखा, भूकम्प, सुनामी, इत्यादि आते हैं।
  • द्वितीय प्रशासनिक सुधार आयोग ने अपनी रिपोर्ट में प्राकृतिक आपदा के जगह आपदा संकट शब्द का प्रयोग किया है।
  • आपदा संकट से निपटने के लिए सितम्बर, 2005 में एक सरकारी आदेश के द्वारा राष्ट्रीय आपदा प्रबंधन प्राधिकरण का गठन किया गया। इस संगठन की अध्यक्षता प्रधानमंत्री के द्वारा किया जाता है। गृहमंत्रालय के अधीन
  • राष्ट्रीय आपदा एक्ट दिसम्बर, 2005 में आया है।
  • भारत को चार भूकम्पीय क्षेत्र में बाँटा गया है। भूकम्प को लेकर अत्यधिक जोखिम वाले क्षेत्र के अंतर्गत हिमालय का गिरिपाद क्षेत्र, उत्तरी-पूर्वी भारत आता है।
  • भारत में सबसे ज्यादा प्राकृतिक आपदा से ओडिशा राज्य प्रभावित है।
  • उत्तरी बिहार बाढ़ के लिए प्रसिद्ध है।
  • 26 दिसम्बर, 2004 को हिंद महासागर में जो सुनामी आया है, उससे भारत का पूर्वी तट अर्थात् कोरोमंडल तट प्रभावित रहा है। इस सुनामी से सबसे ज्यादा प्रभावित तमिलनाडु का नागपट्टिनम जिला रहा है।
  • भारत में सुनामी वार्निंग सेंटर हैदराबाद में स्थित है।
  • भारतीय मौसम वेधशाला पुणे में स्थित है। 
  • केन्द्रीय शुष्क क्षेत्र अनुसंधान संस्थान जोधपुर में स्थित है।

वस्तुनिष्ठ प्रश्न एवं उत्तर

1. इनमें कौन कथन सही नहीं है ?
(a) आपदा मनुष्य के लिए कष्टदायी घटना है
(b) आपदा केवल प्राकृतिक कारणों से ही उत्पन्न होती है
(c) आपदा से बीमारी फैल सकती है
(d) आपदा से धन-संपत्ति की हानि हो सकती है
2. इनमें किस आपदा की गति तीव्र होती है ?
(a) भूस्खलन 
(b) भूअपरदन
(c) भूकंप
(d) भूमि का निम्नीकरण
3. इनमें किससे सीमित क्षेत्र में आपदा उपस्थित होती है ?
(a) भूमंडलीय ऊष्मीकरण
(b) ओजोन परत का क्षरण
(c) नाभिकीय विखंडन
(d) ओला - वृष्टि
4. इनमें कौन मानवजनित आपदा है ?
(a) विनाशकारी रसायनों का फैलना
(b) रेल दुर्घटना
(c) बाँधों का टूटना
(d) इनमें सभी
5. आपदा के समय इनमें से किस प्रबंधन पर तुरंत ध्यान देना आवश्यक नहीं है ?
(a) आपदाग्रस्त लोगों को सुरक्षित स्थान पर पहुँचना
(b) भोजन - जल आदि का प्रबंध करना
(c) स्थानीय लोगों की सहायता से घर बनवा
(d) घायल लोगों के लिए दवा का प्रबंध करना 
6. इनमें कौन-सी घटना आपदा के लिए जिम्मेवार रही है ?
(a) 2001 का गुजरात का भूकंप
(b) 26 दिसंबर, 2004 की सुनामी
(c) अगस्त, 2008 की कोसी की बाढ़
(d) इनमें सभी
7. रेल दुर्घटना का प्रमुख कारण क्या है ?
(a) मानवीय लापरवाही
(b) बिना टिकट यात्रा करना
(c) बिजली से रेलगाड़ी चलाया जाना
(d) रेल द्वारा भारी वस्तुओं की ढुलाई करना
8. किस मानवजनित आपदा का प्रभाव दीर्घकाल तक बना रहता है ?
(a) सड़क दुर्घटना
(b) रेल पुल का टूटना
(c) रासायनिक प्रदूषण
(d) आँधी
9. इनमें कौन - सा उपाय परमाणु संबंधी आपदाओं से बचने के लिए उपयुक्त है ?
(a) परमाणु अस्त्र नहीं बनाए जाएँ
(b) मानव बस्तियाँ परमाणु संयंत्रों से दूर बसाई जाएँ
(c) परमाणु ऊर्जा उत्पन्न करने वाले सभी संयंत्र बंद किए जाएँ 
(d) परमाणु संयंत्रों के निकट भोजनालय नहीं बनाए जाएँ
10. कल-कारखाने कचरे से किस प्रकार की आपदा आ सकती है ?
(a) नाभिकीय आपदा
(b) विस्फोट
(c) रासायनिक प्रदूषण
(d) जैविक आपदा
11. इनमें कौन-सी बीमारी जैविक आपदा लाती है ?
(a) चेचक
(b) इंफ्लूएंजा
(c) एड्स
(d) इनमें सभी
12. जैविक अस्त्रों के भंडारण के लिए इनमें कौन-सा देश बदनाम रहा है ?
(a) इराक
(b) ईरान
(c) पाकिस्तान
(d) जापान
13. सामान्य आपदाओं के लिए इनमें कौन उत्तरदायी है ?
(a) गरीबी
(b) खान-पान में असावधानी
(c) सामाजिक कार्यों में असावधानी
(d) इनमें सभी
14. दूषित जल और दूषित वायु से कौन-सी बीमारी होती है ?
(a) हैजा
(b) प्लेग
(c) कालाजार
(d) इनमें सभी
15. इनमें कौन वृहत आपदा है ?
(a) कुपोषण से बीमार पड़ना
(b) आग लगना
(c) जल का दूषित होना
(d) कमरे का हवादार न होना
16. आतंकवाद किस प्रकार की आपदा है ?
(a) लघु 
(b) प्राकृतिक
(c) वृहत
(d) इनमें से कोई नहीं
17. विमान दुर्घटना का कारण इनमें कौन नहीं है ?
(a) साफ मौसम
(b) वायुयान में तकनीकी खराबी
(c) विमान अपहरण
(d) आग लगना
18. बिजली से आग लगने पर उससे बचने के लिए क्या उपाय करना चाहिए ?
(a) उसे पानी से बुझा देना चाहिए
(b) कमरे की खिड़की और दरवाजा बंद कर देना चाहिए
(c) वर से बाहर निकलते समय बिजली के सभी स्विच बंद कर देना चाहिए
(d) रजाई या कंबल से आग बुझाने का प्रयत्न करना चाहिए
19. इनमें कौन आतंकवाद को जन्म देता है ?
(a) बाजार में भीड़ का बढ़ना
(b) अस्पताल में मरीजों का बढ़ना
(c) कट्टरपंथियों द्वारा धर्म का दुरुपयोग किया जाना
(d) विद्यालयों की संख्या बढ़ना जी.
20. 26 नवंबर, 2008 को भारत के किस नगर में आतंकवादियों ने हमलाकर 166 निर्दोष लोगों की जाने ली थी ?
(a) दिल्ली
(b) इलाहाबाद
(c) सूरत
(d) मुंबई
21. किस आपदा में समुदाय के लोग भाग नहीं लेते ?
(a) आग लगने पर
(b) जंगली जानवरों के आक्रमण होने पर
(c) किसी की तबीयत खराब होने पर
(d) बाढ़ आने पर
22. इनमें सामुदायिक आपदा प्रबंधन का प्रमुख घटक कौन है ? 
(a) पूर्वानुमान
(b) आपदा के समय प्रबंधन की गतिविधियाँ
(c) आपदा समाप्ति के बाद के उपाय
(d) इनमें सभी
23. आपदा प्रबंधन के लिए समुदाय के लोगों में किस गुण का होना अनिवार्य है ?
(a) व्यक्तिगत हित का सोच
(b) ईमानदारी
(c) शारीरिक शक्ति
(d) समाज के हित का सर्वोपरि ध्यान रखना
24. ग्रामीण स्तर पर आपदा प्रबंधन में किनमें सहयोग लिया जा सकता है ?
(a) गाँव के मुखिया और सरपंच में
(b) स्कूल के शिक्षक और प्रधानाचार्य से
(c) प्राथमिक स्वास्थ्य केंद्र के डॉक्टर से
(d) इनमें सभी से
25. इनमें कौन आपदा प्रबंधन का प्रमुख घटक है ?
(a) आपदापूर्व व्यक्तिगत तैयारी
(b) आपदापूर्व सामूहिक तैयारी
(c) आपदा की रोकथाम के लिए दूसरे पर निर्भर रहना
(d) आपदा से असंबद्ध रहना
26. मानवजनित आपदा के प्रभाव को कम करने का कारगर उपाय क्या है ?
(a) आपदारोधी भवन का निर्माण
(b) भूमि उपयोग के विषय में अनजान रहना
(c) जोखिम के क्षेत्र में वस्ती बनाना
(d) सामुदायिक जागरूकता पर ध्यान देना
27. इनमें कहाँ परमाणु ऊर्जा केंद्र स्थापित किया गया है ?
(a) वाराणसी में
(b) दिल्ली में
(c) कँगा में
(d) मेरठ में
28. हिरोशिमा किस देश में स्थित है ?
(a) भारत में
(b) चीन में
(c) जापान में
(d) रूस में
29. रेडियोधर्मी आपदा से बचाव के लिए अंतर्राष्ट्रीय रेडियोधर्मी प्रतीक का विकास किसने किया है ?
(a) कैम्ब्रिज विश्वविद्यालय ने
(b) कॅलिफोर्निया विश्वविद्यालय ने
(c) सक्कियों विश्वविद्यालय ने
(d) संयुक्त राष्ट्र ने
30. भारत में पहला परमाणु ऊर्जा उत्पादक केंद्र कहाँ स्थापित किया गया ?
(a) ट्राम्बे में
(b) पोखरण में
(c) भोपाल में
(d) चेन्नई में
31. विश्व में सबसे पहले किस स्थान पर परमाणु बम गिराया गया था ?
(a) सूरत पर
(b) चेर्नोबिल पर
(c) हिरोशिमा पर
(d) काबुल पर
32. भारत में कहाँ भूमिगत परमाणु विस्फोट परीक्षण हेतु किया गया ?
(a) नरोरा में
(b) कँगा में
(c) पोखरण में
(d) तारापुर में
33. भोपाल गैस त्रासदी किस वर्ष हुई थी ?
(a) 1984 में
(b) 1990 में
(c) 1930 में
(d) 2004 में
34. तमिलनाडु के तूतीकोरिन ताँबा उत्पादन केंद्र में गैस के रिसाव से किस प्रकार की बीमारी फैली थी ?
(a) उलटी होना
(b) सर्दी-खाँसी
(c) उलटी और सीने में जलन
(d) मस्तिष्क ज्वर
35. कौन क्षेत्र अम्ल वर्षा से अधिक प्रभावित होता है ?
(a) उत्तरी बिहार 
(b) दामोदर घाटी
(c) असम घाटी क्षेत्र
(d) इनमें से कोई नहीं
36. डेंगू बीमारी का क्या कारण है ?
(a) आग लगना 
(b) जल जमाव
(c) बाढ़ का प्रकोप
(d) ठंडा भोजन
37. एंथ्रैक्स क्या है ?
(a) एक अतिसूक्ष्म विषाणु 
(b) जंगली
(c) युद्धपोत
(d) युद्ध शु
38. अनुमानतः भारत में एड्स पीड़ितों की संख्या क्या है ?
(a) 20.9 लाख 
(b) 1 करोड़ 
(c) 30 लाख
(d) 50 लाख
39. सामान्य आपदाओं को कितने वर्गों में बाँटा गया है ?
(a) एक वर्ग में
(b) दो वर्गों में
(c) तीन वर्गों में
(d) चार वर्गों में
40. इनमें आतंकवादी आपदा से बचाव के लिए क्या करना चाहिए ?
(a) अनजानी गठरी, झोला आदि नहीं छूना
(b) संदेहात्मक गतिविधि के व्यक्ति की सूचना पुलिस को देना
(c) गाँव, नगर, मुहल्ले में आतंकविरोधी वातावरण बनाना
(d) इनमें सभी
41. सड़क पर जेना का निशान क्यों बना होता है ?
(a) सड़क के सुंदरीकरण के लिए
(b) सड़क पार करने के लिए 
(c) गाड़ी रोकने के लिए
(d) सवारी उतारने के लिए
42. सड़क पर पैदल चलते समय सड़क के किस ओर से चलना चाहिए ?
(a) बाईं ओर से
(b) दाहिनी ओर से
(c) सड़क के बीच के विभाजक पर से
(d) इनमें से कोई नहीं
43. रेलयात्रा में कौन- सा सामान लेकर नहीं चलना चाहिए ? 
(a) अपना सूटकेस 
(b) खाने-पीने का सामान
(c) गैस का सिलिंडर
(d) टिकट
44. निम्नलिखित में कौन आपदा प्रबंधन से संबंधित नहीं है ?
(a) पूर्वानुमान, चेतावनी और प्रशिक्षण
(b) आपदा के समय के प्रबंधन की गतिविधियाँ
(c) आपदा के बाद निश्चित हो जाना
(d) आपदा के बाद भी प्रबंधन कार्य करना
45. ग्रीष्मकाल में निम्नलिखित में किस आपदा की संभावना अधिक रहती है ?
(a) वायुयान दुर्घटना की 
(b) रेल दुर्घटना की
(c) आगजनों की
(d) सड़क दुर्घटना की
46. निम्नलिखित में कौन सामुदायिक आपदा प्रबंधन का प्राथमिक क्रियाकलाप है ?
(a) निकटस्थ स्वास्थ्य केंद्र को सूचित करना
(b) स्वच्छ जल एवं भोजन का प्रबंध
(c) आपातकालीन राहत शिविर की व्यवस्था
(d) इनमें सभी
47. निम्नलिखित में कौन ग्रामीण आपदा प्रबंधन समिति का प्रमुख कार्य है ?
(a) सब की सुरक्षा
(b) चिकित्सा व्यवस्था करना
(c) राहत शिविर की देखभाल 
(d) इनमें सभी
48. प्राकृतिक आपदा किसे कहते हैं ? निम्नांकित विकल्पों में कौन सबसे अधिक सही माना जाएगा ?
(a) सूरज का ताप अधिक प्राप्त होना
(b) घनी वृष्टि होना
(c) नदी का बाँध टूटना
(d) धन-जन को व्यापक हानि पहुँचानेवाली आकस्मिक दुर्घटना
49. आपदा प्रबंधन किसे कहते हैं ?
(a) भूकंप को रोकना
(b) सुनामी उत्पन्न नहीं होने देना
(c) प्राकृतिक आपदाओं के दुष्परिणामों को कम करने का उपाय किया जाना
(d) स्वच्छ पेयजल का प्रबंध करना
50. इनमें कौन विस्तृत क्षेत्र को प्रभावित करने वाली आपदा है?
(a) भूकंप
(b) ओजोन परत का क्षरण
(c) ज्वालामुखी
(d) बाढ़
51. इनमें कम विनाशकारी आपदा कौन है ?
(a) भूस्खलन
(b) सूनामी
(c) बाढ़
(d) सूखा
हमसे जुड़ें, हमें फॉलो करे ..
  • Telegram ग्रुप ज्वाइन करे – Click Here
  • Facebook पर फॉलो करे – Click Here
  • Facebook ग्रुप ज्वाइन करे – Click Here
  • Google News ज्वाइन करे – Click Here
]]>
Sat, 13 Apr 2024 05:11:07 +0530 Jaankari Rakho
General Competition | Geography | भारत की जनजातियाँ https://m.jaankarirakho.com/963 https://m.jaankarirakho.com/963 General Competition | Geography | भारत की जनजातियाँ
  • भारतीय संविधान में अनुसूचित जाति और अनुसूचित जनजाति को परिभाषित नहीं किया गया है लेकिन इन जनजातियों की चर्चा हमारे संविधान में है।
  • भारतीय संविधान के अनुच्छेद- 341 में अनुसूचित जाति और अनुच्छेद-342 में अनुसूचित जनजाति की चर्चा की गई है।
  • किसी भी जाति को SC और ST घोषित करने का अधिकार राष्ट्रपति को होता है ।
  • राष्ट्रीय अनुसूचित जाति आयोग का गठन अनुच्छेद- 338 के तहत तो वही राष्ट्रीय अनुसूचित जनजाति आयोग का गठन अनुच्छेद- 338A के तहत हुआ है।
  • हर्बर्ट रिजले ने सर्वप्रथम भारतीय जनसंख्या में प्रजातियों का विवरण प्रस्तुत किया।
  • 1931 ई० में जाति आधारित जनगणना के बाद बी० एस० गुहा ने भारतीय प्रजातियों का विवरण छ: वर्गों में किया है- 1. नीग्रो/निग्रिटो : यह भारत आने वाली पहली प्रजाति है यह प्रजाति अंडमान निकोबार द्वीप समूह में मुख्य तौर पर निवास करती है। 2. प्रोटे ऑस्ट्रेलायड 3. अल्पाइन 4. मंगोलायड 5. भूमध्य नगरीय 6. नार्डिक (आर्य)। 7. नार्डिक : सबसे अंतिम में भारत आने वाली प्रजाति नॉर्डिक है।
  • भारत की पहली सभ्यता हड़प्पा सभ्यता में चार प्रजाति के लोग निवास करते थे जो निम्न हैं- 1. प्रोटे ऑस्ट्रेलायड 2. अल्पाइन 3. भूमध्यसागरीय 4. मंगोलॉयड।
  • अनुसूचित जाति (SC) : वैदिक काल में अनुसूचित जाति के लिए चाण्डाल शब्द का प्रयोग किया जाता था। भारत वर्ष में साइमन कमीशन आयोग न पहली बार अनुसूचित जाति शब्द का प्रयोग किया। भारत शासन अधिनियम 1935 में अछूतों के लिए अनुसूचित जाति शब्द का प्रयोग किया गया। गिरिजन शब्द का प्रयोग अनुसूचित जातियों के लिए ही किया गया था। लेकिन महात्मा गाँधी ने गिरिजन शब्द के स्थान पर हरिजन शब्द का प्रयोग किया।
  • अनुसूचित जनजाति : भारत में आदिवासी समुदाय के लोगों के लिए अनुसूचित जनजाति शब्द का प्रयोग किया जाता है। आदिवासी शब्द का प्रयोग सर्वप्रथम ठक्कर बप्पा ने किया है जिस कारण उन्हें आदिवासी का मसीहा कहा जाता है। आदिवासी भारत के मूल निवासी है। भारत की कुल आबादी का मध्य प्रदेश में पाया जाता है वही प्रतिशतता के सवासी आबादी है। भारत में सबसे ज्यादा आदिवासी आबादी आदिवासी आबादी मिजोरम में पाया जाता है ।
  • युवा गृह : आदिवासी समुदाय के लोगों के शैक्षणिक संस्थान को युवा गृह कहा जाता है। मुरिया जनजाति के लोग इसे घोटुल, उराँव जनजाति के लोग धूमकुरिया तथा बीरहोर जनजाति के लोग इसे गितुओना कहते हैं।
  • भारत की तीन बड़ी जनजातियों का क्रम निम्न है- भील > गोंड > संथाल |
  • प्रमुख जनजातियाँ
  • टोडा : वह तमिलनाडु के नीलगिरि पहाड़ी क्षेत्र में पायी जाने वाली जनजातियाँ हैं। इस जनजाति के लोग पशुपालन किया करते हैं। इस जनजाति में बहुपति प्रथा का प्रचलन देखने को मिलता है।
  • भील : भील शब्द की उत्पत्ति द्रविड़ भाषा के बील से हुई है जिसका अर्थ धनुषधारी होता है। इस जनजाति के लोग अपने आप को महादेव का संतान बताते हैं। इस जनजाति में संयुक्त परिवार का प्रचलन है । इस जनजाति के लोग कृषक होते हैं। इस जनजाति के लोगों के द्वारा भीली भाषा बोली जाती है। इस जनजाति का संबंध प्रोटो ऑस्ट्रेलायड प्रजाति से है। यह जनजाति निम्नलिखित राज्य जैसे-महाराष्ट्र, राजस्थान, गुजरात, मध्य प्रदेश, आंध्र प्रदेश इत्यादि राज्यों में पायी जाती है।
  • मीणा : यह राजस्थान में पायी जाने वाली प्रमुख जनजाति इस जनजाति के लोग अपने आप को भगवान विष्णु के मीण अवतार से जोड़कर दिखाता है।
  • गोंड : इस जनजाति का संबंध द्रविड़ प्रजाति से है। इस जनजाति के लोग गोंडी भाषा बोलते हैं। यह भारत की दूसरी सबसे बड़ी जनजाति है। यह जनजाति भूत-प्रेत में अत्यधिक विकास करता है। यह जनजाति भारत के 13 राज्यों में निवास करती है जैसे - मध्य प्रदेश, छत्तीसगढ़, ओडिशा, महाराष्ट्र, तेलंगाना, आंध्र प्रदेश, कर्नाटक, गुजरात इत्यादि ।
  • संथाल : यह भारत की तीसरी सबसे बड़ी जनजाति है । इस जनजाति के लोग कृषक और शिकारी होते हैं। इसका संबंध प्रोटो ऑस्ट्रेलायड प्रजाति से है। यह जनजाति मुख्यत: झारखंड राज्य में निवास करती हैं इसके अलावे यह जनजाति बिहार, पश्चिम बंगाल, मध्य प्रदेश इत्यादि राज्यों में निवास करती है। इस जनजाति के लोग नाटे कद, चौड़े और चपटी नाकों वाले होते हैं।
  • मुंडा : इस जनजाति का प्रमुख क्षेत्र झारखंड राज्य है। इस जनजाति का ही सबसे प्रमुख त्योहार सरहुल हैं।
  • थारू : यह जनजाति मुख्यत: उत्तराखण्ड और उत्तर प्रदेश राज्यों में पायी जाती है। इस जनजाति के लोगों का संबंध हिंदू धर्म से है। इस जनजाति के लोग हिंदू धर्म के सभी त्योहारों को मनाते है । इस जनजाति के लोग दीवाली को पहले शोक पर्व के रूप में मनाते थे लेकिन अब शोक पर्व के रूप में नहीं मनाते हैं।
  • बुक्सा : यह जनजाति मूलत: उत्तराखंड राज्य में निवास करती है। इस जनजाति का संबंध राजपूत घरानों से है। इस जनजाति -के लोग हिंदी बोलते हैं। इस जनजाति में अनुलोम विवाह और प्रतिलोम विवाह का प्रचलन देखने को मिलता है।
  • जौनसारी : यह जनजाति मुख्यत: उत्तराखंड और उत्तर प्रदेश में पायी जाती है। इस जनजाति का संबंध भूमध्यसागरीय प्रजाति से है। इस जनजाति में बहुपति विवाह का प्रचलन देखने को मिलता है ।
  • नागा : यह मूलत: नागालैंड में पायी जाने वाली जनजाति है। नागालैंड के अलावा यह जनजाति पूर्वोत्तर भारत के अन्य राज्य जैसे - असम, मणिपुर राज्यों में निवास करती है। इस जनजाति के लोग नागमिस भाषा बोला करते हैं। नागा भाषा और असमिया भाषा के सम्मिश्रण से नागमिस भाषा का विकास हुआ है।
  • Note : मीथन नागा जनजाति का त्योहार हैं।
  • भूटिया / भोटिया : यह जनजाति मुख्यतः पर्वतीय राज्य जैसे- सिक्किम, अरूणाचल प्रदेश, उत्तराखंड में पायी जाती है। यह पशुपालक जनजाति होती है।
  • Note : ऋतु प्रवास भोटिया / भूटिया जनजाति में पायी जाती है।
  • बोडो जनजाति : यह मूलत: असम में पायी जाने वाली जनजाति है। यह जनजाति बोडी नामक भाषा बोलती है। इस जनजाति को 6ठी अनुसूची में शामिल किया गया है।
राज्य जनजाति
अरूणाचल प्रदेश डाफला, अबोर, मीशमी, आपतानी, अकावट
नागालैंड नागा, अंगामी, कूकी, मिकिर
मणिपुर कूकी, माओ, मारिंग, कोम
असम चकमा, डिमासा
मिजोरम मिजो, लाखेर, पाबी, चकमा
मेघालय गारो, खासी, जैनतिया
त्रिपुरा त्रिपुरी, रियांग, ओरांग
अंडमान निकोबार द्वीप समूह सेंटिलिज, जारबा, जोरबे, सोम्पैन, अंडमानी, निकोबारी, टाबू, येरे
सिक्किम लेपचा, तमांग, शेरपा, भूटिया
जम्मू-कश्मीर एक्करवाल, गुज्जर, गद्दी
हिमाचल प्रदेश गद्दी, किन्नौर, श्वांगला
झारखंड संथाल, मुंडा, हो, उराँव, बीरहोर, खड़िया, असुर, मालपहाड़िया
राजस्थान मीणा, सहारिया, गरसिया
महाराष्ट्र बरदा, भील, कोल, वर्दी
गुजरात भील, बंजारा, पटेलिया
मध्य प्रदेश भील, अगरिया, खैरबार, कोल, कार्कू, कमार
छत्तीसगढ़ धनवार, खड़िया, अगरिया
केरल इरुला, पनियान
बिहार कौल, मुंडा, हौ, उराँव

वस्तुनिष्ठ प्रश्न एवं उत्तर

1. " ........ ये पीले वर्ण के, तिर्यक नेत्र, उठी हुई कपोल अस्थि, छुटपूट केश और मध्यम ऊंचाई वाले व्यक्ति होते हैं।" इसका संदर्भ है-
(a) नॉर्डिक आर्यों से
(b) ऑस्ट्रियाई जनों से
(c) नीग्रोसम जनों से
(d) मंगोलायड जनों से
2. उत्तर-पूर्वी भारत के पहाड़ी एवं जंगली क्षेत्रों में निम्नलिखित में से कौन-सा एक प्रजातीय समूह पाया जाता है ? 
(a) दिनारिक 
(b) मेडिटेरेनियन
(c) मंगोलायड
(d) प्रोटो-ऑस्ट्रेलॉयड
3. भारत की निम्नलिखित में से कौन-सी जनजाति प्रोटो-ऑस्ट्रेलॉयड प्रजाति से संबंधित है ? 
(a) इरुलर 
(b) खासी
(c) संथाल
(d) थारू
4. भारत के निम्नलिखित भागों में द्रविड़ियन प्रजाति मुख्यतः कहाँ संकेंद्रित है ?
(a) दक्षिण भारत
(b) उत्तर-पश्चिम भारत
(c) उत्तर-पूर्वी भारत
(d) उत्तर भारत
5. भारत में जो एकमात्र मानवाभ कपि पाया जाता है, वह है -
(a) हनुमान बंदर
(b) पश्चिमी घाट सिंह- पुच्छी काक
(c) असम का मनथर लोरीस
(d) असम का श्वेतभौं गिबन
6. निम्नलिखित में से किस राज्य में जनजातीय समुदाय की पहचान नहीं की गई है ?
(a) महाराष्ट्र
(b) छत्तीसगढ़
(c) हरियाणा
(d) कर्नाटक
7. कौन-सी जनजाति दीवाली को शोक का त्योहार मानती है ?
(a) खासी
(b) मुंडा
(c) भील
(d) थारू
8. भारत के निम्नलिखित में से किन राज्यों में “थारू जनजाति" निवास कर रही है ?
(a) बिहार तथा मध्य प्रदेश 
(b) झारखंड तथा मध्य प्रदेश
(c) छत्तीसगढ़ तथा हिमाचल प्रदेश
(d) उत्तराखंड तथा उत्तर प्रदेश
9. निम्न में से कौन धौलाधार श्रेणी क्षेत्र की प्रमुख जनजाति है ?
(a) अबोर
(b) गद्दी
(c) लेप्चा
(d) थारू
10. गद्दी (Gaddi ) लोग निवासी हैं- 
(a) मध्य प्रदेश के
(b) हिमाचल प्रदेश के
(c) अरुणाचल प्रदेश के
(d) मेघालय के
11. संथाल निवासी हैं- 
(a) मध्य भारत के 
(b) दक्षिणी भारत के
(c) पश्चिमी भारत के
(d) पूर्वी भारत के
12. निम्नांकित में कौन सुमेलित नहीं है ? 
(a) भील - गुजरात 
(b) जौनसारी- उत्तराखंड
(c) संथाल - छत्तीसगढ़
(d) खासी - मेघालय
13. संथालो में विवाह का सबसे सामान्य रूप कौन-सा है ?
(a) इतूत
(b) सांगा
(c) निर-बलोक
(d) बुपला
14. ऋतु प्रवास किया करते हैं-
(a) भूटिया
(b) भुक्सा
(c) जौनसारी
(d) थारू
15. बोडो निवासी ( Inhabitants) हैं- 
(a) गारो पहाड़ी के
(b) संथाल परगना के
(c) अमेजन बेसिन के
(d) मध्य प्रदेश के
16. निम्नलिखित जनजातियों में से कौन-सी केरल में पाई जाती है ? 
(a) चेंचू
(b) लेप्चा
(c) डफला
(d) डाफर
17. भारत की सर्वाधिक बड़ी जनजाति है- 
(a) भील 
(b) गोंड
(c) संथाल
(d) थारू
18. टोडा एक जनजाति है, जो निवासी करती है - 
(a) अरावली पहाड़ियों पर
(b) मध्य प्रदेश
(c) नीलगिरि की पहाड़ियों पर
(d) विंध्याचल की पहाड़ियों पर
19. निम्नलिखित क्षेत्रों में से कौन 'टोडा जनजाति' का मूल निवास क्षेत्र है ?
(a) जौनसार पहाड़ियाँ
(b) गारो पहाड़ियाँ
(c) नीलगिरि पहाड़ियों
(d) जयंतिया पहाड़ियाँ
20. 'अंडी' और 'ओपोरतीपि' नाम से प्रचलित विवाह किस आदिवासी समुदाय से संबंधित है ? 
(a) हो
(b) पहाड़ियाँ
(c) मुंडा
(d) उरांव
21. निम्नलिखित जनजातियों में कौन बहुपति विवाह की प्रथा को मानता है ?
(a) कादर
(b) लोढ़ा
(c) मुंडा
(d) टोडा
22. 'सरहुल त्यौहार ' कौन-सी जनजाति मनाते हैं ?
(a) उरांव 
(b) कमार
(c) बैगा
(d) गोंड
23. 'घुमकरिया' किस जनजाति की सामाजिक संस्था है ?
(a) उरांव
(b) हो
(c) गोंड
(d) कोल
24. उत्तराखंड की सबसे बड़ी अनुसूचित जनजाति है-
(a) भोक्सा
(b) भोटिया
(c) जौनसारी
(d) थारू
25. मिजोरम में बस्ती मुख्यतः कटकों के साथ-साथ रैखिक प्रतिरूप' का है, क्योंकि-
(a) घाटियां कटकों की अपेक्षा ठंडी हैं
(b) कटक शिखरों पर पहुँचना सरल हैं।
(c) कटक घाटियों की अपेक्षा ठंडे हैं।
(d) घाटियों में सघन वन हैं
26. निम्नलिखित में से कौन-सा एक सही सुमेलित नहीं है ?
(a) भोटिया उत्तर प्रदेश
(b) खासी मेघालय
(c) संथाल - झारखंड
(d) तोडा तमिलनाडु
27. निम्नलिखित में कौन-सा एक सुमेलित नहीं है ?
(a) बिहू - असम 
(b) ओणम - आंध्र प्रदेश
(c) पोंगल - तमिलनाडु
(d) बैसाखी - पंजाब
28. भील जाति कहा पाई जाती है ?
(a) असम
(b) झारखंड
(c) पश्चिम बंगाल
(d) महाराष्ट्र
29. सहरिया जनजाति के लोग, जो हाल में चर्चा में थे, कहां के निवासी हैं ?
(a) आंध्र प्रदेश
(b) असम
(c) राजस्थान
(d) ओडिशा
30. भारत में जनजातियों के निर्धारण का क्या आधार है ?
(a) सांस्कृतिक विशेषीकरण और विभिन्न आवास
(b) भाषा और बोली
(c) सामाजिक रीति-रिवाज की विभिन्नताएं
(d) आर्थिक स्तर
31. निम्नलिखित कथनों में से कौन-सा सही नहीं है ? 
(a) भारत के संविधान में अनुसूचित जनजाति की कोई परिभाषा नहीं है।
(b) देश की कुल जनजाति जनसंख्या के आधे से कुछ अधिक संख्या उत्तर-पूर्वी भारत में है।
(c) टोडा कहलाने वोल लोग नीलगिरि क्षेत्र में रहते हैं 
(d) नगालैंड में बोली जाने वाली भाषाओं में से लोथा एक है।
32. भारत के 'चांग्पा' के संदर्भ में निम्नलिखित कथनों पर विचार कीजिए-
1. वे मुख्यत: उत्तराखंड राज्य में रहते हैं।
2. वे अच्छे किस्म का ऊन देने वाले पश्मीना बकरों-बकरियों को पालते हैं।
3. उन्हें अनुसूचित जनजातियों की श्रेणी में रखा जाता है।
उपर्युक्त में से कौन - सा /से कथन सत्य है/हैं ?
(a) केवल 1 
(b) केवल 2 और 3 
(c) केवल 3 
(d) 1, 2 और 3
33. नीचे लिखे राज्यों और जनजातियों के युग्मों में से कौन-सा एक सही सुमेलित नहीं है ?
(a) असम - मीरी 
(b) नगालैंड – कोन्यक 
(c) अरूणाचल प्रदेश - अप्तानी
(d) मध्य प्रदेश - लंबाडा
34. निम्नलिखित में से कौन एक सुमेलित नहीं है ?
(a ) शेरपा - नेपाल
(b) थारू - उत्तराखंड
(c) टोडा - दक्षिण भारत
(d) जुलू - ओडिशा
35. निम्नलिखित जनजातियों में से कौन-सा भारत के अंडमान और निकोबार द्वीपसमूह की मूल निवासी नहीं है ?
(a) हालचू
(b) रेंगमा
(c) ओन्जे
(d) शोम्पेन
36. निम्नलिखित केंद्रशासित प्रदेशों में से ओंजे जनजाति के लोग किसमें रहते हैं ?
(a) अंडमान और निकोबार द्वीपसमूह
(b) दादरा और नगर हवेली
(c) दमन और दीव
(d) लक्षद्वीप
37. निम्नलिखित स्थानों में से कहां शोम्पेन जनजाति पाई जाती है ?
(a) नीलगिरि पहाड़ियां
(b) निकोबार द्वीपसमूह 
(c) स्पीति घाटी
(d) लक्षद्वीप द्वीपसमूह
38. 'जारवा जनजाति' पाई जाती है-
(a) अरूणाचल प्रदेश
(b) मेघालय
(c) मिजोरम
(d) निकोबार द्वीप
हमसे जुड़ें, हमें फॉलो करे ..
  • Telegram ग्रुप ज्वाइन करे – Click Here
  • Facebook पर फॉलो करे – Click Here
  • Facebook ग्रुप ज्वाइन करे – Click Here
  • Google News ज्वाइन करे – Click Here
]]>
Thu, 11 Apr 2024 11:03:54 +0530 Jaankari Rakho
General Competition | Geography | भारत की जनसंख्या https://m.jaankarirakho.com/962 https://m.jaankarirakho.com/962 General Competition | Geography | भारत की जनसंख्या
  • किसी निश्चित क्षेत्र में निश्चित समय में लोगों की संख्या उस क्षेत्र की जनसंख्या कहलाती है । 
  • आधुनिक विश्व में सुव्यवस्थित और वैज्ञानिक ढंग से जनगणना कराने वाला विश्व का पहला देश स्वीडन बना, जो 1749 ई० में जनगणना करवाया।
  • प्रत्येक 10 वर्ष पर जनगणना होनी चाहिए अर्थात् दशकीय जनगणना की शुरूआत करने वाला विश्व का पहला देश अमेरिका बना जो 1790 ई० से दर्शकीय जनगणना की शुरूआत की।
  • ब्रिटेन में पहली बार जनगणना 1801 ई० में हुआ।
  • ब्रिटिश सरकार द्वारा भारत में सर्वप्रथम जनगणना वायसराय लॉर्ड मेयो के काल में 1872 ई० में हुआ।
  • भारत में नियमित रूप से प्रत्येक 10 वर्ष जनगणना की शुरुआत 1881 ई० से हुआ है।
  • जनगणना के इतिहास में वर्ष 1921 को महान विभाजक वर्ष की संज्ञा दी जाती है। क्योंकि 1911 से 1921 के दौरान जनसंख्या में कमी दर्ज की गई थी। 
  • भारत में पहली बार जाति आधारित जनगणना 1931 ई० में हुई है।
  • स्वतंत्र भारत की पहली जनगणना 1951 ई. में हुई है।
  • 2011 की जनगणना भारत की 15वीं तथा स्वतंत्र भारत की 7वीं जनगणना है।
  • भारत में पशुओं की गणना की शुरूआत 1919 ई० से हुई है। पशुओं की गणना प्रत्येक पाँच वर्ष पर होती है। स्वतंत्र भारत की प्रथम पशु गणना 1951 में हुई है।
  • Note : भारत में प्रथम आर्थिक गणना 1977 ई॰ में हुआ है।
  • भारत में वनों को लेकर रिपोर्ट जारी फॉरेस्ट सर्वे ऑफ इंडिया के द्वारा 1987 ई॰ से प्रत्येक 2 वर्ष पर किया जा रहा है।
  • राष्ट्रीय जनसंख्या नीति : भारत विश्व का पहला देश बना जिसने परिवार नियोजन को अपनाया। भारत ने 1952 ई० में परिवार नियोजन को अपनाया। 
  • भारत ने पहली राष्ट्रीय जनसंख्या नीति 1976 ई० में घोषित किया है।
  • भारत की नई जनसंख्या नीति 2000 ई० में आई है जिसके तहत निम्न लक्ष्य निर्धारित किये गये हैं-
    1. 2045 तक जनसंख्या स्थिरता के लक्ष्य को प्राप्त कर लेना है।
    2. 2010 तक कुल प्रजनन दर (Total fertility rate) 2.1 तक प्राप्त कर लेना है। लेकिन 2018 ई. तक कुल प्रजनन दर 2.2 था।
    3. 80% तक प्रसव अस्पताल में होंगे।
    4. लड़कियों का विवाह 18 से कम उम्र तथा लड़कों का विवाह 21 वर्ष से कम उम्र में नहीं किया जायेगा।
  • जनसंख्या 2011 : 2011 की जनगणना के अनुसार भारत की कुल आबादी लगभग 121 करोड़ बताई गई है जो कि विश्व की कुल आबादी का 17.5% है।
  • जनघनत्व (Population density) : प्रति वर्ग किमी० क्षेत्र में पाये जाने वाले लोगों की कुल संख्या जनघनत्व कहलाता है।

  • 2011 की जनगणना के अनुसार भारत का जनघनत्व 382 व्यक्ति प्रति वर्ग किमी है। वही 2001 के जनगणना के अनुसार भारत का जनघनत्व 325 व्यक्ति प्रति वर्ग किमी था । अर्थात् हम कह सकते हैं कि 2001 की तुलना में 2011 में जनघनत्व में 57 की वृद्धि दर्ज की गई है।
  • सर्वाधिक जन वाला राज्य बिहार ( 1106 व्यक्ति प्रति वर्ग किमी) हैं वही सबसे कम जनघनत्व वाला राज्य अरूणाचल प्रदेश है (17 व्यक्ति प्रति वर्ग किमी ) ।
  • केन्द्रशासित प्रदेशों में सर्वाधिक जनघनत्व वाला केन्द्रशासित प्रदेश दिल्ली (11320 व्यक्ति प्रति वर्ग किमी) हैं वही सबसे कम जनघनत्व वाला केन्द्रशासित प्रदेश अंडमान निकोबार द्वीप समूह (46 व्यक्ति प्रति वर्ग किमी) है।
  • सर्वाधिक जनघनत्व वाले राज्यों का क्रम (घटते क्रम में ) : - 1. बिहार ( 1106) 2. पश्चिम बंगाल ( 1028 ) 3. केरल (860)।
  • न्यूनतम जनघनत्व वाले राज्यों का क्रम (बढ़ते क्रम में) : - 1. अरुणाचल प्रदेश (17) 2. मिजोरम ( 52 ) 3. सिक्किम ( 86 ) ।
  • लिंगानुपात ( Sex ratio) : प्रति 1000 पुरुष पर महिलाओं की कुल संख्या लिंगानुपात कहलाती है।
  • 2011 की जनगणना के अनुसार भारत का सर्वाधिक लिंगानुपात वाला राज्य केरल (1084 महिलायें / 1000 पुरुष ) हैं वही न्यूनतम लिंगानुपात वाला राज्य हरियाणा (879 महिलायें / 1000 पुरुष ) है।
  • सर्वाधिक लिंगानुपात वाला केन्द्रशासित प्रदेश पुडुचेरी (1037 महिलायें / 1000 पुरुष ) हैं वही सबसे कम लिंगानुपात वाला केन्द्रशासित प्रदेश दमन और दीव (618 महिलायें / 1000 पुरुष ) है।
  • न्यूनतम लिंगानुपात वाले राज्य य (बढ़ते क्रम में) :- 1. हरियाणा (879), 2. जम्मू कश्मीर (889), 3. सिक्किम (890) है।
  • 2011 के जनगणना के अनुसार भारत का लिंगानुपात 943 महिला प्रति 1000 पुरुष है वही 2001 में लिंगानुपात 933 महिला प्रति 1000 पुरुष हुआ करता था अर्थात् 2001 की तुलना में 2011 में लिंगानुपात में 10 की वृद्धि दर्ज की गई है।
  • Note : भारत के पड़ोसी राष्ट्रों में चीन का लिंगानुपात (926), पाकिस्तान (943), बांग्लादेश (978), नेपाल (1014), श्रीलंका (1034), म्यांमार (1048) है।
  • 2011 की जनगणना के समय मुख्य जनगणना आयुक्त डी० सी० चन्द्रमौली थे। इस जनगणना की शुभंकर प्रगणक शिक्षिका थी । इस जनगणना का आदर्श वाक्य "हमारी जनगणना हमारा भविष्य" (Our census our future) था। इस जनगणना में कुल 29 प्रश्न पूछे गये थे ।
  • साक्षरता दर (Literacy rate ) भारत वर्ष में 7 वर्ष या उससे अधिक उम्र के कोई भी व्यक्ति किसी एक भाषा में पढ़ना और लिखना दोनों जानता हो, तो उसे साक्षर माना जाता है।
  • 2011 की जनगणना के अनुसार भारत की साक्षरता दर 74.04% है। इसमें पुरुष साक्षरता 82.14% जबकि महिला साक्षरता 65.46% है। 2011 की जनगणना के अनुसार महिला साक्षरता और पुरुष साक्षरता में अंतर 16.68% का है।
  • आजादी के समय भारत की साक्षरता दर 18.33% था, जिसमें पुरुष साक्षरता 27% तथा महिला साक्षरता लगभग 8% था।
  • 2011 की जनगणना के अनुसार भारत का सबसे अधिक साक्षर राज्य केरल है जबकि सबसे कम साक्षर राज्य बिहार है वही केन्द्र शासित प्रदेशों में सबसे अधिक साक्षर केन्द्रशासित प्रदेश लक्षद्वीप तथा सबसे कम साक्षर केन्द्रशासित प्रदेश दादर नागर हवेली है।
  • महिला साक्षरता और पुरुष साक्षरता दोनों में सर्वोच्च स्थान पर केरल है जबकि महिला साक्षरता और पुरुष साक्षरता में सबसे निचले पायदान पर बिहार है।
  • धर्म आधारित जनगणना 2011 की जनगणना के दौरान धार्मिक जानकारियाँ भी एकत्रित की गई जिसका प्रकाशन 2015 में हुआ। इस रिपोर्ट के अनुसार भारत में सर्वाधिक हिन्दू आबादी (79.8%) है। यह रिपोर्ट हमें यह जानकारी प्रदान करता है कि सर्वाधिक हिंदू वाला राज्य हिमाचल प्रदेश है जबकि सबसे अधिक मुस्लिम आबादी वाला राज्य असम है। इस रिपोर्ट के मुताबिक 2001 की तुलना में 2011 में हिन्दुओं की जनसंख्या में 0.7% की कमी गिरावट दर्ज की गई है वही मुस्लिम आबादी में 0.8% की वृद्धि दर्ज की गई है।
धर्म जनसंख्या % लिंगानुपात
हिन्दू धर्म 79.8% 931
मुस्लिम 14.2% 936
ईसाई 2.3% 1006
सिक्ख 1.7% 893
बौद्ध 0.7% 953
जैन 0.4% 940
  • Note : 1. सर्वाधिक लिंगानुपात वाला धर्म ईसाई धर्म है वही सबसे कम लिंगानुपात वाला धर्म सिख धर्म है।
    2. भारत में 15 से 64 आयु वर्ग के लोगों को उत्पादक वर्ग की श्रेणी में रखा जाता है। 
  • निर्भरता अनुपात : उत्पादक वर्ग के ऊपर अनुत्पादक वर्ग के निर्भरता की निर्भरता अनुपात कहते हैं।
नगरीय क्षेत्र (Urban Area) :
  • शहर / नगर (City) : वैसे क्षेत्र जहाँ की आबादी 1 लाख से अधिक हो शहर/नगर कहलाता है।
  • महानगर (Metropolitan city ) : वैसे शहर / नगर जहाँ की आबादी 10 लाख से अधिक हो महानगर कहलाता है। 2011 की जनगणना के अनुसार भारत में कुल 53 महानगर हैं जिसमें सबसे ज्यादा महानगर उत्तर प्रदेश और केरल (7) में है। बिहार में एकमात्र महानगर पटना है।
  • मेगा सिटी (Mega city) : वैसे महानगर जिसकी आबादी 1 करोड़ से अधिक हैं, मेगा सिटी कहलाती है। 2011 की जनगणना के अनुसार भारत में 3 मेगा सिटी है- 1. मुंबई, 2. दिल्ली, 3. कलकत्ता ।
  • 2011 की जनगणना के अनुसार भारत की कुल आबादी का 68.8% लोग गाँव में निवास करता है जबकि 31.2% लोग शहर में निवास करता है।
  • प्रतिशतता के आधार पर सर्वाधिक ग्रामीण जनसंख्या वाला राज्य हिमाचल प्रदेश है।
  • प्रतिशतता के आधार पर सर्वाधिक शहरी जनसंख्या वाला राज्य गोवा है।
  • संख्या के आधार पर सर्वाधिक शहरी जनसंख्या वाला राज्य महाराष्ट्र है तथा न्यूनतम शहरी जनसंख्या वाला राज्य सिक्किम है।
  • अनुसूचित जाति और अनुसूचित जनजाति : 2011 की जनगणना के अनुसार भारत की कुल आबादी का 16.6% अनुसूचित जाति तथा 8.6% अनुसूचित जनजाति निवास करती है।
  • भारत में 2 राज्य अरूणाचल प्रदेश और नागालैंड हैं तथा 2 केन्द्रशासित प्रदेश अंडमान निकोबार द्वीप समूह और लक्षद्वीप ऐसे हैं जहाँ SC जनसंख्या निवास नहीं करती है।
  • भारत में 2 राज्य पंजाब और हरियाणा तथा 3 केन्द्रशासित प्रदेश चंडीगढ़, दिल्ली और पुडुचेरी हैं जहाँ अनुसूचित जनजाति निवास नहीं करती है।
  • भारत में अनुसूचित जाति में लिंगानुपात 945 महिला प्रति 100  पुरुष तथा अनुसूचित जनजाति में लिंगानुपात 990 महिला प्रति 1000 पुरुष है।
  • प्रतिशतता के आधार पर सर्वाधिक SC की जनसंख्या वाला राज्य पंजाब है वही संख्या के आधार पर सर्वाधिक SC वाला राज्य उत्तर प्रदेश है।
  • प्रतिशतता के आधार पर सर्वाधिक ST वाला राज्य मिजोरम है वही संख्या के आधार पर ST वाला राज्य मध्य प्रदेश है।
  • दशकीय वृद्धि दर : दस वर्षों में होने वाला जनसंख्या में प्रतिशतता वृद्धि को दशकीय वृद्धि दर कहा जाता है।

  • 2011 की जनगणना के अनुसार भारत की दशकीय वृद्धि दर 17.7% दर्ज की गई है।
  • सर्वाधिक दशकीय वृद्धि दर वाला राज्य मंत्रालय है वही सबसे कम दशकीय वृद्धि दर वाला राज्य नागालैंड है !
  • शिशु लिंगानुपात : जन्म से 6 आयु वर्ग के अंतर्गत 1000 बालक शिशुओं पर बालिका शिशुओं की संख्या शिशु लिंगानुपात कहलाता है।
  • भारत में सर्वाधिक शिशु लिंगानुपात वाला राज्य अरुणाचल प्रदेश (972) जबकि सबसे कम शिशु लिंगानुपात वाला राज्य हरियाणा (834) है।
  • सर्वाधिक शिशु लिंगानुपात वाला केन्द्रशासित प्रदेश अंडमान निकोबार द्वीप समूह (968) है तथा सबसे कम शिशु लिंगानुपात वाला केन्द्रशासित प्रदेश दिल्ली (871) है।
  • शिशु मृत्यु दर : प्रति वर्ष 1000 जीवित शिशुओं के जन्म पर एक वर्ष के भीतर मरने वाले शिशुओं की औसत संख्या शिशु मृत्यु दर कहलाता है। अखिल भारतीय स्तर पर भारत का शिशु मृत्यु दर 32 है। वही सर्वाधिक शिशु मृत्यु दर वाला राज्य मध्य प्रदेश (48) तथा सबसे कम शिशु मृत्यु दर वाला राज्य केरल (7) है।
  • 2011 की जनगणना के अनुसार भारत की कुल आबादी का 51.47% पुरुष आबादी तथा 48.53% महिला आबादी है। इस अनुसार महिला आबादी और पुरुष आबादी में अंतर 2.94% का है।
  • 2011 के जनगणना के अनुसार भारत के 3 सर्वाधिक जनसंख्या वाले राज्य निम्न हैं- 1. उत्तर प्रदेश (16.51%), 2. महाराष्ट्र (9%), 3. बिहार (8%)।
  • न्यूनतम जनसंख्या वाले राज्य सिक्किम, मिजोरम, अरूणाचल प्रदेश है।
  • जॉन माल्थस का जनसंख्या सिद्धांत : ब्रिटिश अर्थशास्त्री जॉन माल्थस ने 1798 में जनसंख्या का सिद्धांत दिया। अपने जनसंख्या सिद्धांत में माल्थस ने कहा कि एक समय ऐसा आयेगा जब खाद्य आपूर्ति की तुलना में मानव जनसंख्या में तेजी से वृद्धि होगा जिस कारण खाद्यान्नों की कमी होगी और अकाल, भूखमरी, बेरोजगारी तथा युद्ध जैसी स्थिति उत्पन्न हो जाएगी। जिससे अचानक जनसंख्या में गिरावट होगी।
  • जिला के संदर्भ में कुछ तथ्य :
  • भारत के दो सबसे अधिक आबादी वाले जिले महाराष्ट्र का थाणे तथा पश्चिम बंगाल का उत्तरी 24 परगना है।
  • न्यूनतम जनसंख्या वाला क्षेत्र या जिला अरूणाचल प्रदेश का दिबांग है।
  • पूरे भारत वर्ष में सर्वाधिक लिंगानुपात वाला जिला पुडुचेरी केन्द्रशासित प्रदेश का माहे (1176) है जबकि न्यूनतम लिंगानुपात वाला जिला दमन (533) है।
  • भारत का सबसे अधिक साक्षर जिला मिजोरम का सेरछिप (99%) तथा सबसे कम साक्षर जिला मध्य प्रदेश का अलीराजपुर (37.22%) है।

वस्तुनिष्ठ प्रश्न एवं उत्तर

1. विश्व की विशाल जनसंख्या वाले देशों में भारत को कौन-सा स्थान प्राप्त है ?
(a) पहला
(b) दूसरा
(c) तीसरा
(d) चौथा
2. इनमें कौन भारत की घनी जनसंख्या का प्रमुख कारक हैं ?
(a) मॉनसूनी जलवायु
(b) उद्योगों का विकास
(c) बहुत दिनों तक ब्रिटिश हुकूमत
(d) बड़ी-बड़ी नदियों का मिलना
3. इनमें कहाँ जनसंख्या का घनत्व अधिक मिलता है ?
(a) उत्तर प्रदेश
(b) बिहार
(c) गुजरात
(d) कर्नाटक
4. असम में अधिक वर्षा होते हुए भी जनसंख्या का घनत्व कम क्यों है ?
(a) मरुस्थल के कारण
(b) जलवायु सर्द होने के कारण
(c) ऊँचे-ऊँचे पहाड़ों के कारण
(d) वन, पहाड़ी भूमि और अस्वास्थ्यकर जलवायु के कारण
5. पंजाब और हरियाणा में वर्षा कम होते हुए भी घनी जनसंख्या मिलने का क्या कारण है ?
(a) उपजाऊ मिट्टी का मिलना
(b) सिंचाई की सुविधा
(c) रेलमार्ग का होना
(d) खनिज सम्पदा का मिलना
6. पश्चिम बंगाल में घनी जनसंख्या क्यों मिलती है ?
(a) यहाँ कृषि की सारी सुविधाएँ उपलब्ध है
(b) यहाँ धान की उपज साल में दो बार ली जाती है
(c) यहाँ छोटे-बड़े उद्योगों का विकास हुआ है
(d) इनमें सभी
7. पश्चिमी पठार में घनी जनसंख्या नहीं मिलने का क्या कारण है ?
(a) यहाँ धान नहीं उपजता है
(b) यहाँ खनिजों का अभाव है
(c) यहाँ व्यापार की सुविधाएँ नहीं है
(d) यहाँ वर्षा का अभाव है और जमीन ऊँची-नीची है।
8. छोटानागपुर पठार पर यत्र-तत्र घनी जनसंख्या क्यों मिलती है ?
(a) वहाँ वैज्ञानिक कृषि की जाती है
(b) वहाँ सोना और चाँदी जैसी बहुमूल्य धातुएँ मिलती है
(c) वहाँ उद्योगों का विकास हुआ है।
(d) वहाँ रेलमार्गों का जाल बिछा हुआ है
9. केरल में घनी जनसंख्या क्यों मिलती है ?
(a) वहाँ शिक्षा का अत्यधिक प्रसार है
(b) वहाँ के लोगों का जीवन-स्तर ऊँचा है
(c) वहाँ कृषि एवं उद्योगों का विकास हुआ है
(d) इनमें सभी
10. जनसंख्या की प्राकृतिक वृद्धि में कौन सहायक है ?
(a) लोगों का कार्यनिपुण होना
(b) व्यापार में वृद्धि होना
(c) जन्म-दर का अधिक होना
(d) आर्थिक संसाधनों का विकास न होना
11. ग्रामीण जनसंख्या का प्रतिशत पिछले सौ वर्षों में भरकर कितना आ गया है ?
(a) 89 प्रतिशत
(b) 82 प्रतिशत
(c) 68.8 प्रतिशत
(d) 62 प्रतिशत
12. किस राज्य की जनसंख्या में औरतों का अनुपात अधिक है?
(a) बिहार 
(b) पश्चिम बंगाल
(c) केरल
(d) उत्तर प्रदेश
13. भारत में सबसे अधिक जनसंख्या किस राज्य में मिलती है ?
(a) मध्य प्रदेश
(b) उत्तर प्रदेश
(c) पश्चिम बंगाल
(d) बिहार
14. भारत में सबसे घनी जनसंख्या किस राज्य में मिलती है ?
(a) उत्तर प्रदेश
(b) बिहार
(c) पश्चिम बंगाल
(d) केरल
15. निम्नांकित केंद्रशासित प्रदेशों में सबसे कम जनसंख्या किसकी है ?
(a) पुदुचेरी
(b) लक्षद्वीप
(c) अंडमान और निकोबार द्वीप समूह
(d) दादरा और नगर हवेली और दमन और दीव
16. निम्नांकित में किसका जनसंख्या घनत्व सबसे कम हैं ?
(a) नागालैंड 
(b) मिजोरम
(c) अरुणाचल प्रदेश
(d) मणिपुर
17. जनसंख्या वितरण के प्रतिरूपों को निम्नलिखित में से कौन-सा/से भौतिक कारक निर्धारित करता है / करते हैं?
(a) भू-विन्यास 
(b) जल की उपलब्धता
(c) जलवायु
(d) उपर्युक्त सभी
18. निम्नलिखित राज्यों में से किस एक में जनसंख्या घनत्व सबसे कम है ?
(a) तमिलनाडु
(b) केरल
(c) अरुणाचल
(d) दिल्ली
19. निम्नलिखित केंद्रशासित राज्यो पिस किस एक राज्यचीजनसंख्या घनत्व सर्वाधिक है ?
(a) अंडमान निकोबार द्वीप समूह
(b) दिल्ली
(c) दादरा और नगर हवेली
(d) लक्षद्वीप
20. निम्नलिखित में से कौन-सा/से कथन सत्य है/हैं?
1. दो समय बिंदुओं के बीच किसी क्षेत्र-विशेष में रहने वाले लोगों की संख्या में परिवर्तन जनसंख्या वृद्धि कहलाती है।
2. प्राकृतिक जनसंख्या वृद्धि का विश्लेषण अशोधित जन्म दर और मृत्यु दरों में निर्धारित किया जाता है।
कूट :
(a) केवल 1
(b) केवल 2
(c) 1 और 2 दोनों
(d) न तो 1 और न ही 2
21. 2001-2011 के दौरान भारत के किस राज्य की जनसंख्या वृद्धि सबसे कम दर्ज की गई है ?
(a) आन्ध्र प्रदेश
(b) ओडिशा
(c) केरल
(d) नागालँड
22. भारत में सबसे बड़। भाषायी समूह है
(a) द्रविड़
(b) चीनी-तिब्बती
(c) भारतीय-यूरोपीय (आर्य)
(d) ऑस्ट्रिक
23. जनसंख्या आँकड़ों के स्रोत के संदर्भ में निम्नलिखित कथनों पर विचार कीजिये :
1. भारत में जनसंख्या के आँकड़ों को प्रति 10 वर्ष बाद होने वाली जनगणना द्वारा एकत्रित किया जाता है।
2. भारत में पहली संपूर्ण जनसंख्या 1881 में संपन्न हुई थी ।
उपर्युक्त में से कौन-सा/से कथन सही है/हैं ?
(a) केवल 1 
(b) केवल 2 
(c) 1 और 2 दोनों
(d) न तो 1 और न ही 2
24. निम्नलिखित में से भारत के किस राज्य की जनसंख्या सर्वाधिक है ?
(a) महाराष्ट्र
(b) बिहार 
(c) पश्चिम बंगाल
(d) उत्तर प्रदेश
25. निम्नलिखित कथनों पर विचार कीजिये :
1. उत्तरी भारत के राज्यों में सर्वाधिक जनसंख्या घनत्व बिहार में है।
2. प्रायद्वीपीय भारत के तमिलनाडु राज्य में सर्वाधिक जनसंख्या घनत्व है।
3. भारत के पर्वतीय राज्यों और उत्तर-पूर्वी राज्यों में निम्न जनसंख्या घनत्व है।
उपर्युक्त में से कौन-सा/से कथन सही नहीं है/है ?
(a) केवल 1 और 2
(b) केवल 2
(c) केवल 2 और 3
(d) 1, 2 और 3
26. भारत में निम्नलिखित में से किस दशक की वृद्धि दर ऋणात्मक दर्ज की गई ? 
(a) 1911-1921
(b) 1901-1911
(c) 1971-1981
(d) 1951-1961
27. भारत के किन दशकों को जनसंख्या की स्थिर वृद्धि की अवधि के रूप में जाना जाता है ?
(a) 1951-1981
(b) 1981-2011
(c) 1921-1951
(d) इनमें से कोई नहीं
28. 2011 की जनगणना के अनुसार भारत की जनसंख्या निम्नलिखित में से कौन-सी है ?
(a) 101.5 करोड़
(b) 108.5 करोड़
(c) 111.5 करोड़
(d) 121.05 करोड़
29. निम्नलिखित राज्यों में से किस राज्य की ग्रामीण जनसंख्या का प्रतिशत सर्वाधिक है ?
(a) उत्तर प्रदेश
(b) ओडिशा
(c) महाराष्ट्र
(d) बिहार
30. केंद्रशासित प्रदेशों में किस एक की ग्रामीण जनसंख्या का प्रतिशत सर्वाधिक है ?
(a) दमन एवं दीव
(b) दादरा और नगर हवेली
(c) अंडमान और निकोबार द्वीप समूह
(d) लक्षद्वीप
31. जनगणना 2011 के अनुसार भारत की ग्रामीण जनसंख्या है
(a) 68.84% 
(b) 64.84%
(c) 66.84%
(d) 62.84%
32. निम्नलिखित में से किसकी जनसंख्या सबसे कम है ?
(a) लक्षद्वीप 
(b) दमन और दीव
(c) सिक्किम
(d) अंडमान और निकोबार द्वीप समूह
33. जनगणना, 2011 के अनुमानों के अनुसार भारत में निम्न में से कौन-सा एक सर्वाधिक जनसंख्या वाला राज्य है ? 
(a) बिहार 
(b) मध्य प्रदेश
(c) महाराष्ट्र
(d) उत्तर प्रदेश
34. निम्नलिखित में से किसको नगरीकरण वक्र में त्वरित अवस्था कहा जाता है ?
(a) प्रथम अवस्था 
(b) द्वितीय अवस्था
(c) तृतीय अवस्था
(d) चतुर्थ अवस्था
35. 'जन्मदर' के संदर्भ में निम्नलिखित कथनों में से कौन-सा से सही है/हैं?
1. नगरीकरण जन्मदर को कम करने में सहायक है।
2. ऊँची साक्षरता दर का निम्न जन्मदर से सीधा सम्बन्ध है।
नीचे दिये हुए कूट से सही उत्तर चुनिये :
कूट :
(a) केवल 1
(b) केवल 2
(c) 1 और 2 दोनों
(d) न तो 1, न ही 2
36. निम्नलिखित जनगणना वर्षों में किसे भारत के जनाकिकीय इतिहास में 'महान विभाजक' के रूप में जाना जात है ?
(a) 1901
(b) 1921
(c) 1931
(d) 1941
37. 2011 की जनगणना के अनुसार किस राज्य का जनसंख्या घनत्व सबसे कम था ?
(a) हिमाचल प्रदेश 
(b) अरुणाचल प्रदेश
(c) पश्चिम बंगाल
(d) मिजोरम
38. वर्ष 2011 की जनगणना के अनुसार किस राज्य में जनसंख्या की दशकीय वृद्धि दर न्यूनतम रही है ?
(a) अरुणाचल प्रदेश 
(b) नागालैंड
(c) मध्य प्रदेश
(d) पंजाब
39. 2011 की जनगणना के अनुसार भारत के किस राज्य के आदिवासियों की आबादी का प्रतिशत उसकी कुल आबादी की तुलना में सबसे अधिक है ?
(a) अरुणाचल प्रदेश 
(b) नागालँड
(c) मिजोरम
(d) मंत्रालय 
40. निम्नलिखित में से किस जनगणना दशक में लिंग अनुपात में भारत वर्ष में सबसे अधिक गिरावट दर्ज की गई ?
(a) 1931-41 
(b) 1961-71
(c) 1981-91
(d) 2001-11
41. 2011 की जनगणना के अनुसार, भारत की कुल जनसंख्या में नगरीय जनसंख्या का प्रतिशत था लगभग
(a) 21 
(b) 31
(c) 36
(d) 40
42. भारतीय जनगणना 2011 के अनुसार निम्न राज्यों में से , किस एक में जनसंख्या घनत्व सर्वाधिक है ?
(a) केरल
(b) हरियाणा
(c) बिहार
(d) उत्तर प्रदेश
43. भारतीय जनगणना 2011 के अनुसार बिहार राज्य में लिंगानुपात क्या है ? 
(a) 893
(b) 916
(c) 918
(d) 925
44. 2011 की जनगणना के अनुसार निम्नलिखित राज्यों में किसमें अनुसूचित जनजातियों की जनसंख्या का प्रतिशत उसके कुल जनसंख्या के प्रतिशत के हिसाब से न्यूनतम है ?
(a) उत्तर प्रदेश
(b) बिहार 
(c) तमिलनाडु
(d) केरल
45. जनगणना 2011 के अनुसार निम्न में से किस राज्य में जनसंख्या में कमी आई है ?
(a) नागालैंड
(b) मणिपुर
(c) त्रिपुरा
(d) सिक्किम
46. भारत में नियमित एवं वैज्ञानिक ढंग से जनगणना किस वर्ष प्रारंभ हुई ?
(a) 1861
(b) 1871
(c) 1881
(d) 1891
47. 2011 की जनगणना के अनुसार भारत में जनसंख्या घनत्व है
(a) 325
(b) 335
(c) 382
(d) 385
हमसे जुड़ें, हमें फॉलो करे ..
  • Telegram ग्रुप ज्वाइन करे – Click Here
  • Facebook पर फॉलो करे – Click Here
  • Facebook ग्रुप ज्वाइन करे – Click Here
  • Google News ज्वाइन करे – Click Here
]]>
Thu, 11 Apr 2024 10:09:29 +0530 Jaankari Rakho
General Competition | Geography | भारत का परिवहन https://m.jaankarirakho.com/961 https://m.jaankarirakho.com/961 General Competition | Geography | भारत का परिवहन
  • किसी व्यक्ति या वस्तु को एक स्थान से दूसरे स्थान तक पहुँचाना परिवहन कहलाता है। भारतीय परिवहन को तीन वर्गों में बाँटा गया है जो निम्न है-
    1. स्थलमार्ग परिवहन
    2. जलमार्ग परिवहन
    3. वायुमार्ग परिवहन
  • भारत का सबसे प्राचीनतम परिवहन का मार्ग जलमार्ग का परिवहन है।
  • सबसे तीव्रतम परिवहन का मार्ग वायुमार्ग परिवहन है।
  • भारत में रेल परिवहन की शुरूआत 16 अप्रैल, 1853 को मुंबई से थाणे के बीच हुआ हैं।
  • भारत में वायु परिवहन की शुरूआत 1911 ई. इलाहाबाद से नैनी के बीच हुआ है।
  • भारत में सड़क मार्ग और जलमार्ग परिवहन की शुरूआत प्राचीन काल (हड़प्पा काल) से हुई मानी जाती है।
  • सबसे ज्यादा रेलवे लाइन का विस्तार उत्तर प्रदेश राज्य में है। (10,324 km)
  • सभी रेलवे जोन या मंडल में सबसे अधिक लंबाई उत्तरी रेलवे (7301 km) का है।
  • रेल गेज : दो समानांतर पटरियों के बीच की दूरी को रेल गेज कहते हैं। भारतीय रेल को तीन गेज में बाँटा गया हैं-
    1. ब्रॉड गेज (बड़ी लाइन ) : इसमें दो पटरियों के बीच की दूरी 1.676m होता है।
    2. मीटर गेज (मीटर लाइन) : इसमें दो पटरियों के बीच की दूरी 1 मीटर होती है ।
    3. नैरो गेज (छोटी लाइन ) : इसमें दो पटरियों के बीच की दूरी 0.762m या 0.610m होता है।
  • कोंकण रेलवे : इस रेलवे का विस्तार पश्चिमी घाट पर्वत श्रेणी में महाराष्ट्र के रोहा से कर्नाटक के मंगलुरू तक देखने को मिलता है। इसकी लंबाई 760 km है (कोंकण रेलवे कारपोरेशन के अनुसार लंबाई 741 km) है। इस रेलमार्ग का सबसे ज्यादा विस्तार महाराष्ट्र में है। इसका मुख्यालय नवी मुंबई में है वहीं कोंकण रेलवे कारपोरेशन का मुख्यालय गोवा में है। कोंकण रेल परियोजना की शुरुआत 1990 में हुआ। यह परियोजना पूर्ण 1998 ई० में हुआ। कोंकण रेलवे में एडभांस Controlling Device तकनीक का प्रयोग किया गया है जो इस मार्ग पर ट्रेनों को आपसी टक्कर होने से बचाता है। ACD तकनीक GPS (Global position system) प्रणाली पर काम करता है। कोंकण रेलवे 91 सुरंग, 146 नदियाँ और 2000 पुलों से होकर गुजरती है। इस रेलमार्ग में सबसे ऊंचा पुल महाराष्ट्र में पनवल नदी पर है तो वही सबसे लंबा पुल कर्नाटक में शरावती नदी पर है। कोंकण रेलवे का विस्तार महाराष्ट्र, गोवा और कर्नाटक राज्य में है लेकिन उससे लाभांवित होने वाला राज्य महाराष्ट्र, गोवा, केरल और कर्नाटक के इस रलमार्ग का सबसे लंबा सुरंग महाराष्ट्र राज्य में है।
  • नेटवर्क की दृष्टिकोण से भारतीय रेलवे का पूरे विश्व में चौथा स्थान है। पहला अमेरिका, दूसरा चीन, तीसरा रूस का है। तो वही एशिया में भारत का स्थान दूसरा है। 
  • विश्व में सबसे बड़ा नियोक्ता भारतीय रलवे है।
  • भारत में राष्ट्रीय रेल दिवस 16 अप्रैल को मनाया जाता है तो वही 10-16 अप्रैल के सप्ताह को राष्ट्रीय रेल सप्ताह के रूप में मनाया जाता है।
  • विश्व में सर्वप्रथम रेलगाड़ी का परिचालन 1825 ई. में ब्रिटेन में स्टॉकटन से डार्लिंगटन के बीच हुआ ।
  • भारत में रेलवे परिवहन का विकास अंग्रेजों के द्वारा किया गया। सर्वप्रथम भारतीय गवर्न जनरल लार्ड हार्डिंग प्रथम ने परिवहन को लेकर भारत में रेलमार्ग परिवहन के विकास कही। लॉर्ड हार्डिंग प्रथम के बाद भारतीय गवर्नर जनरल बनकर लॉर्ड डलहौजी आए जिन्होंने भारत में रेलमार्ग का विकास किया। जिस कारण लॉर्ड डलहौजी को भारत में रेलमार्ग के पिता या जनक कहा जाता है।
  • लॉर्ड कर्जन के ही शासनकाल में भारतीय रेलवे बोर्ड का गठन 1905 ई० में हुआ है।
  • एकवर्थ कमिटी की सिफारिश पर आम बजट को रेल बजट से अलग 1924 ई० में किया गया पुनः विवेक देवराय कमिटी के. सिफारिश पर 2017 ई॰ से रेल बजट और आम बजट दोनों एक साथ पेश किये जाने लगे।
  • प्रथम बार भारत के लिए बजट 1860 ई. में जेम्स विलसन ने पेश किया था।
  • मोनो रेल : जब किसी क्षेत्र में भूमि की अनउपलब्धता तथा अधिक जनघनत्व के कारण एक ही पटरी पर रेलगाड़ी को दौड़ाया जाता है तो इसे ही मोनो रेल कहा जाता है। मोनो रेल का परिचालन करने वाला भारत विश्व का सातवाँ देश बना हैं।
  • भारत से पहले मोनो रेल का परिचालन अमेरिका, चीन, जापान, जर्मनी, मलेशिया, ऑस्ट्रेलिया जैसे देशों में हो चुका है। भारत में सर्वप्रथम मोनो रेल का परिचालन 1 फरवरी, 2014 को महाराष्ट्र के मुम्बई में वडाला से चेम्बूर के बीच हुआ। लंबाई = 19.8 km, उद्घाटनकर्ता पृथ्वी राज चौहान (CM of महाराष्ट्र ) ।
  • समझौता एक्सप्रेस भारत और पाकिस्तान के बीच 1976 ई० से चलना प्रारंभ हुआ।
  • थार एक्सप्रेस भारत और पाकिस्तान के बीच 2006 ई० से चलना प्रारंभ हुआ।
  • मैत्री एक्सप्रेस भारत और बांग्लादेश के बीच 2008 ई० से चलना प्रारंभ हुआ।
  • बंधन एक्सप्रेस भारत और बांग्लादेश के बीच 2017 ई० से चलना प्रारंभ हुआ।
  • विवेक एक्सप्रेस : यह भारत के भीतर सबसे लंबी दूरी तय करने वाली ट्रेन हैं जो असम के डिब्रूगढ़ से तमिलनाडु के कन्याकुमारी तक जाती है। यह ट्रेन 4286 km की दूरी तय करती है। 
  • भारत की सबसे तेज गति से चलने वाली ट्रेन ट्रेन-18 हैं जिसे वंदे मातरम एक्सप्रेस के नाम से भी जाना जाता है। इसकी अधिकतम गति 200 km/h हो सकती है।
  • बुलेट ट्रेन : जापान के सहयोग से भारत में बुलेट ट्रेन चलाया जाना प्रस्तावित है। भारत की पहली बुलेट ट्रेन गुजरात के अहमदाबाद से महाराष्ट्र के मुंबई के बीच चलाया जाना प्रस्तावित है। वही भारत की दूसरी बुलेट ट्रेन दिल्ली से बनारस के बीच चलाया जाना प्रस्तावित है।
  • पूर्वोत्तर भारत में स्थित सिक्किम एक ऐसा राज्य है जहाँ रेलगाड़ी का परिचालन नहीं होता है।
  • बिहार के छपरा में रेलगाड़ी के पहिये निर्माण का कारखाना खोला गया है।
  • बिहार के मधेपुरा और मदौरा (छपरा) में रेल इंजन कारखाना खोला गया है।
  • ट्रांस साइबेरियन रेलमार्ग : यह विश्व की सबसे लंबी रेलमार्ग है इसका विस्तार लेनिनग्राड से ब्लाडीवास्टक तक है, जो रूस में है। इस रेलमार्ग की लंबाई 9438 km है।
  • भारत में मेट्रो रेल : भारत में मेट्रो रेल परियोजना प्रधानमंत्री इंदिरा गाँधी के द्वारा 1972 ई० में रखी गई। यह परियोजना पूर्ण 1984 ई० में हुई। भारत में सर्वप्रथम मेट्रो रेल का परिचालन 24 अक्टूबर, 1984 को बंगाल प्रांत में दमदम से टॉलीगंज के बीच हुआ।
  • दिल्ली : दिल्ली में मेट्रो रेल का परिचालन जापान और कोरिया की मदद से 2002 ई० में तीस हजारी से शाहदरा के बीच हुआ।
  • बंगलुरू : बंगलुरू में मेट्रो रेल की शुरूआत 2011 ई. में जापान की मदद से हुआ। बंगलुरू में मेट्रो ट्रेन नम्मा मेट्रो के नाम से चलाया गया।
  • मुंबई : मुंबई में मेट्रो ट्रेन का परिचालन 2014 ई० में हुआ।
  • जयपुर, लखनऊ, चेन्नई जैसे महत्वपूर्ण शहरों में भी मेट्रो रेल का परिचालन हो चुका है लेकिन पटना, भोपाल, बिलासपुर जैसे शहरों में मेट्रो रेल चलाया जाना प्रस्तावित है।
  • विश्व में सर्वप्रथम मेट्रो रेल का परिचालन 10 मई, 1963 को लंदन में हुआ।
  • विश्व की सबसे लंबी मेट्रो शंघाई मेट्रो ट्रेन हैं जो 434 km की दूरी तय करता  है।
  • रेलवे विश्वविद्यालय : 2018 ई० में रेलवे विश्वविद्यालय का निर्माण भारत में हुआ । भारतीय रेलवे विश्वविद्यालय गुजरात के बड़ोदरा में है।
  • रेल गति : भारत का रेलवे गीत 'प्यार हैं हमें भारतीय रेलवे से" हैं। इस गीत की रचना सत्यप्रकाश ने किया है। गाने में 3 मिनट का समय लगता है।
  • सड़क परिवहन : भारतीय सड़क को तीन वर्गों में बाँटा गया है- 1. राष्ट्रीय राजमार्ग (NH), 2. राज्य राजमार्ग, 3. अन्य सड़कें (ग्रामीण सड़कें / शहरों की गलियों की सड़कें ) ।
  • स्वर्णिम चतुर्भुज योजना : देश के चार प्रमुख महानगर दिल्ली, मुंबई, चेन्नई, कोलकाता को आपस में 4 लेन या 6 लेन के माध्यम से जोड़ा गया इसे ही स्वर्णिम चतुर्भुज योजना कहते हैं। यह सपना अटल बिहारी वाजपेयी ( 1998-2004) ने देखा था जो 2012 ई० में जाकर पूर्ण हुआ ।
  • उत्तरी- दक्षिणी गलियारा : भारत के उत्तर में स्थित श्रीनगर को भारत के दक्षिण में स्थित तमिलनाडु के कन्याकुमारी से आपस में उतरी दक्षिणी गलियारा जोड़ता है।
  • पूर्वी-पश्चिमी गलियारा : भारत के पूरब में स्थित असम के सिलचर को भारत के पश्चिम में स्थित गुजरात के पोरबंदर को आपस में पूर्वी-पश्चिमी गलियारा जोड़ता है।
  • Note : पूर्वी-पश्चिमी गलियारा और उत्तरी दक्षिणी गलियारा आपस में एक-दूसरे को उत्तर प्रदेश के झाँसी में काटता है।
  • राष्ट्रीय राजमार्ग (NH) : देश के कुल सड़क नेटवर्क के मात्र 2% हिस्सा पर राष्ट्रीय राजमार्ग का विस्तार देखने को मिलता है। देश में वर्तमान समय में सड़कों की कुल लंबाई लगभग 62 लाख किमी० है । सड़क नेटवर्क के दृष्टिकोण से भारत का पूरे विश्व में दूसरा स्थान है वही सड़क नेटवर्क में पहला स्थान USA का है। तीसरा स्थान चीन ।
  • Note : राष्ट्रीय राजमार्ग का प्रबंधन एवं रख-रखाव तथा देखरेख भारत सरकार के अधीन स्थापित संस्था राष्ट्रीय राजमार्ग प्राधिकरण के द्वारा किया जाता है। राष्ट्रीय राजमार्ग प्राधिकरण की स्थापना 1995 ई० में हुई है।
  • भारत के सीमावर्ती इलाकों में तीव्र गति से सड़क निर्माण हेतु भारत के प्रथम प्रधानमंत्री पंडित जवाहर लाल नेहरू के द्वारा 1960 ई० में सीमा सड़क संगठन की स्थापना की गई।
  • NH-44 : यह देश का सबसे लंबा NH है। इसकी लंबाई 3745 km है। यह NH उत्तर में स्थित श्रीनगर को दक्षिण में स्थित कन्याकुमारी से आपस में जोड़ता है। यह NH निम्न राज्य और केन्द्रशासित प्रदेश से तरीके गुजरती हैं- जम्मू-कश्मीर, पंजाब, हरियाणा, दिल्ली, उत्तर प्रदेश, राजस्थान, मध्य प्रदेश, महाराष्ट्र, तेलंगाना, आंध्र प्रदेश, कर्नाटक, तमिलनाडु ।
  • देश का सबसे छोटा NH, NH-118 (झारखंड) और NH-548 (महाराष्ट्र) है जिसकी लंबाई 5 किमी० है ।
  • चेनानी-नाशरी सुरंग : यह देश का सबसे बड़ा सड़क सुरंग है। इसकी लंबाई 9.2 km है। यह जम्मू-कश्मीर में स्थित है।
  • भूपेन हजारिका पुल : यह देश का सबसे बड़ा नदियों पर निर्मित सड़क हैं। यह पुल ब्रह्मपुत्र की सहायक नदी लोहित नदी पर निर्मित है। यह पुल ढोला का सदिया से आपस में जोड़ता है। पुल की लंबाई 9.15 km है।
  • देश के भीतर सबसे अधिक NH की लंबाई महाराष्ट्र में है। महाराष्ट्र के बाद उत्तर प्रदेश, राजस्थान, मध्य प्रदेश, कर्नाटक का क्रमश: दूसरा, तीसरा, चौथा एवं पाँचवाँ स्थान हैं।
  • राज्य राजमार्ग (SH) : राज्य राजमार्ग का निर्माण राज्य सरकार के द्वारा किया जाता है तथा इसकी देखरेख एव रख-रखाव राज्य सरकार के द्वारा किया जाता है। भारत में राज्य राजमार्ग या प्रांतीय राजमार्ग की सर्वाधिक लम्बाई महाराष्ट्र राज्य में देखने को मिलती है।
  • प्रधानमंत्री ग्राम सड़क योजना : वैसा मैदानी क्षेत्र जहाँ की आबादी कम-से-कम 500 हो तथा वह क्षेत्र बारहमासी सड़क मार्ग से नहीं जुड़ा हो तो उन क्षेत्रों को बारहमासी सड़कों से जोड़ने का काम प्रधानमंत्री सड़क योजना के तहत किया जाता है। इस योजना की शुरुआत 25 दिसंबर, 2000 ई० को प्रधानमंत्री अटल बिहारी वाजपेयी के द्वारा किया गया। यह पूर्णत: केन्द्र सरकार की योजना है।
  • सड़क घनत्व : किसी भी क्षेत्र में प्रति 100 वर्ग किमी० में औसत सड़कों की लंबाई सड़क घनत्व कहलाता है। राज्य और केन्द्रशासित प्रदेश दोनों का समस्त रूप से मिला कर देखा जाए तो सर्वाधिक सड़क घनत्व दिल्ली का है वही सिर्फ राज्यों के दृष्टिकोण से देखा जाए तो सर्वाधिक सड़क घनत्व वाला राज्य केरल है।
  • प्रोजेक्ट दंतक : इसके तहत भूटान में सड़क अवसंरचना का निर्माण किया गया है।
  • प्रोजेक्ट बीकन : इसके तहत जम्मू-कश्मीर के पर्वतीय इलाकों में सड़क का निर्माण किया गया है। प्रोजेक्ट बीकन को जम्मू- कश्मीर की जीवन रेखा कहा जाता है।
  • प्रोजेक्ट हीरक : महाराष्ट्र के नक्सल प्रभावी क्षेत्रों में सड़कों का निर्माण जिस योजना के तहत हुआ है उसे प्रोजेक्ट हीरक कहा जाता है।
  • ग्रीन मफलर : पर्यावरणीय संतुलन बनाए रखने तथा ध्वनि प्रदूषण को कम करने के लिए सड़कों के किनारे-किनारे नीम और अशोक जैसे वृक्ष लगाए जाते हैं इस हेतु ही ग्रीन मफलर शब्द का प्रयोग किया गया है।
  • पर्यावरणीय संतुलन बनाए रखने के लिए कुल क्षेत्रफल के 33% भू-भाग पर वन होना चाहिए ।
  • स्वतंत्र भारत की प्रथम राष्ट्रीय वन नीति 1952 ई० में आई है जिसमें संशोधन 1988 में हुआ है।
  • भारत में राष्ट्रीय वन महोत्सव के० एम० मुंशी के प्रयास से मनाया जाना प्रारंभ हुआ।
  • भारत में राष्ट्रीय वन महोत्सव 1 जुलाई को मनाया जाता है तथा वन सप्ताह के रूप में जुलाई के प्रथम सप्ताह को मनाया जाता है।
  • प्रमुख राष्ट्रीय राजमार्ग :
  • NH-1 : यह श्रीनगर को लेह से आपस में जोड़ता है।
  • NH-2 : यह असम के डिब्रूगढ़ को मिजोरम के तुइपांग से आपस में जोड़ता है।
  • NH-3 : यह लेह को भारत-पाक बॉर्डर पर स्थित अटारी से आपस में जोड़ता है |
  • NH-4 : यह अंडमान निकोबार द्वीप समूह में स्थित है। यह NH पोर्ट ब्लेयर को चिड़िया टापू (लिटिल अंडमान) से आपस में जोड़ता है।
  • NH-5 : यह पंजाब के फिरोजपुर को शिपकीला (हिमाचल प्रदेश) से आपस में जोड़ता है।
  • NH-6 : मेघालय के जोरबाट को मिजोरम के सीलिंग से आपस में जोड़ता है।
  • NH-7 : पंजाब के फजिल्ला को उत्तराखंड के माना से आपस में जोड़ता है।
  • NH-8 : यह असम के करीमगंज को त्रिपुरा के शबरूम से आपस में जोड़ता है।
  • NH-9 : यह पंजाब के मलौट को उत्तराखण्ड के पिथौरागढ से आपस में जोड़ता है।
  • NH-10 : गंगटोक को सिलिगुड़ी से NH-10 जोड़ता है।
  • NH-16 : यह कोलकाता को चेन्नई से जोड़ता है।
  • NH-19 : यह दिल्ली को कोलकाता से जोड़ता हैं।
  • NH-48 : यह दिल्ली को चेन्नई से आपस में जोड़ता है।
  • Note : उत्तरी-दक्षिणी गलियारा को NH-44 और पूर्वी-पश्चिमी गलियारा को NH-27 कहा जाता है।
  • NH-30 : यह उत्तराखंड के सितारगंज को आंध्र प्रदेश के कोडापल्ली से आपस में जोड़ता है।
  • NH-52 : यह पंजाब के संघरूढ़ को कर्नाटक के अंकोला से आपस में जोड़ता है |
  • NH-53 : यह गुजरात के हजीरा को उड़ीसा के पाराद्वीप से आपस में जोड़ता है |
  • NH-66 : यह महाराष्ट्र के नवी मुंबई में स्थित पनवेल को तमिलनाडु के कन्याकुमारी से आपस में जोड़ता है।
  • NH-129 : यह असम के नुमालीगढ़ को नागालैंड के दीमापुर से आपस में जोड़ता है ।
  • जल परिवहन : भारत का सबसे प्राचीनतम या आयात का साधन जल परिवहन है।
  • भारत में बड़े बंदरगाहों की संख्या 13 है जबकि छोटे और मंझले बड़े बंदरगाह की संख्या 200 से भी अधिक है। बड़ा बंदरगाह संघ सूची में आता है जिस कारण इस पर कानून भारत की संसद बनाता है वही छोटे बंदरगाह समवर्ती सूची में आता है।
  • भारत के पश्चिमी तट पर छः बड़े बंदरगाह है जो निम्न हैं-
    1. कांडला बंदरगाह : यह गुजरात राज्य के कच्छ की खाड़ी में स्थित एक प्राकृतिक और ज्वारीय बंदरगाह हैं। इस बंदरगाह का नाम बदलकर पंडित दीनदयाल उपाध्याय बंदरगाह कर दिया गया है।
    2. मुंबई बंदरगाह : यह भारत के महाराष्ट्र राज्य में स्थित है। यह देश का सबसे बड़ा प्राकृतिक बंदरगाह है। इस बंदरगाह को भारत का प्रवेश द्वार कहा जाता है।
    3. न्हावाशोवा बंदरगाह : इसे जवाहरलाल नेहरू बंदरगाह के नाम से भी जाना जाता है। मुंबई बंदरगाह के भार को कम करने के लिए इस बंदरगाह का निर्माण किया गया है। यह एक कृत्रिम बंदरगाह है। यहाँ हर प्रकार की सुविधा उपलब्ध है।
    4. मार्मुगाओ बंदरगाह : यह गांवा में मांडवी और जुआरी नदी के किनारे स्थित एक प्राकृतिक बंदरगाह है।
    5. मंगलुरू बंदरगाह : यह कर्नाटक में स्थित एक प्राकृतिक बंदरगाह नगर है ।
    6. कोच्चि बंदरगाह : यह केरल में स्थित एक प्राकृतिक बंदरगाह है।
  • भारत के पूर्वी तट पर छ: बड़े बंदरगाह हैं, जो निम्न हैं-
    1. कोलकाता बंदरगाह : यह पश्चिम बंगाल में स्थित एक नदीय और प्राकृतिक बंदरगाह है। इस बंदरगाह को भारत का पूर्वी प्रदेश द्वार कहा जाता है। इस बंदरगाह के भार को कम करने के लिए हल्दिया बंदरगाह का निर्माण किया गया है।
    2. पाराद्वीप बंदरगाह : यह ओडिशा में स्थित एक कृत्रिम बंदरगाह है।
    3. विशाखापत्तनम बंदरगाह : यह आंध्र प्रदेश में स्थित एक प्राकृतिक बंदरगाह है। यह देश का सबसे गहरा बंदरगाह है।
    4. एन्नौर बंदरगाह : यह तमिलनाडु में स्थित एक प्राकृतिक बंदरगाह है। इसका नाम बदलकर कामराज बंदरगाह किया गया है।
    5. बंदरगाह है। यह एक कृत्रिम बंदरगाह है। वह बंदरगाह हर मौसम में चेन्नई बंदरगाह : यह देश का सबसे प्राचीनतम् बंदरगाह है। खुला रहता है।
    6.  तूतीकोरिन बंदरगाह : यह तमिलनाडु में स्थित एक कृत्रिम बंदरगाह है। इसका नाम बदलकर वी० ओ० चिदंबरमनार बंदरगाह किया गया है।
    7. पोर्ट ब्लेयर बंदरगाह : यह अंडमान निकोबार द्वीप समूह में स्थित एक प्राकृतिक बंदरगाह है।
  • Note : (i) गुजरात में स्थित दाहेज को रसायन बंदरगाह के नाम से जाना जाता है।
    (ii) देश में सबसे अधिक छोटे और मंझले बंदरगाह गुजरात राज्य में स्थित है।
    (iii) ओखा बंदरगाह गुजरात में कच्छ की खाड़ी में स्थित है।
  • सागरमाला परियोजना : इस परियोजना की घोषणा 15 अगस्त, 2003 को प्रधानमंत्री अटल बिहारी वाजपेयी के द्वारा की गई। इस परियोजना के तहत देश के सभी बड़े बंदरगाह को आपस में जोड़ने का काम किया जाना है तथा बंदरगाह के आस - पास के क्षेत्रों में विशेष आर्थिक क्षेत्र का विकास किया जाना है ताकि व्यापार में वृद्धि हो सके। इस परियोजना की स्वीकृति केन्द्रीय कैबिनेट ने 2015 में प्रदान किया। अब इस परियोजना पर काम चल रहा है।
  • सेतु समुद्रम परियोजना : बंगाल की खाड़ी और अरब सागर के बीच की दूरी को कम करने के लिए पाक जलसंधि को मन्नार की खाड़ी से जोड़ने की जो योजना है, उसे ही सेतुसमुद्रम परियोजना कहा गया है। सेतुसमुद्रम परियोजना का सपना सर्वप्रथम 1860 ई० में ब्रिटिश कमांडर ए० डी० टेलर ने देखा था ।
  • राष्ट्रीय जलमार्ग : 2016 ई० में जो राष्ट्रीय जल नीति की घोषणा हुई उसके तहत 105 नये जलमार्ग बनाया जाना प्रस्तावित हैं इसके पहले 6 जलमार्ग कार्यरत था। कुल मिलाकर वर्तमान में 111 राष्ट्रीय जलमार्ग है । कुछ महत्वपूर्ण राष्ट्रीय जलमार्ग निम्न हैं-
    1. राष्ट्रीय जलमार्ग संख्या - 1: इसका निर्माण 1986 ई० में गंगा नदी पर प्रयागराज से हल्दिया के बीच किया गया। इस जलमार्ग से उत्तर प्रदेश, बिहार, झारखंड और पश्चिम बंगाल लाभांवित होता है। इसकी लंबाई 1620 km है।
    2. राष्ट्रीय जलमार्ग संख्या-2 : इसका निर्माण 1988 ई० में ब्रह्मपुत्र नदी पर असम राज्य में सदिया से धुबरी के बीच स्थित है। इसकी लंबाई 891 km है। इस जलमार्ग से मुख्यतः लाभांवित होने वाला राज्य असम है।
    3. राष्ट्रीय जलमार्ग संख्या-3 : इसका निर्माण 1993 ई० में केरल में कोल्लम कोट्टापुरम के बीच स्थापित किया गया। इस जलमार्ग का संबंध मुख्यतः पश्चिमी तटीय नहर से है। इसकी लंबाई 375 km है।
    4. राष्ट्रीय जलमार्ग संख्या-4 : इसका निर्माण 2008 ई० में काकीनाडा से मरक्कानन के बीच हुआ। इस जलमार्ग का संबंध कृष्णा और गोदावरी दोनों नदियों से है। इसकी कुल लम्बाई 2916km है। इस जलमार्ग से लाभांवित होने वाला राज्य आंध्र प्रदेश, तेलंगाना और तमिलनाडु है ।
    5. राष्ट्रीय जलमार्ग संख्या-5 : इस जलमार्ग का विस्तार मुख्यत: ओडिशा राज्य में ब्राह्मणी नदी पर तलचर से घमरा के बीच है। इस जलमार्ग का निर्माण 2008 ई० में हुआ है। इसकी लंबाई 588 km है।
    6. राष्ट्रीय जलमार्ग संख्या - 6 : इस जलमार्ग का संबंध बराक नदी से है इसका विस्तार मुख्यतः लखीपुर (असम) से भागा ( बांग्लादेश) के बीच है। इसकी लंबाई लगभग 526 km है।
  • इस जलमार्ग से लाभांवित होने वाला राज्य अरूणाचल प्रदेश, असम, नागालैंड, मणिपुर, मिजोरम, त्रिपुरा है।
  • वर्तमान में भारत का सबसे बड़ा राष्ट्रीय जलमार्ग संख्या - 4 है जबकि सबसे छोटा राष्ट्रीय जलमार्ग राष्ट्रीय जलमार्ग संख्या 69 है जो तमिलनाडु राज्य में मणिमुधाई नदी के तट पर है। इसकी लंबाई 5 km है।
  • वायु परिवहन : भारत में वायु परिवहन की शुरूआत 1911 ई० में डाक सेवा के रूप में इलाहाबाद से नैनी के बीच हुई। भारत में वायु परिवहन का राष्ट्रीयकरण 1953 ई० में हुआ है। भारत में वायु परिवहन का प्रबंधन और निरीक्षण भारतीय विमान पत्तनम प्राधिकरण के द्वारा किया जाता है। भारतीय विमान पत्तनम प्राधिकरण की स्थापना अप्रैल, 1995 ई० में हुआ।
हवाई अड्डा स्थान
वीर सावरकर अंतर्राष्ट्रीय हवाई अड्डा पोर्ट ब्लेयर
सुभाषचन्द्र बोस अंतर्राष्ट्रीय हवाई अड्डा कोलकाता
गोपीनाथ बारदोलोइ अंतर्राष्ट्रीय हवाई अड्डा गुवाहाटी
इंदिरा गाँधी अंतर्राष्ट्रीय हवाई अड्डा नई दिल्ली
चौधरी चरण सिंह अंतर्राष्ट्रीय हवाई अड्डा लखनऊ
लाल बहादुर शास्त्री अंतर्राष्ट्रीय हवाई अड्डा वाराणसी
छत्रपति शिवाजी अंतर्राष्ट्रीय हवाई अड्डा मुंबई
देवी अहिल्याबाई होलकर अंतर्राष्ट्रीय हवाई अड्डा इंदौर
स्वामी विवेकानन्द अंतर्राष्ट्रीय हवाई अड्डा रायपुर
जयप्रकाश नारायण अंतर्राष्ट्रीय हवाई अड्डा पटना
मीनाम्बकम अंतर्राष्ट्रीय हवाई अड्डा चेन्नई
बागडोगरा अंतर्राष्ट्रीय हवाई अड्डा सिलीगुड़ी (बंगाल)
डॉ॰ भीमराव अंतर्राष्ट्रीय हवाई अड्डा नागपुर
केम्पेगोंडा अंतर्राष्ट्रीय हवाई अड्डा बंगलुरू
ताबोलिम अंतर्राष्ट्रीय हवाई अड्डा गोवा
राजीव गाँधी अंतर्राष्ट्रीय हवाई अड्डा हैदराबाद
सरदार वल्लभ भाई पटेल अंतर्राष्ट्रीय हवाई अड्डा अहमदाबाद
कालीकट अंतर्राष्ट्रीय हवाई अड्डा कोझीकोड (केरल)
नांदिबचेरी अंतर्राष्ट्रीय हवाई अड्डा केरल
श्रीगुरु रामदासपुर अंतर्राष्ट्रीय हवाई अड्डा अमृतसर

वस्तुनिष्ठ प्रश्न एवं उत्तर

1. भारत में सबसे ऊँची सड़क कहाँ मिलती है ?
(a) दार्जिलिंग में 
(b) शिमला में
(c) पश्चिमी घाट में
(d) लेह में
2. ग्रैंड-ट्रंक रोड किस नाम से जाना जाता है ?
(a) NH-1 ए NH-2
(b) NH-8
(c) NH-3
(d) NH-10
3. भारत में रेलमार्गों की कुल लंबाई कितनी है (31 मार्च 2013 तक ) ? 
(a) 63,332 किलोमीटर 
(b) 65,436 किलोमीटर
(c) 64,015 किलोमीटर
(d) 63,327 किलोमीटर
4. पर्वतीय भाग में किस प्रकार के रेलमार्ग बनाए गए हैं ?
(a) बड़ी लाइन
(b) सँकरी लाइन
(c) छोटी लाइन
(d) इनमें से कोई नहीं
5. पूर्वोत्तर रेलवे का मुख्यालय कहाँ है ?
(a) कटिहार में
(b) मुजफ्फरपुर में
(c) झाँसी में
(d) गोरखपुर में
6. विवेक एक्सप्रेस कहाँ से कहाँ तक जाती है ?
(a) जम्मू से हावड़ा तक
(b) नई दिल्ली से मुंबई तक
(c) डिब्रूगढ़ से कन्याकुमारी तक
(d) गुवाहाटी से नई दिल्ली तक
7. बरौनी में किस मार्ग से कच्चा तेल पहुँचाया जाता है ?
(a) रेलमार्ग से
(b) पक्की सड़क
(c) पाइपलाइन से
(d) वायुयान से
8. मुंबई अंतरराष्ट्रीय हवाई पत्तन का नया नाम क्या है ? 
(a) सांताक्रूज हवाई पत्तन
(b) इंदिरा गांधी हवाई पत्तन
(c) छत्रपति शिवाजी हवाई पत्तन
(d) सुभाषचंद्र बोस हवाई पत्तन
9. इनमें सबसे दक्षिणी पंत्तन कौन है ?
(a) कांडला
(b) कोच्चि
(c) तूतीकोरिन
(d) पारादीप
10. सिकंदराबाद किस रेलवे क्षेत्र का मुख्यालय है ?
(a) उत्तर - पूर्व सीमांत रेलवे का
(b) दक्षिण - पूर्व रेलवे का
(c) दक्षिण मध्य रेवले का
(d) पश्चिम-मध्य रेलवे का
11. राँची किस रेलवे क्षेत्र में स्थित है ?
(a) दक्षिण रेलवे क्षेत्र
(b) दक्षिण - पूर्व रेलवे क्षेत्र
(c) दक्षिण-पश्चिम रेलवे क्षेत्र
(d) दक्षिण-मध्य रेलवे क्षेत्र
12. भारत में कितने डाकघर थे (31 मार्च 2021 तक ) ?
(a) 50 हजार
(b) 125 हजार
(c) 154 हजार
(d) 255 हजार से ऊपर
13. हल्दिया कहाँ स्थित है ?
(a) चेन्नई के निकट
(b) कोलकाता के निकट
(c) मुंबई के निकट
(d) गोवा के निकट
14. इनमें भारत किस वस्तु का मुख्य रूप से आयात करता है?
(a) पेट्रोलियम
(b) गेहूँ
(c) कोयला
(d) सूती कपड़ा
15. भारत में कितने आकाशवाणी केंद्र हैं ?
(a) 52
(b) 152
(c) 403
(d) लगभग 450
16. भारत में टेलीफोन की सेवाएँ कब आरंभ हुई ?
(a) 1815 में
(b) 1881 में
(c) 1912 में
(d) 1947 में
17. इनमें कौन व्यापारिक केंद्र गंगा नदी के तट पर स्थित नहीं है ?
(a) कानपुर
(b) भागलपुर 
(c) जबलपुर 
(d) वाराणसी
18. दिल्ली से कोलकाता तक की राष्ट्रीय सड़क किस नाम से प्रसिद्ध है ?
(a) NH-1
(b) NH-2
(c) NH-4
(d) NH-5
19. भारत के किस राज्य में सड़कों की लंबाई सबसे अधिक है ?
(a) बिहार में 
(b) तमिलनाडु में
(c) उत्तर प्रदेश में
(d) केरल में
20. भारतीय अंतरदेशीय जलमार्ग प्राधिकरण का गठन किस वर्ष हुआ था ?
(a) 1986 में
(b) 1988 में
(c) 1989 में
(d) 1985 में
21. निम्नलिखित में कौन नई आर्थिक नीति का अंग नहीं है ?
(a) राष्ट्रीयकरण
(b) वैश्वीकरण
(c) उदारीकरण 
(d) निजीकरण
22. निम्नलिखित कथनों पर विचार कीजिए-
1. 'नागपुर योजना' का संबंध सड़क विकास योजना से है।
2. भारतीय राष्ट्रीय राजमार्ग प्राधिकरण (NHAI) एक स्वायत्तशासी निकाय है।
3. राष्ट्रीय महामार्गों के विकास, रख-रखाव तथा प्रचालन की जिम्मेदारी भारतीय राष्ट्रीय राजमार्ग प्राधिकरण को सौंपी गई है।
उपर्युक्त में से कौन-सा/से कथन सही है/हैं
(a) केवल 1
(b) केवल 2 और 3
(c) केवल 2
(d) 1, 2 और 3
23. राष्ट्रीय राजमार्ग विकास परियोजनाओं के संदर्भ में कौन-सा/से कथन सही है / हैं ?
1. स्वर्णिम चतुर्भुज परियोजना की शुरूआत भारतीय राष्ट्रीय राजमार्ग प्राधिकरण के एक भाग के रूप में की गई थी।
2. उत्तर - दक्षिण तथा पूर्व-पश्चिम गलियारा परियोजना भारतीय राष्ट्रीय राजमार्ग प्राधिकरण के अधिकार क्षेत्र में नहीं है। गुजरात को
3. पूर्वी - पश्चिमी गलियारे का उद्देश्य असम से जोड़ना है।
कूट :
(a) केवल 1 
(b) केवल 1 और
(c) केवल 2 और 3
(d) 1, 2 और 3
24. निम्नलिखित में से सीमा सड़क संगठन ( BRO) की स्थापना किस वर्ष की गई ? 
(a) 1960 
(b) 1963
(c) 1972
(d) 1957
25. निम्नलिखित में से कौन-सा/से कथन सही है/हैं ?
1. सीमा सड़क संगठन का उद्देश्य उत्तरी एवं उत्तर-पूर्वी सीमा से सटी सामरिक दृष्टि से महत्वपूर्ण सड़कों के विकास से आर्थिक विकास को गति वरक्षा तैयारियों को मजबूती प्रदान करना है।
2. राष्ट्रीय महामार्ग NHI -1 दिल्ली से कोलकाता के बीच विस्तृत है।
3. सीमा सड़क संगठन ( बी.आर.ओ.) सड़कें बनाने व अनुरक्षण करने के साथ-साथ अति ऊँचाई वाले क्षेत्रों में बर्फ हटाने की जिम्मेदारी भी संभालना है।
कूट :
(a) केवल 1
(b) केवल 2 और 3
(c) केवल 1 और 3
(d) 1,2 और 3
26. भारतीय रेल के संदर्भ में निम्नलिखित कथनों पर विचार कीजिये :
1. भारत में वर्ष 1905 से रेल परिवहन की शुरूआत की गई।
2. पहली रेल लाइन मुंबई से थाणे के बीच निर्मित की गई।
3. भारतीय रेल को 14 जोन में विभाजित किया गया है।
उपर्युक्त में से कौन-सा/से कथन सही नहीं है/हैं ?
(a) केवल 1
(b) केवल 2
(c) केवल 1 और 3
(d) 1, 2 और 3
27. कोंकण रेलवे निम्नलिखित में से किन दो राज्यों को जोड़ती है ?
(a) महाराष्ट्र - केरल
(b) गुजरात - कर्नाटक
(c) राजस्थान - आंध्र प्रदेश
(d) महाराष्ट्र - कर्नाटक
28. भारतीय रेल (रेलवे पटरी) के संदर्भ में निम्नलिखित कथनों पर विचार कीजिये :
1. ब्रॉड गे (Broad Gauge) या बड़ी लाइन में रेल पटरियों के बीच की दूरी 1.616 मीटर होती है।
2. छोटी लाइन (Narrow Gauge) रेल प्राय: पर्वतीय क्षेत्रों तक सीमित है।
3. छोटी लाइन में दो रेल पटरियों के बीच की दूरी 762 मीटर होती है।
उपर्युक्त में से कौन - सा/से कथन सही है/हैं ?
(a) केवल 1 और 2 
(b) केवल 1 और 3
(c) केवल 3
(d) उपर्युक्त सभी
29. भारत का राष्ट्रीय जलमार्ग-2 किस नदी पर तथा किन दो स्थान के बीच पड़ता है ? 
(a) गंगा नदी - (प्रयागराज - हल्दिया)
(b) ब्रह्मपुत्र नदी - (सादिया - धुबरी )
(c) पश्चिमी तट नहर - ( कोट्टापुरम - कोल्लम)
(d) इनमें से कोई नहीं 
30. निम्नलिखित कथनों पर विचार कीजिये :
1. भारत में वायु परिवहन की सर्वप्रथम शुरूआत प्रयागराज से नैनी के बीच की गई।
2. वायु परिवहन का राष्ट्रीकरण 1953 में हुआ था ।
3. राष्ट्रीय वायु परिवहन क्षेत्र में सुरक्षित, सक्षम वायु यातायात एवं संचार सेवाएँ प्रदान करने के लिए एयरपोर्ट अथॉरिटी ऑफ इंडिया (AAI) उत्तरदायी है।
4. 'पवन हंस' एक हेलीकॉप्टर सेवा है।
उपर्युक्त में से कौन -सा/से कथन सही है/हैं ?
(a) केवल 1
(b) केवल 2, 3 और 4
(c) केवल 2 और 4 
(d) 1, 2, 3 और 4
31. तेल एवं गैस पाइपलाइन के संबंध में निम्नलिखित कथनों पर विचार कीजिए-
1. इसमें गैसों व तरल पदार्थों के साथ ठोस पदार्थों को भी घोल या गारा में बदलकर परिवहित किया जा सकता है। 
2. ऑयल इंडिया लिमिटेड (OIL) कच्चे तेल एवं प्राकृति गैस के अन्वेषण, उत्पादन और परिवहन में संलग्न है।
3. एशिया में पहली देशपारीय पाइपलाइन का निर्माण भारत के असम राज्य में नाहरकटिया तेल क्षेत्र से बरौनी के तेलशोधन कारखाने के बीच हुआ।
उपर्युक्त में से कौन-सा/से कथन सही है/हैं ?
(a) केवल 1
(b) केवल 2 और 3
(c) केवल 1 और 3
(d) उपर्युक्त सभी
32. निम्नलिखित कथनों पर विचार कीजिये:
1. सभी वैयक्तिक संचार तंत्रों में इंटरनेंट सर्वाधिक प्रभावी एवं अधुनातन है।
2. भारत में रेडियो का प्रसारण रेडियो क्लब ऑफ बॉम्बे द्वारा प्रारंभ किया गया था।
3. ऑल इंडिया रेडियो सिर्फ सूचना व शिक्षा कार्यक्रमों को ही प्रसारित करता है। 
4. उपग्रह संचार आर्थिक एवं सामरिक दृष्टि से महत्व नहीं रखते हैं।
उपर्युक्त में से कौन - सा /से कथन सही है/हैं ?
(a) केवल 1 
(b) केवल 2, 3 और 4
(c) केवल 3 और 4
(d) केवल 2 और 4
33. निम्नलिखित में से किस संचार तंत्र का उपयोग ई-कॉमर्स तथा मौद्रिक लेन-देन के लिये अधिक किया जा रहा है?
(a) टेलीफोन
(b) इंटरनेट 
(c) ईमेल
(d) टेलीविजन
34. निम्नलिखित कथनों पर विचार कीजिये:
1. इंडियन नेशनल सेटेलाइट सिस्टम ( INSAT ) तथा इंडियन रिमोट सेंसिंग सेटेलाइट सिस्टम (IRS) उपग्रह प्रणाली से ही संबद्ध है।
2. इनसैट बहुउद्देश्यीय उपग्रह प्रणाली है।
3. नेशनल रिमोट सेंसिंग एजेंसी ( NRSA ) हैदराबाद में स्थित है।
4. IRS उपग्रह अनेक वर्णक्रमीय (स्पेक्ट्रम) बैंड (समूह) को एकत्रित करते हैं तथा विविध उपयोगों हेतु भू-स्टेशनों पर संप्रेषित करते ।
उपर्युक्त में से कौन-सा/से कथन सही है/हैं ?
(a) केवल 1 और 2
(b) केवल 1, 2 और 4
(c) केवल 3
(d) 1, 2, 3 और 4
35. भारत की 'स्वर्णिम चतुर्भुज' परियोजना जोड़ती हैं-
(a) दिल्ली - मुंबई - चेनई - कोलकाता को
(b) दिल्ली - झाँसी - बंगलूरू - कन्याकुमारी को
(c) श्रीनगर - दिल्ली - कानपुर - कोलकाता को
(d) पोरबंदर - बंगलूरू - कोलकाता - कानपुर को
36. निम्न में से कौन-सा तट 'कोच्चि बंदरगाह' से संबंधित है? 
(a) मालाबार तट 
(b) कोंकण तट
(c) कोरोमंडल तट
(d) उत्तरी सरकार तट 
37. भारत में देश के कुल यातायात में सड़क यातायात का भाग है-
(a) 100%
(b) 80%
(c) 60%
(d) 40%
38. भारत के निम्न राज्यों में से किसके प्रांतीय राजमार्गों की सकल लंबाई सबसे अधिक है ?
(a) गुजरात
(b) राजस्थान
(c) उत्तर प्रदेश
(d) महाराष्ट्र
39. स्वर्णिम चतुर्भुज परियोजना का संबंध किसके विकास से है ?
(a) राजमार्ग
(b) बंदरगाह
(c) विद्युत श्रृंखला (पॉवर ग्रिड)
(d) पर्यटन चक्र ( नेटर्व)
40. 'प्रधानमंत्री भारत जोड़ो परियोजना' संबंधित है-
(a) संचार से
(b) सामाजिक एकीकरण से
(c) नदियों के जुड़ाव से
(d) राजमार्गों के विकास से
हमसे जुड़ें, हमें फॉलो करे ..
  • Telegram ग्रुप ज्वाइन करे – Click Here
  • Facebook पर फॉलो करे – Click Here
  • Facebook ग्रुप ज्वाइन करे – Click Here
  • Google News ज्वाइन करे – Click Here
]]>
Thu, 11 Apr 2024 08:10:20 +0530 Jaankari Rakho
General Competition | Geography | भारत की प्राकृतिक वनस्पति https://m.jaankarirakho.com/960 https://m.jaankarirakho.com/960 General Competition | Geography | भारत की प्राकृतिक वनस्पति
  • बिना मानवीय हस्तक्षेप के जो पेड़, पौधे, वृक्ष, घास, इत्यादि उगता है, उसे प्राकृतिक वनस्पति कहते हैं।
  • प्राकृतिक संतुलन बनाए रखने के लिए कुल क्षेत्रफल के 33% भू-भाग पर वन होना चाहिए ।
  • भारत की प्रथम राष्ट्रीय वन नीति 1952 में आयी हैं, जिसमें संशोधन 1988 ई० में हुआ है। इस वन नीति का लक्ष्य भारत के कुल क्षेत्रफल के 33% भूभाग पर वन लगाया जाना है।
  • भारतीय वन रिपोर्ट फॉरेस्ट सर्वे ऑफ इंडिया नामक संस्था के द्वारा 1987 ई० से प्रत्येक दो वर्ष की अवधि पर जारी किया जाता रहा है।
  • 2021 के आँकड़ों के अनुसार भारत में सर्वाधिक वनों का क्षेत्रफल वाला राज्य मध्य प्रदेश (77,493 वर्ग किमी) तथा सबसे कम वनों का क्षेत्रफल वाला राज्य हरियाणा (1603 वर्ग किमी) है।
  • भारत में कुल 17 राज्य और केन्द्रशासित प्रदेश ऐसे हैं जहाँ 33% से अधिक वन पाया जाता है। जैसे-मध्य प्रदेश, अरूणाचल प्रदेश, छत्तीसगढ़, मिजोरम, लक्षद्वीप इत्यादि ।
  • प्रतिशतता के आधार पर सर्वाधिक वन प्रतिशत वाला राज्य मिजोरम (84% से अधिक) है।
  • प्रतिशतता के आधार पर सबसे कम वन प्रतिशत वाला राज्य हरियाणा है।
  • केन्द्रशासित प्रदेशों में सर्वाधिक वन प्रतिशत वाला केन्द्रशासित प्रदेश लक्षद्वीप जबकि सबसे कम वन प्रतिशत लद्दाख में पाया जाता है।
  • भारतीय वन अनुसंधान केन्द्र देहरादून में स्थित हैं।
  • सामाजिक वानिकी : समाज के लोगों के हस्तक्षेप से सरकारी भूमि पर जो वनों का विस्तार किया जाता है उसे सामाजिक वानिकी कहते हैं। इस कार्यक्रम की शुरूआत 1978 ई० से हुई है। 1980 ई० में इस कार्यक्रम को छठी पंचवर्षीय योजना का अंग बनाया गया।
  • कृषि वानिकी : किसी भूमि पर जब कृषि कार्य के साथ-साथ वन लगाये जाते हैं उसे ही कृषि वानिकी कहते हैं। कृषि वानिकी से मृदा अपरदन को रोका जाता है। 
  • वर्षा के आधार पर वनस्पति कई प्रकार के होते हैं-
    1. उष्णकटिबंधीय सदाबहार वनस्पति : वैसा क्षेत्र जहाँ 250 cm से अधिक वर्षा होता है उन क्षेत्रों में पायी जाने वाली वनस्पति को उष्णकटिबंधीय सदाबहार वनस्पति कहा जाता है। इसके अंतर्गत आने वाले वन सदा हरा-भरा रहता है जिस कारण इसे सदाबहार वनस्पति भी कहते हैं। यह वनस्पति पश्चिमी घाट, उत्तरी-पूर्वी पहाड़ी अंडमान निकोबार द्वीप समूह इत्यादि क्षेत्रों में पायी जाती हैं।
      • Note : (i) अंडमान निकोबार द्वीप समूह को उष्णकटिबंधीय सदाबहार वनस्पति का घर कहा जाता है।
        (ii) सभी प्रकार की वनस्पतियों में सर्वाधिक उत्पादकता वाला वनस्पति उष्णकटिबंधीय सदाबहार वनस्पति है। इसके प्रमुख वृक्ष निम्न हैं, जैसे- बाँस, रबड़, सिनकोना, आबनूस, रोजवुड इत्यादि ।
    2. अर्ध सदाबहार वनस्पति : वैसा क्षेत्र जहाँ 200 cm से 250 cm के बीच वर्षा होता है, उन क्षेत्रों में पायी जाने वाली वनस्पति को अर्धसदाबहार वनस्पति कहा जाता है। इस वनस्पति के प्रमुख वृक्ष साइडर, होलक, चंपा इत्यादि प्रमुख हैं। यह वन पश्चिमी घाट, मेघालय का पठारी इलाका, अंडमान निकोबार द्वीप समूह में पाया जाता है।
    3. उष्णकटिबंधीय आर्द्र पर्णपाती वनस्पति : वैसा क्षेत्र जहाँ 100 cm से 200 cm के बीच में वर्षा होता हैं, उन क्षेत्रों में पायी जाने वाली वनस्पति उष्ण कटिबंधीय आर्द्र पर्णपाती वनस्पति कहलाता है। भारत में सबसे ज्यादा यही वनस्पति पायी जाती है। वसंत एवं ग्रीष्म ऋतु के शुरू होने पर यह वनस्पति पत्ता गिरा देती है जिस कारण इसे पर्णपाती वनस्पति कहते हैं। इसके प्रमुख वृक्ष सागवान, शीशम, सखवा, साल, आम, कटहल, चंदन इत्यादि है। यह वनस्पति मध्य प्रदेश, महाराष्ट्र, छत्तीसगढ़, कर्नाटक, बिहार, उत्तर प्रदेश में पायी जाती है।
  • उष्ण कटिबंधीय शुष्क पर्णपाती वनस्पति : वैसा क्षेत्र जहाँ 70 cm से 100 cm के बीच में वर्षा होता है, उन क्षेत्रों में जो वनस्पति पायी जाती है, उसे उष्णकटिबंधीय शुष्क पर्णपाती वनस्पति कहा जाता है। इसके प्रमुख वृक्ष तेंदु, बेल इत्यादि हैं।
  • शुष्क कटीली वनस्पति : वैसा क्षेत्र जहाँ 70 cm से कम वर्षा होता है उन क्षेत्रों में पायी जाने वाली वनस्पति शुष्क कटीली वनस्पति कहलाती है, जैसे- बबूल, खजूर इत्यादि ।
  • सवाना वनस्पति : वैसा क्षेत्र जहाँ 40-60cm के बीच में वर्षा होता है, उन क्षेत्रों में पायी जाने वाली वनस्पति को सवाना वनस्पति कहते हैं। इस वनस्पति के अंतर्गत छोटे-छोटे घास के मैदान तथा छोटी-छोटी वृक्ष और झारियाँ आती है।
  • मरूस्थलीय वनस्पति : वैसा क्षेत्र जहाँ 50 cm से कम वर्षा होता है, उन क्षेत्रों में पायी जाने वाली वनस्पति मरुस्थलीय वनस्पति कहलाती है। जैसे- नागफनी, आकाशिया के वृक्ष इत्यादि ।
  • ज्वारीय वनस्पति : भारत के समुद्र तटीय राज्यों में ज्वारीय वनस्पति पायी जाती है। भारत में सबसे ज्यादा ज्वारीय वनस्पति पश्चिम बंगाल राज्य में पायी जाती है ! पश्चिम बंगाल के बाद दूसरा सबसे अधिक ज्वारीय वनस्पति गुजरात राज्य में पायी जाती है। ज्वारीय जन के प्रमुख वृक्ष सुंदरी और मैंग्रोव है। ज्वारीय वनस्पति को कई अन्य नाम जैसे- अनूप वन, कच्छ वनस्पति, वेलांचली वन कहा जाता है।
Note : 1. समुद्री और स्थलीय पारिस्थितिकी के बीच ज्वारीय वनस्पति सेतु के रूप में काम करता है ।
2. पश्चिमी हिमालय की तुलना में पूर्वी हिमालय में जैव विविधता अधिक पायी जाती है क्योंकि पूर्वी हिमालय विषुवत रेखा के नजदीक स्थित हैं तथा इन क्षेत्रों में अधिक होता है।
3. यूकेलिप्टस को पारिस्थितिकी का आतंकवाद कहा जाता है।
4. प्लास को जंगल की आग कहा जाता है। (ब्यूटियामोनोस्पर्मा)
  • आर्द्र भूमि (Wetlands) : वैसा क्षेत्र जहाँ नम और शुष्क दोनों प्रकार के वातावरण मौजूद हो उसे आर्द्र भूमि कहा जाता है। आर्द्रभूमि वाला क्षेत्र जैव विविधता को लेकर महत्वपूर्ण माना जाता है। इसलिए आर्द्र भूमि का संरक्षण प्रदान किया जाता है। आर्द्र भूमि के प्रति लोगों को जागरूक करने के उद्देश्य से प्रति वर्ष 2 फरवरी को विश्व आर्द्र भूमि दिवस मनाया जाता है।
  • विश्व के समस्त आर्द्र भूमियों को संरक्षण प्रदान करने के उद्देश्य से ईरान के रामसर शहर में 1971 में सम्मेलन आयोजित किया गया जिसे रामसर सम्मेलन कहते हैं।
  • भारतीय आर्द्र भूमि को रामसर साइट्स में शामिल किया जाना प्रारंभ 1982 ई० से हुआ।
  • बिहार का एकमात्र बेगुसराय का कांवर ताल झील को रामसर साइट्स में शामिल किया गया है। वही भारत के भीतर सर्वाधिक उत्तर प्रदेश के 10 स्थलों को रामसर साइट्स में शामिल किए गए हैं। जैसे - बखिरा वन्य जीव अभ्यारण्य, हैदरपुर आर्द्र भूमि, ऊपरी- गंगा नदी तंत्र, etc.
  • वर्तमान में भारत के 49 स्थलों को रामसर साइट्स में शामिल किए गए हैं। फरवरी, 2022 ई० में 2 स्थलों को रामसर साइटस में शामिल किया गया है जो निम्न हैं- 1. गुजरात का खिजड़िया वन्य जीव अभ्यारण्य और 2. उत्तर प्रदेश का बखिरा वन्य जीव अभयारण्य।

वस्तुनिष्ठ प्रश्न एवं उत्तर

1. इनमें कौन वन्य पदार्थ नहीं हैं ?
(a) इमारती लकड़ी 
(b) लाह
(c) ताँबा
(d) बाँस
2. महोगनी किस वन की उपज है ?
(a) पतझड़ वन
(b) चिरहरित वन
(c) कोणधारी वन
(d) ज्वारीय वन  
3. इनमें वनों का सबसे ऊँचा क्षेत्र कौन है ?
(a) कश्मीर से कुमाऊँ तक का क्षेत्र
(b) असम क्षेत्र
(c) नीलगिरि क्षेत्र
(d) मालाबार तटीय क्षेत्र
4. काजीरंगा राष्ट्रीय उद्यान कहाँ स्थित है ?
(a) मध्य प्रदेश में
(b) हिमालच प्रदेश में
(c) गुजरात में
(d) असम में
5. इनमें उत्तराखंड का अभयारण्य कौन है ?
(a) कार्बेट
(b) कान्हा
(c) शिवपुरी
(d) रणथंभौर
6. गिर कहाँ स्थित है ?
(a) राजस्थान में
(b) गुजरात में
(c) असम में
(d) मिजोरम में
7. भारत के किस राज्य में सुरक्षित वन सबसे अधिक मिलते हैं?
(a) झारखंड में 
(b) महाराष्ट्र में
(c) असम में
(d) मध्य प्रदेश में 
8. इनमें कौन कथन सही नहीं है ?
(a) वनों से भूमि कटाव रूकता है।
(b) वनों से पशुओं के लिए पर्याप्त चारा प्राप्त होता है।
(c) ऊष्मा जलवायु मिलने के कारण वनों से तापमान मे कमी आती है।
(d) वनों की उपस्थिति से नदियो में बाढ़ आ जाती है।
9. इनमें कौन वन्य जीव नहीं हैं ?
(a) भेड़िया
(b) भालू
(c) हाथी
(d) गाय
10. नदियों के डेल्टा भाग में किस प्रकार के वन मिलते हैं ?
(a) पतझड़ वन 
(b) ज्वारीय (मैग्रांत्र) वन
(c) सदाबहार वन
(d) शुष्क वन
11. देवदार के पेड़ किस प्रकार के वन में उगा करते हैं ?
(a) पर्णपाती वन में  
(b) पर्वतीय वन में
(c) शुष्क वन में
(d) चिरहरित वन में
12. नारियल, काजू और रवर के पेड़ अधिक कहाँ मिलते हैं ?
(a) पश्चिमी हिमालयी क्षेत्र में में
(b) पूर्वी हिमालयी क्षेत्र
(c) मालाबार तटीय क्षेत्र में
(d) असम क्षेत्र में
13. निम्नांकित में कौन भारत को छोड़ अन्य देश में दुर्लभ है ?
(a) भालू और गीदड़
(b) बाघ और सिंह
(c) गधे और हाथी
(d) हनुमान और हरिण
14. इनमें कौन हमारे देश से प्रायः लुप्त हो चुका है ?
(a) चीता
(b) हनुमान
(c) गैंडा
(d) तेंदुआ
15. इनमें कौन शाकाहारी नहीं है ? 
(a) गैंडा
(b) बारहसिंगा
(c) हरिण
(d) भेड़िया
16. इनमें किस पेड़ क लकड़ी बहुत कड़ी होती हैं ?
(a ) देवदार
(b) इंसलॉक 
(c) सागवान
(d) बलुत
17. इनमें कौन वन और वन्य प्राणियों के विनाश का कारण नहीं बनता ?
(a) कृषि क्षेत्रों में अत्यधिक वृद्धि
(b) बड़े पैमाने पर विकास कार्यों का होना
(c) व्यापार की वृद्धि
(d) पशुचारण और लकड़ी कटाई
18. 2001 में भारत में वनाच्छादित भू-भाग देश के कुल भू-भाग का कितना प्रतिशत था ?
(a) 19.27
(b) 20.60
(C) 20
(d) 25
19. भारत की राष्ट्रीय वन नीति के अनुसार, देश के कितने प्रतिशत भू-भाग पर वन रहना चाहिए ?
(a) 20 
(b) 22.5
(c) 33.3
(d) 40
20. भारत के किस राज्य में वनाच्छादित भू-भाग का क्षेत्र सबसे अधिक है ?
(a) केरल
(b) कर्नाटक
(c) मध्य प्रदेश
(d) उत्तर प्रदेश
21. वन संरक्षण और प्रबंधन की दृष्टि से वनों को कितने वर्गों में रखा गया है ?
(a) 3
(b) 4
(c) 5
(d) 6
22. भारतीय संविधान की धारा 21 का संबंध किससे है ?
(a) जल संसाधन संरक्षण
(b) मृदा संसाधन संरक्षण
(c) वन्य जीव और वनस्पतियों का संरक्षण
(d) सभी प्राकृतिक संसाधनों का संरक्षण
23. प्राकृतिक संसाधनों के संरक्षण हेतु अफ्रीकी कन्वेंशन किस वर्ष हुआ था ?
(a) 1960 में
(b) 1968 में
(c) 1972 में
(d) 1980 में
24. भारत का राष्ट्रीय पक्षी कौन है ?
(a) तोता
(b) कबूतर
(c) मोर
(d) हंस
25. भारत में सफेद बाघ के लिए प्रजनन केंद्र किस राज्य में स्थापित किया है ?
(a) महाराष्ट्र में
(b) गुजरात में
(c) ओडिशा में
(d) आंध्र प्रदेश में
26. किस वर्ग की विलुप्त प्रजातियों की संख्या सबसे अधिक है ?
(a) वनस्पति 
(b) पक्षी
(c) मछलियाँ
(d) सरीसृप
27. किस भाग में मैंग्रोव वन का विस्तार सबसे अधिक है ?
(a) पूर्वोत्तर राज्य
(b) पश्चिमी तट
(c) सुंदरवन
(d) अंडमान-निकोबार द्वीपसमूह
28. चरक का संबंध किस देश से है ?
(a) नेपाल 
(b) भारत
(c) श्रीलंका
(d) चीन
29. चिपको आंदोलन से संबंधित व्यक्ति कौन है ?
(a) सुंदरलाल बहुगुणा 
(b) मेघा पाटेकर
(c) अमत्वं सेन 
(d) हेमवतीनंदन बहुगुणा
30. वह वनस्पति जो मनुष्य की सहायता के बिना अपने आप पैदा होती है और लंबे समय तक बिना किसी मानवीय हस्तक्षेप के उगती है, इस प्रकार की वनस्पति को कहते हैं-
(a) देशज वनस्पति (Endemic or Indigenous Vegetation)
(b) विदेशज वनस्पति (Exotic Vegetation)
(c) अक्षत वनस्पति (Virgin Vegetation)
(d) इनमें से कोई नहीं
31. उष्णकटिबंधीय सदाबहार वन भारत के किन-किन क्षेत्रों में पाये जाते हैं ?
1. पश्चिमी घट के पूर्वी ढाल पर
2. अंडमान और निकोबार द्वीप समूह में
3. अरावली पर्वत श्रृंखला पर
4. उत्तर-पूर्वी क्षेत्र की पहाड़ियों पर
नीचे दिए गए कूट का प्रयोग कर सही उत्तर चुनिये :
(a) केवल 1 और 2
(b) केवल 1, 2 और 3
(c) केवल 2 और 4
(d) केवल 3 और 4
32. निम्नलिखित में से कौन-सा से कथन उष्णकटिबंधीय सदाबहार वनों के संदर्भ में सही है/हैं ?
1. ये वन शीत और शुष्क प्रदेशों में पाये जाते हैं।
2. इन वन क्षेत्रों में वार्षिक वर्षा 200 सेंटीमीटर से अधिक होती है।
3. ये वन विरल होते हैं।
कूट :
(a) केवल 1 और 2
(b) केवल 2 
(c) केवल 2 और 3
(d) 1, 2 और 3
33. भारत में सर्वाधिक क्षेत्र पर किस प्रकार के वनों का विस्तार है ?
(a) उष्णकटिबंधीय पर्णपाती वन
(b) कँटीले वन
(c) उष्णकटिबंधीय सदाबहार वन
(d) पर्वतीय वन
34. चंदन, आँवला, महुआ, सागौन, बाँस आदि प्रजातियों के वृक्ष किस प्रकार के वनों मं पाये जाते हैं ?
(a) उष्णकटिबंधीय सदाबहार वन
(b) शीतोष्ण कटिबंधीय सदाबहार वन
(c) आर्द्र पर्णपाती वन
(d) शुष्क पर्णपाती वन
35. शुष्क पर्णपाती वनों के संदर्भ में विचार कीजिये -
1. ये वन उन क्षेत्रों में पाये जाते हैं, जहाँ वर्षा 70 से 100 सेंटीमीटर होती है।
2. शुष्क ऋतु शुरू होते ही शुष्क पर्णपाती वृक्षों के पत्ते झड़ जाते हैं और घास के मैदान में नग्न पेड़ खड़े रह जाते हैं।
उपर्युक्त में से कौन- सा/से कथन सत्य है / हैं ? 
(a) केवल 1 
(b) केवल 2
(c) 1 और 2 दोनों
(d) न तो 1 और न ही 2
36. हिमालयी पर्वतीय क्षेत्र में नीचे से ऊपर की ओर जाने पर पाई जाने वाली वनस्पति का सही क्रम कौन-सी है ?
(a) पर्णपाती वन - आर्द्र शीतोष्ण कटिबधाय वन - शीतोष्ण कटिबंधीय घास - लाइकेन
(b) पर्णपाती वन - आर्द्र शीतोष्ण कटिबंधीय वन - लाइकेन - शीतोष्ण कटिबंधीय घास
(c) आर्द्र शीतोष्ण कटिबंधीय वन - पर्णपाती वन - लाइकेन- शीतोष्ण कटिबंधीय घास
(d) आर्द्र शीतोष्ण कटिबंधीय वन - पर्णपाती वन - शीतोष्ण कटिबंधीय घास - लाइकेन 
37. बहुमूल्य वृक्ष प्रजाति 'देवदार के वन' पाये जाते हैं-
(a) पूर्वी हिमालय में
(b) पश्चिमी हिमालय में 
(c) पश्चिमी घाट में
(d) नीलगिरि पर्वत श्रृंखला में 
38. प्रायद्वीपीय भारत में पर्वतीय वन मुख्यतः पाये जाते हैं-
1. पश्चिमी घाट में
2. पूर्वी घाट में
3. नीलगिरि पर्वत श्रृंखला में
4. विंध्याचल पर्वत श्रृंखला में
कूट :
(a) केवल 1 और 2
(b) केवल 2 और 3
(c) केवल 1, 3 और 4
(d) केवल 3 और 4
39. भारत में 'शोलास' वन पाये जाते हैं-
1. नीलगिरी पहाड़ियों पर
2. विंध्याचल पहाड़ियों पर
3. अन्नमलाई पहाड़ियों पर
4. पालनी पहाड़ियों पर
कूट :
(a) केवल 1 और 2
(b) केवल 2, 3 और 4
(c) केवल 3 और 4
(d) केवल 1, 3 और 4
40. मैंग्रोव वनों के संदर्भ में नीचे दिये गए कथनों पर विचार कीजिये :
1. मैंग्रोव तटवर्ती क्षेत्रों में जहाँ ज्वार-भाटा आते हैं, वहाँ की सबसे महत्वपूर्ण वनस्पति है ।
2. मैंग्रोव पौधों की जड़ें पानी में डूबी रहती है।
उपर्युक्त में से कौन-सा/से कथन सत्य है/हैं ?
(a) केवल 1 
(b) केवल 2
(c) 1 और 2 दोनों
(d) न तो । और न ही 2
41. निम्नलिखित में से किन नदियों के डेल्टाई भाग में मैंग्रोव वन पाये जाते हैं ?
1. गंगा
2. गोदावरी
3. महानदी
4. कृष्णा
कूट:
(a) केवल 1 और 2
(b) केवल 1, 2 और 3
(c) केवल 1, 3 और 4
(d) उपर्युक्त सभी
42. सामाजिक वानिकी में शामिल किया जाता है-
1. शहरी वानिकी 
2. ग्रामीण वानिकी
3. मरुस्थलीय वानिकी
4. फॉर्म वानिकी
कूट :
(a) केवल 1 और 2
(b) केवल 2 और 3
(c) केवल 1, 2 और 3
(d) केवल 1, 2 और 4
43. निम्नलिखित में से कौन-सा / से युग्म सुमेलित है / हैं ?
1. वन्य प्राणी अधिनियम - 1972 
2. प्रोजेक्ट टाइगर - 1973
3. प्रोजेक्ट एलीफेंट - 1992
कूट:
(a) केवल 1 और 2
(b) केवल 2 और 3
(c) केवल 3
(d) उपर्युक्त सभी
44. वन्य प्राणी अधिनियम, 1972 के संबंध में नीचे दिये गए कथनों पर विचार कीजिये :
1. यह अधिनियम वन्य प्राणियों के संरक्षण की कानूनी रूपरेखा तैयार करता है ।
2. इस अधिनियम में नेशनल पार्क, पशु विहार जैसे संरक्षित क्षेत्रों को कानूनी सहायता प्रदान की गई है।
उपर्युक्त में से कौन -सा/से कथन सत्य है हैं ?
(a) केवल 1
(b) केवल 2
(c) 1 और 2 दोनों
(d) न तो 1 और न ही 2
हमसे जुड़ें, हमें फॉलो करे ..
  • Telegram ग्रुप ज्वाइन करे – Click Here
  • Facebook पर फॉलो करे – Click Here
  • Facebook ग्रुप ज्वाइन करे – Click Here
  • Google News ज्वाइन करे – Click Here
]]>
Thu, 11 Apr 2024 07:03:26 +0530 Jaankari Rakho
General Competition | Geography | भारत के उद्योग https://m.jaankarirakho.com/959 https://m.jaankarirakho.com/959 General Competition | Geography | भारत के उद्योग
  • उद्योग का तात्पर्य वैसी आर्थिक उच्च क्रियाओं से है जिसके अंतर्गत वस्तु एवं सेवाओं का उत्पादन एवं संवर्धन किया जाता है। 
  • स्वतंत्र भारत की प्रथम औद्योगिक नीति अप्रैल, 1948 में आयी थी वही भारत की द्वितीय औद्योगिक नीति अप्रैल, 1956 में आयी थी।
  • भारत की तीसरी और नई औद्योगिक नीति जुलाई, 1991 में आयी है जिसमें उदारीकरण, निजीकरण और वैश्वीकरण की घोषणा की गई है।
  • लौह-इस्पात उद्योग : इस उद्योग को आधारभूत उद्योग कहा जाता है क्योंकि यह अन्य उद्योगों जैसे - मशीनरी उद्योग, औजार उद्योग इत्यादि को कच्चा माल प्रदान करता है। भारत में आधुनिक रूप से इस्पात बनाने का प्रथम प्रयास 1830 ई० में तमिलनाडु के पोटोनोवा में किया गया। लेकिन यह असफल रहा।
  • देश का प्रथम लौह इस्पात संयंत्र पश्चिम बंगाल के कल्टी में 1874 ई. में बंगाल आयरन वर्क्स के नाम से स्थापित हुआ लेकिन फंड के अभाव के कारण यह असफल हो गया। बाद में बंगाल की सरकार ने इसका अधिग्रहण कर लिया और इसका नाम बदलकर बराकर आयरन वर्क्स कर दिया। 
  • देश का बड़े पैमाने पर प्रथम सफलतम लौह इस्पात उद्योग TISCO (Tata Iron and Steel Company) है। यह 1907 ई० में जमशेदजी टाटा के द्वारा झारखंड राज्य के जमशेदपुर के साक्ची में स्थापित किया गया था।
  • TISCO को लौह अयस्क की प्राप्ति ओडिशा के बादाम पहाड़ी, गुरूमहिसानी और झारखंड के सिंहभूम से होता है।
  • TISCO को कोयला की प्राप्ति झरिया और बोकारो से होता है।
  • IISCO (Indian iron and steel company) : इसकी स्थापना 1918 ई० में पश्चिम बंगाल के बर्नपुर में दामोदर नदी के किनारे की गई थी। इसको कोयला की प्राप्ति झारखंड के झरिया और रामगढ़ से तथा लौह अयस्क की प्राप्ति सिंहभूम से होता है।
  • विश्वेरैया आयरन एण्ड स्टील लिमिटेड (VISL) : यह कर्नाटक के शिमोगा जिले में भद्रा नदी के तट पर 1923 ई० में स्थापित किया गया है। इस संयंत्र को लौह-अयस्क की प्राप्ति कर्नाटक के बाबाबूदन की पहाड़ी से होता है। यह एक ऐसा संयंत्र है जो कोयला क्षेत्र से दूर स्थापित किया गया है। इसे पहले मैसूर आयरन वर्क्स के नाम से जाना जाता था ।
  • द्वितीय पंचवर्षीय योजना के दौरान स्थापित लौह इस्पात संयंत्र : द्वितीय पंचवर्षीय योजना के दौरान भारत सरकार ने भारी उद्योग को बढ़ावा दिया। इस पंचवर्षीय योजना के दौरान तीन बड़े लौह-इस्पात संयंत्र भारत के अलग-अलग क्षेत्रों में स्थापित किये गये जो निम्न हैं-
    1. हिंदुस्तान स्टील लिमिटेड, भिलाई : यह वर्तमान में छत्तीसगढ़ राज्य में है। यह इस्पात संयंत्र रूस की मदद से स्थापित हुआ। 1959 से प्लांट उत्पादन कार्य आरंभ हुआ। इसका कोयला की प्राप्ति कोरबा (छत्तीसगढ़), बोकारो व झरिया से होता है तथा लौह अयस्क की प्राप्ति दल्लिराजहरा (ओडिशा) से होता है ।
    2. हिंदुस्तान स्टील लिमिटेड, राउरकेला : इसकी स्थापना जर्मनी की सहयोग से वर्ष 1959 ई॰ में ओडिशा में शंख एवं कोयल नदी के संगम पर हुआ है। इसको कोवला की प्राप्ति झरिया व कोरबा से होता है।
    3. हिन्दुस्तान स्टील लिमिटेड, दुर्गापुर: इसकी स्थापना 1959 ई० में ब्रिटेन की सहायता से प॰ बंगाल में हुआ। इसको कोयला की प्राप्ति रानीगंज और झरिया से होता है।
  • तीसरे पंचवर्षीय योजना के दौरान झारखंड के बोकारो में 1964 ई० में रूस की मदद से बोकारो इस्पात संयंत्र की स्थापना की गई। इसको कोयला बोकारो व झरिया से लौह अयस्क किरिबुरू से प्राप्त होता है।
  • विशाखापत्तनम लौह इस्पात संयंत्र : यह देश का प्रथम तटवर्ती या तटीय लौह इस्पात संयंत्र है इसकी स्थापना रूस की मदद से हुई है। यह आंध्र प्रदेश राज्य में स्थित हैं। इसका लौह अयस्क की प्राप्ति बेलाडिला के खान (छत्तीसगढ़) से होता है।
  • सलेह लौह इस्पात संयंत्र : यह तमिलनाडु राज्य में स्थित है। यह लौह इस्पात संयंत्र 1982 ई० से काम करना शुरू किया है। यह लौह इस्पात संयंत्र स्टेनलेस स्टील का उत्पादन करता है।
  • विजयनगर स्टील संयंत्र (कर्नाटक) : यह कर्नाटक के विजयनगर जिले के दासपेट क्षेत्र में तुंगभद्रा जलाशय के समीप स्थापित किया गया है। इसे लौह अयस्क की प्राप्ति बाबाबूदन की पहाड़ी से होता है।
  • एल्युमीनियम उद्योग : लौह इस्पात उद्योग के बाद दूसरा सबसे महत्वपूर्ण उद्योग एल्युमीनियम उद्योग है। देश में सर्वप्रथम एल्युमीनियम कम्पनी की स्थापना 1937 ई० में पश्चिम बंगाल के जे० के० नगर में एल्युमीनियम कॉर्पोरेशन ऑफ इंडिया नाम से किया गया।
  • द्वितीय पंचवर्षीय योजना के दौरान ओडिशा के के मेटुर तथा चतुर्थ पंचवर्षीय योजना के दौरान एल्युमीनियम कम्पनी हीराकुंड, उत्तर प्रदेश के रेणुकुंड, तृतीय पंचवर्षीय योजना के दौरान तमिलनाडु कर्नाटक के बेलगाँव में नये एल्युमीनियम कम्पनी की स्थापना हुई।
एल्युमीनियम कम्पनी सहयोगी देश स्थापित केन्द्र
BALCO (भारत एल्युमीनियम कम्पनी) रूस कोरबा (छत्तीसगढ़)
NALCO (नेशनल एल्युमीनियम कम्पनी) फ्रांस रत्नागिरी (महाराष्ट्र)
HINDALCO (हिंदुस्तान एल्युमीनियम कम्पनी) यू० एस० ए० रेणुकुंड (उत्तर प्रदेश)
IINDALCO (इंडियन एल्युमीनियम कम्पनी) कनाडा मुरी (बिहार), अल्वाय (केरल) और जे० के० नगर (प. बंगाल)
MALCO (मद्रास एल्युमीनियम कम्पनी) इटली चेन्नई, मेटुर, सलेम
वेदांता एल्युमीनियम कम्पनी जर्मनी ओडिशा के झारसुगुरा
  • सूती वस्त्र उद्योग : प्राचीन काल से ही भारत में सूती वस्त्र का उत्पादन किया जा रहा है। भारत में सर्वाधिक रोजगार कृषि के क्षेत्र में लोगों को प्राप्त होता है। कृषि के बाद दूसरा सर्वाधिक रोजगार देने वाला क्षेत्र सूती वस्त्र उद्योग है। भारत में पहला सूती वस्त्र उद्योग पश्चिम बंगाल के फोर्ट ग्लास्टर में 1818 ई. में स्थापित हुआ जो कि असफल रहा। वही देश का प्रथम सफलतम सूती वस्त्र उद्योग 1854 में मुंबई में कावस जी डाबर के द्वारा स्थापित किया गया। इस उद्योग में उत्पादन कार्य 1856 ई० में प्रारंभ हुआ।
  • मैनचेस्टर : यह ब्रिटेन का एक प्रमुख औद्योगिक नगर है जो सूती वस्त्र उत्पादन के लिए प्रसिद्ध है। उत्तर भारत का मैनचेस्टर कानपुर को कहा जाता है। दक्षिण भारत का मैनचेस्टर कोयम्बटूर को कहा जाता है।
  • भारत का मैनचेस्टर अहमदाबाद को कहा जाता है।
  • सूती वस्त्रों की राजधानी मुंबई को कहा जाता है।
  • वस्त्र पार्क : सिले सिलाए वस्त्रों के निर्यात को बढ़ावा देने के लिए देश का पहला वस्त्र पार्क तमिलनाडु के तिरूपुर जिला में स्थापित किया गया।
  • रेशमी वस्त्र उद्योग : भारत एक ऐसा देश है जहाँ शहतूती, इरी, टसर, ओक एवं मूँगा सभी किस्मों के रेशम का उत्पादन करता है। मूँगा रेशम के उत्पादन में भारत का एकाधिकार है। भारत में सबसे अधिक रेशम का उत्पादन कर्नाटक राज्य करता है। रेशमी वस्त्र उद्योग का प्रमुख केन्द्र कर्नाटक, आंध्र प्रदेश, असम और प० बंगाल का क्षेत्र है ।
  • Note : केन्द्रीय रेशम अनुसंधान एवं प्रशिक्षण केन्द्र मैसूर एवं ब्रह्मपुर में स्थित है।
  • ऊनी वस्त्र उद्योग : भारत में प्रथम ऊनी वस्त्र उद्योग 1876 ई० में कानपुर में स्थापित किया गया लेकिन ऊनी वस्त्र उद्योग का विकास आजादी के बाद भारत में संभव हो पाया। सामान्यतः ऊनी वस्त्र उद्योग को बढ़ावा देने के लिए अंगोरा ऊन और पश्मीना ऊन का प्रयोग किया जाता है। अंगोरा ऊन की प्राप्ति खरगोश के रोएँ से होता है जबकि पश्मीना ऊन की प्राप्ति बकरियों के रोएँ से होता है। ऊनी वस्त्र उद्योग का प्रमुख केन्द्र जम्मू-कश्मीर, पंजाब, हरियाणा इत्यादि राज्य है।
  • जूट उद्योग (पटसन ) : जूट को गोल्डन फाइबर ऑफ इंडिया (सोने का रेशा ) कहा जाता है। जूट उद्योग कच्चा माल पर आधारित उद्योग है। भारत में प्रथम जूट उद्योग 1855 ई० में जॉर्ज ऑकलैंड के द्वारा पश्चिम बंगाल के रिसरों में स्थापित किया गया। भारत में जूट का सर्वाधिक उत्पादन पश्चिम बंगाल राज्य करता है। पश्चिम बंगाल के बाद जूट उत्पादन में बिहार का दूसरा स्थान हैं। भारतीय जूट को कड़ी प्रतिस्पर्धा बांग्लादेश के जूट से मिलती है। भारतीय जूट के निर्यात को बढ़ावा देने हेतु भारतीय जूट निगम की स्थापना 1971 ई० में हुई थी। वही अंतर्राष्ट्रीय जूट संगठन की स्थापना 1984 ई० में हुई जिसका मुख्यालय ढाका में है।
  • चीनी उद्योग : यह कच्चे माल पर आधारित उद्योग हैं। यह उद्योग गन्ना की उपलब्धता पर निर्भर करता है । भारत वर्ष के भीतर प्रथम चीनी मिल या चीनी उद्योग बिहार के सारण के मढ़ौरा में 1903 ई० में स्थापित हुआ (कुछ अन्य स्रोत 1904 ई० देता है)।
  • गन्ना उत्पादन की आदर्श जलवायु दक्षिण भारत में पायी जाती है। (उष्ण कटिबंधीय ) 100-150 cm वर्षा ।
  • देश में सबसे ज्यादा गन्ना का उत्पादन उत्तर प्रदेश राज्य करता है ।
  • प्रति हेक्टेयर के दृष्टिकोण से सबसे ज्यादा गन्ना का उत्पादन तमिलनाडु राज्य करता है।
  • चीनी का सबसे ज्यादा उत्पादन उत्तर प्रदेश राज्य करता है ।
  • सबसे ज्यादा चीनी मील महाराष्ट्र राज्य में है।
  • दियासलाई उद्योग : भारत में प्रथम दियासलाई उद्योग गूजरात के अहमदाबाद में 1921 ई० में स्थापित हुआ था। यह कुटीर उद्योग के अंतर्गत आता हैं।
  • कागज उद्योग : कागज की खोज 12वीं, 13वीं सदी में चीन में हुआ था। कागज का उपयोग शैक्षणिक कार्यों में, अखबारों में, नोटों के रूप में किया जाता है। देश में प्रथम कागज उद्योग 1812 ई० में पश्चिम बंगाल के सेरामपुर में किया गया जो कि असफल रहा वही देश का प्रथम सफलतम कागज उद्योग पश्चिम बंगाल के बालीगंज में 1867 ई० में स्थापित हुआ। उत्तर प्रदेश के लखनऊ में प्रथम कागज उद्योग 1879 ई० में स्थापित हुआ।
  • Note : मध्य प्रदेश के नेपानगर में अखबारी कागज का उत्पादन होना शुरू 1947 ई० से हुआ है।
  • उर्वरक उद्योग : अधिक फसलों के उत्पादन हेतु किसानों के द्वारा अपने खेत में उर्वरक का प्रयोग किया जाता है। भारत में प्रथम उर्वरक उद्योग तमिलनाडु के रानीपेट 1906 में स्थापित किया गया। लेकिन देश में वास्तविक तौर पर उर्वरक उद्योग का विकास होना प्रारंभ आजादी के बाद तब हुआ जब 1951 में झारखंड के सिंदरी में सिंदरी उर्वरक कारखाना की स्थापना हुई ।
  • सीमेंट उद्योग : बड़े-बड़े पुल भवन अथवा वारभूत संरचना के विकास हेतु सीमेंट का उपयोग किया जाता है। आधुनिक ढंग से पहली बार सीमेंट का उत्पादन 1824 में ब्रिटेन के पोर्टलैंड शहर में हुआ। भारत में प्रथम सीमेंट उद्योग कारखाना 1904 में मद्रास में स्थापित हुआ जो असफल रहा। देश का प्रथम सफलतम सीमेंट उद्योग गुजरात के पोरबंदर में 1914 ई० में स्थापित हुआ। प्रथम स्थान - गुजरात |
  • Note : ACC (Associate Cement Company) की स्थापना 1936 ई० में हुई।
  • औद्योगिक क्षेत्र : भारत के उन क्षेत्रों में बड़े उद्योग मिलते हैं जहाँ कच्चे माल की उपलब्धता हो, परिवहन की अच्छी व्यवस्था हो, कम मजदूरी पर श्रमिकों की उपलब्धता हो इत्यादि । भारत को 8 बड़े औद्योगिक क्षेत्र और 13 लघु औद्योगिक क्षेत्र में बाँटा गया है। 8 बड़े औद्योगिक क्षेत्र निम्न हैं-
    1. गुरूग्राम - दिल्ली-मेरठ औद्योगिक क्षेत्र 
    2. गुजरात औद्योगिक क्षेत्र 
    3. 3. मुंबई - पुणे औद्योगिक क्षेत्र
    4. बंगलुरू- तमिलनाडु औद्योगिक क्षेत्र
    5. कोल्लम-तिरूवनंतपुरम औद्योगिक क्षेत्र
    6. विशाखापत्तनम - गुंटुर औद्योगिक क्षेत्र
    7. छोटानागपुर औद्योगिक क्षेत्र
    8. हुगली औद्योगिक क्षेत्र |
  • Note : 1. मोटरगाड़ी उद्योग का विकास उद्योग कहा जाता है।
    2. प्लास्टिक उद्योग को सनराइज इंडस्ट्री कहा जाता है।
    3. सॉफ्टवेयर उद्योग को फुटलुज उद्योग की श्रेणी में रखा जाता है।
  • फुटलुज उद्योग : वैसे उद्योग जिसको किसी भी स्थान पर स्थापित करने से कोई हानि नहीं होता है, उसे फुटलुज उद्योग कहा जाता है।

वस्तुनिष्ठ प्रश्न एवं उत्तर

1. इनमें कौन-सा कथन सही नहीं है ?
(a) जमशेदपुर का लोहा-इस्पात कारखाना सुवर्णरेखा नदी के तट पर स्थित है।
(b) दुर्गापुर सूती कपड़े का उद्योग केंद्र है।
(c) भिलाई छत्तीसगढ़ राज्य में स्थित है।
(d) कानपुर में सूती और ऊनी कपड़े बनते हैं।
2. भद्रावती में कीन उद्योग स्थापित है ?
(a) जूट
(b) रेशमी वस्त्र
(c) रासायनिक
(d) लोहा-इस्पात
3. जूट उद्योग के सबसे अधिक कारखाने कहाँ मिलते हैं ?
(a) ब्रह्मपुत्र के तट पर
(b) कोसी के तट पर
(c) हुगली के तट पर
(d) महानदी के तट पर
4. बिहार राज्य के किस क्षेत्र में चीनी की अधिकतर मिलें स्थापित हैं ?
(a) पूर्णिया
(b) भागलपुर
(c) पटना
(d) सीवान चंपारण
5. भारत में तांबा गलाने का आधुनिक कारखाना सबसे पहले कहाँ खुला ?
(a) तूतीकोरिन में
(b) खेतड़ी में
(c) घाटशिला में
(d) जमशेदपुर में
6. कहाँ पर रेलवे वर्कशॉप एशिया में सबसे पुराना है ?
(a) वाराणसी का
(b) कपूरथला का
(c) पेराम्बूर का
(d) जमलापुर का
7. भारत का कौन-सा इलेक्ट्रॉनिक उद्योग की राजधानी है ?
(a) कोलकाता 
(b) बेंगलुरु 
(c) दिल्ली
(d) पुणे
8. बेंगलुरु किस राज्य में है ?
(a) कर्नाटक में 
(b) महाराष्ट्र में
(c) तमिलनाडु में
(d) आंध्र प्रदेश में
9. सलेम (सेलम) किस राज्य में स्थित है ?
(a) केरल में
(b) तमिलनाडु में
(c) कर्नाटक में
(d) महाराष्ट्र में
10. ऐलुमिनियम उद्योग का कच्चा माल क्या है ?
(a) हेमाटाइट
(b) बॉक्साइट
(c) सिडेराइट
(d) इनमें से कोई नहीं
11. सीमेंट उद्योग का कच्चा माल क्या है ?
(a) लौह अयस्क
(b) बलुआ पत्थर
(c) चूना पत्थर
(d) अवरक
12. भारत में डीजल इंजन कहाँ बनते हैं ?
(a) वाराणसी में
(b) कपूरथला में
(c) पेराम्बूर मैं
(d) बेंगलुरु में
13. सिंदरी कहाँ स्थित है ?
(a) पश्चिम बंगाल में
(b) झारखंड में
(c) छत्तीसगढ़ में
(d) ओडिशा में
14. जमशेदपुर किस उद्योग के लिए प्रसिद्ध है ?
(a) चीनी
(b) सीमेंट
(c) लोहा-इस्पात
(d) सूती कपड़ा
15. चीनी उत्पादन में कौन राज्य भारत में सबसे आगे है ?
(a) पश्चिम बंगाल
(b) उत्तर प्रदेश
(c) महाराष्ट्र
(d) पंजाब
16. इनमें कौन सूती वस्त्र उत्पादन में अग्रणी है ?
(a) कानपुर
(b) कोलकाता
(c) चेन्नई
(d) नागपुर
17. इनमें कोन इस्पात केंद्र समुद्र के निकट है ?
(a) त्रिनगर
(b) बोकारो
(c) भिलाई
(d) भद्रावती
18. बॉदसाइट का उपयोग किस उद्योग में होता है ?
(a) बूट
(b) सीमेंट
(c) ऐलुमिनियम
(d) ताँबा
19. चूनापत्थर का उपयोग किस उद्योग में होता है ?
(a) पटसन 
(b) सीमेंट
(c) चीनी
(d) ऐलुमिनियम
20. निम्नलिखित में कौन उद्योगों का स्थिति के निर्धारण में महत्वपूर्ण है ?
(a) बाजार
(b) जनसंख्या
(c) ऊर्जा
(d) इनमें सभी
21. भारत में स्थापित सबसे पहला सफल लोहा-इस्पात उद्योग कौन है ?
(a) भारतीय लोहा और इस्पात कंपनी (IISCO)
(b) बोकारो स्टील लिमिटेड (BSL)
(c) टाटा लोहा और इस्पात कंपनी (TISCO)
(d) विश्वेश्वरैया लोहा और इस्पात लिमिटेड (VISL)
22. भारत में पहली सूती मिल की स्थापना मुंबई में करने का कारण क्या था ?
(a) पत्तन की सुविधा 
(b) कपास - उत्पादन क्षेत्र होना
(c) पूँजी की उपलब्धता
(d) इनमें सभी
23. निम्नलिखित में कौन कृषि आधारित उद्योग नहीं है ?
(a) सूती वस्त्र उद्योग 
(b) चीनी उद्योग
(c) सीमेंट उद्योग
(d) जूट-वस्त्र उद्योग जी.एस
24. निम्नलिखित में कौन सार्वजनिक क्षेत्र का उद्योग है ?
(a) जे. के. सीमेंट
(b) टाटा लोहा और इस्पात
(c) बोकारो इस्पात
(d) रेमंड-वस्त्र उद्योग
25. विश्व के साइकिल उत्पादन देशों में भारत को कौन-सा स्थान प्राप्त है ?
(a) पाँचवाँ
(b) चौथा
(c) तीसरा
(d) दूसरा
26. बिजनेस प्रोसेस आउटसोर्सिंग (BPO) में विश्व में भारत का क्या स्थान है ?
(a) पहला 
(b) दूसरा
(c) तीसरा
(d) चौथा
27. निम्नलिखित में से कौन-से कारक उद्योगों की स्थापना को प्रभावित करते हैं ?
1. कच्चे माल की उपलब्धता
2. श्रमिक
3. ऊर्जा के साधन
4. बाजार
कूट :
(a) केवल 1 और 2
(b) केवल 1, 2 और 4 
(c) कंवल 2, 3 और 4
(d) उपर्युक्त सभी
28. निम्नलिखित में से कौन-सा कथन 'समूहन बचत' (Ag- glomeration Economics) को परिभाषित करता है ?
(a) सुदूर पिछड़े एवं अविकसित क्षेत्रों में उद्योगों का संकेंद्रण
(b) उद्योगों का नगरों के पास संकेंद्रण
(c) लोगों द्वारा अपनी आवश्यकता के अनुरूप उद्योगों की स्थापना
(d) इनमें से कोई नहीं
29. पटसन उद्योग के संबंध में नीचे दिये गए कथनों पर विचार कीजिये :
1. भारत पटसन व पटसन निर्मित सामान का सबसे बड़ा उत्पादक देश है।
2. भारत में पटसन उद्योग का अधिकांश भाग हुगली नदी के तट पर विकसित हुआ है।
उपर्युक्त में से कौन - सा/से कथन सत्य है / हैं ?
(a) केवल 1
(b) केवल 2
(c) 1 और 2 दोनों
(d) न तो 1 और न ही 2
30. चीनी उद्योग के संबंध में नीचे दिये गए कथनों पर विचार कीजिये:
1. गन्ने की चीनी मिलों तक दुलाई में ज्यादा समय लगने से इसके सुक्रोज की मात्रा घट जाती है।
2. दक्षिणी और पश्चिमी राज्यों में गन्ने में उत्तर भारत के गन्ने की अपेक्षा अधिक मात्रा में सुक्रोज पाया जाता है।
उपर्युक्त में से कौन - सा/से कथन सत्य है / हैं ?
(a) केवल 1
(b) केवल 2
(c) 1 और 2 दोनों
(d) न तो 1 और न ही 2
31. लौह-इस्पात उद्योग में कच्चे माल के रूप में किन खनिजों का उपयोग किया जाता है ?
1. लौह अयस्क
2. कोकिंग कोल
3. सिलिका
4. चूना पत्थर
कूट :
(a) केवल 1 और 2
(b) केवल 1, 2 और 4
(c) केवल 1, 2 और 3
(d) केवल 1 और 4
32. विभिन्तन उर्वरकों उनमें प्रमुख यौगिक के संबंध में नीचे दिये गए युग्मों में से कौन-सा / से सुमेलित है / हैं ?
1. यूरिया - नाइट्रोजन उर्वरक
2. डी.ए.पी. (DAP) - नाइट्रोजन, फॉस्फोरस और पोटाश
3. मिश्रित उर्वरक - फॉस्फेट उर्वरक
कूट :
(a) केवल 1
(b) केवल 1 और 2 
(c) केवल 2 और 3 
(d) केवल 3
33. सीमेंट उद्योग में कच्चे माल के रूप में किन खनिजों का प्रयोग किया जाता है ?
1. चूना पत्थर
2. सिलिका
3. एल्युमिना
4. जिप्सम
कूट :
(a) केवल 1 और 2
(b) केवल 1 और 3
(c) केवल 1, 3 और 4
(d) 1, 2, 3 और 4
34. निम्नलिखित में से कौन-सा उद्योग बॉक्साइट को कच्चे माल के रूप में प्रयोग करता है?
(a) एल्युमीनियम 
(b) सीमेंट
(c) स्टील
(d) काँच
35. निम्नलिखित कथनों पर विचार कीजिये:
1. सार्वजनिक क्षेत्र के उद्योग सरकार द्वारा निधि प्रदत्त होते हैं।
2. सार्वजनिक क्षेत्रक में सामरिक और राष्ट्रीय महत्त्व के उद्योग-धंधे नहीं आते हैं।
उपर्युक्त में से कौन-सा /से कथन सही नहीं है / हैं?
(a) केवल 1
(b) केवल 2
(c) 1 और 2 दोनों
(d) न तो 1 और न ही 2 
36. भारत के उत्तर प्रदेश, हरियाणा, पंजाब, राजस्थान और गुजरात राज्यों में लौह इस्पात उद्योग के न होने का प्रमुख कारण है; 
(a) जनसंख्या घनत्व
(b) कच्चा माल
(c) बाजार
(d) परिवहन
37. 'टिस्को' संयंत्र किसके नजदीक स्थित है ?
(a) पटना 
(b) दरभंगा
(c) धनबाद
(d) टाटानगर
38. निम्नलिखित कथनों पर विचार कीजिये :
1. इस्को (IISCO) ने अपना पहला कारखाना झारखंड में स्थापित किया।
2. भारत के राउरकेला इस्पात संयंत्र की स्थापना जापान के सहयोग से की गई थी।
3. भिलाई इस्पात संयंत्र की स्थापना जर्मनी के सहयोग से की गई थी। 
उपर्युक्त में से कौन /सा/से कथन सही नहीं है/हैं?
(a) केवल 1 
(b) केवल 3
(c) केवल 2 और 3
(d) 1, 2 और 3
39. एकीकृत इस्पात कारखानों के संदर्भ में निम्नलिखित कथनों पर विचार कीजिये:
1. एकीकृत इस्पात संयंत्रों के प्रबंधन के लिये 1973 में हिंदुस्तान स्टील लिमिटेड की स्थापना की गई।
2. दुर्गापुर इस्पात संयंत्र को लौह अयस्क की प्राप्ति 'नोआमुंडी' और गुआ खान से होती है ।
3. बोकारो इस्पात संयंत्र 1964 में सोवियत संघ के सहयोग से स्थापित किया गया था।
4. विशाखापत्तनम (आंध्र प्रदेश) में स्थित विज्ञाग इस्पात संयंत्र भारत का पहला पत्तन आधारित संयंत्र है।
उपर्युक्त में से कौन - सा/से कथन सही नहीं है/हैं?
(a) केवल 1 
(b) कंवल 1, 2 और 3
(c) केवल 1, 3 और 4
(d) केवल 2, 3 और 4
40. निम्नलिखित कथनों पर विचार कीजिये:
1. बोकारो और राउरकेला इस्पात संयंत्र संयुक्त रूप से राउरकेला प्रदेश से लौह अयस्क प्राप्त करते हैं।
2. सेलम (Salem ) इस्पात संयंत्र तमिलनाडु में 1982 में चालू किया गया।
उपर्युक्त में से कौन-सा/से कथन सही नहीं है/हैं?
(a) केवल 1
(b) कंवल 2
(c) 1 और 2 दोनों
(d) न तो 1 और न ही 2
41. सूती वस्त्र उद्योग के संदर्भ में निम्नलिखित में से कौन-सा / से कथन सही है/हैं?
1. सूती वस्त्र उद्योग की अवस्थिति के निर्धारण में कच्चा "माल अन्य कारकों की अपेक्षा कम महत्त्व रखता है।
2. भारत के तमिलनाडु राज्य में सूत उत्पादन की सर्वाधिक मिलें स्थित हैं।
3. भारत में पहली सफल आधुनिक सूती मिल की स्थापना मुंबई में की गई।
कूट :
(a) केवल 1 और 2
(b) केवल 3
(c) केवल 1 और 3
(d) 1, 2 और 3
42. भारत के किस राज्य में सर्वाधिक चीनी मिलें हैं ?
(a) महाराष्ट्र 
(b) उत्तर प्रदेश
(c) कर्नाटक
(d) आंध्र प्रदेश
43. भारत में औद्योगिक प्रदेश के संबंध में निम्नलिखित पर विचार कीजिये:
1. मुंबई में सूती वस्त्र उद्योग की स्थापना होने का एकमात्र कारण मुंबई में पत्तन का होना है।
2. स्वेज नहर का मुंबई में पचन विकारा व सूती वस्त्र उद्योग के विकास में अत्यधिक योगदान रहा है।
3. पश्चिमी घाट प्रदेश में जलविद्युत शक्ति का विकास सूती वस्त्र उद्योग की आवश्यकताओं की पूर्ति के लिये किया गया।
उपर्युक्त में से कौन-सा /से कथन सही नहीं है / हैं?
(a) केवल 1
(b) कंवल 2 और 3
(c) केवल 3
(d) 1, 2 और 3
हमसे जुड़ें, हमें फॉलो करे ..
  • Telegram ग्रुप ज्वाइन करे – Click Here
  • Facebook पर फॉलो करे – Click Here
  • Facebook ग्रुप ज्वाइन करे – Click Here
  • Google News ज्वाइन करे – Click Here
]]>
Thu, 11 Apr 2024 05:19:08 +0530 Jaankari Rakho
General Competition | Geography | भारत का खनिज संसाधन एवं उर्जा संसाधन https://m.jaankarirakho.com/958 https://m.jaankarirakho.com/958 General Competition | Geography | भारत का खनिज संसाधन एवं उर्जा संसाधन
  • मानवों के लिए ऊर्जा का सबसे प्रमुख स्रोत सौर ऊर्जा है। प्रत्यक्ष या अप्रत्यक्ष तौर पर मानव सौर ऊर्जा का ही प्रयोग करता है।
  • परम्परागत ऊर्जा स्रोत : वैसी ऊर्जा स्रोत जिसका उपयोग मानव प्राचीन काल से परम्परागत ढंग से करता हुआ आ रहा है उसे परम्परागत ऊर्जा स्रोत कहते हैं। जैसे- लकड़ी, कोयला, पेट्रोलियम, गोबर, इत्यादि ।
  • गैर-परम्परागत ऊर्जा स्रोत : नये नवाचारों से खोजे गए ऊर्जा स्रोत का प्रयोग जो मानव के द्वारा किया जाता है उसे गैर-परम्परागत ऊर्जा स्रोत कहते हैं। जैसे-सौर ऊर्जा, पवन ऊर्जा, ज्वारीय ऊर्जा इत्यादि ।
  • सतत उपलब्धता के आधार पर ऊर्जा स्रोत दो प्रकार के होते हैं- 1. नवीकरणीय ऊर्जा स्रोत, 2. अनवीकरणीय ऊर्जा स्रोत ।
    1. नवीकरणीय ऊर्जा स्त्रोत : वैसी ऊर्जा स्रोत जिसका उपयोग दुबारा किया जा सकता है नवीकरणीय ऊर्जा स्रोत कहलाता है। जैसे- सौर ऊर्जा, पवन ऊर्जा, ज्वारीय ऊर्जा इत्यादि ।
    2. अनवीकरणीय ऊर्जा स्रोत : वैसी ऊर्जा स्रोत जिसका प्रयोग दुबारा नहीं किया जा सकता है, अनवीकरणीय ऊर्जा स्रोत कहलाता है। जैसे - कोयला, लकड़ी, पेट्रोलियम इत्यादि ।
  • पवन ऊर्जा : पवन चक्कियों का प्रयोग कर पवन ऊर्जा का उत्पादन किया जाता है। विश्व में पवन ऊर्जा उत्पादन में भारत का चौथा स्थान है वही प्रथम, द्वितीय और तृतीय स्थान पर क्रमशः चीन, अमेरिका और जर्मनी है ।
  • भारत में पवन ऊर्जा का सबसे ज्यादा उत्पादन तमिलनाडु राज्य करता है।
  • राष्ट्रीय पवन ऊर्जा संस्थान चेन्नई में स्थित है।
  • भारत का सबसे बड़ा पवन ऊर्जा केन्द्र तमिलनाडु के कन्या कुमारी में स्थित मप्पंडल पवन ऊर्जा केन्द्र हैं।
  • ज्वारीय ऊर्जा : समुद्रतटीय क्षेत्र में ज्वार और भाटा की उत्पत्ति होती है। ज्वारीय जल से विद्युत का उत्पादन किया जाता है। ज्वारीय ऊर्जा एक नवीकरणीय ऊर्जा है। इससे से प्रदूषण नगण्य होता है। भारत में विशेषकर ज्वारीय ऊर्जा का उत्पादन गुजरात राज्य के कच्छ की खाड़ी और खंभात की खाड़ी वाले इलाकों में होता है। 
  • जल विद्युत ऊर्जा : बहते हुए जल को बाँध बनाकर रोका जाता है और टरबाइन का नामक यंत्र का प्रयोग कर उस जल से विद्युत का उत्पादन किया जाता है जिसे जल विद्युत ऊर्जा कहते हैं। भारत में सबसे ज्यादा जल विद्युत ऊर्जा का उत्पादन कर्नाटक राज्य करता है।
  • भारत में प्रथम जल विद्युत संयंत्र 1897 ई० में दार्जिलिंग में स्थापित किया गया, उसके बाद 1902 ई० में कावेरी नदी पर शिवसमुद्रम में स्थापित किया गया ।
  • भारत विद्युत ऊर्जा केन्द्रों का परिचालन NHPC (National Hydro Electric Power Corporation) नामक संस्था के द्वारा किया जाता है। इसकी स्थापना 1975 ई० में हुई है।
  • प्रमुख NHPC केन्द्र :
NHPC केन्द्र राज्य
सलाल और ऊरी जम्मू-कश्मीर
चमेरा और बाउरा सियल हिमाचल प्रदेश
टनकपुर उत्तराखंड
रंगीत सिक्किम
लोकटक मणिपुर
झझर हरियाणा
दुलहरती और सेवा जम्मू-कश्मीर
धौलीगंगा उत्तराखण्ड
पार्वती हिमाचल प्रदेश
ओंकारेश्वर मध्य प्रदेश
तीस्ता प. बंगाल और सिक्किम
  • सौर ऊर्जा : सूर्य से प्राप्त होने वाली विकिरण ऊर्जा को सौर ऊर्जा कहते हैं। यह एक नवीकरणीय ऊर्जा स्रोत है। इस ऊर्जा से नगण्य प्रदूषण होता है। बारिश के समय इसका उपयोग कर पाना संभव नहीं रह जाता है।
  • विश्व का सबसे बड़ा सोलर पार्क भडला सोलर पार्क है जो राजस्थान के जोधपुर में स्थित है वही विश्व का दूसरा सबसे बड़ा सोलर पार्क शक्ति स्थल है जो कर्नाटक में स्थित है।
  • सौर ऊर्जा का सबसे ज्यादा उत्पादन करने वाला राज्य कनाटक है।
  • उत्तर प्रदेश के झाँसी का रामपुरा गाँव सौर ऊर्जा प्लांट स्थापित करने वाला भारत का पहला गाँव है।
  • सौर तालाब परियोजना गुजरात के भुज में स्थापित किया गया है।
  • भारत और फ्रांस के सहयोग से हरियाणा के ग्वाल पहाड़ी में सौर ऊर्जा अनुसंधान संस्थान स्थापित किया गया।
  • सबसे ज्यादा सौर ऊर्जा का उत्पादन चीन करता है वही भारत का स्थान पाँचवाँ है।
  • जैव ऊर्जा या बायोगैस ऊर्जा : कृषि उत्पाद जैसे धान की भुस्सी, गन्ने की खोई और प्राकृतिक वनस्पति जैसे-जेट्रोफा एवं शहरों के अवशिष्ट पदार्थ से प्राप्त ऊर्जा को जैव ऊर्जा कहते हैं।
  • जेट्रोफा की सहायता से बायोडीजल का निर्माण किया जाता है।
  • भारत का प्रथम बायोडीजल संयंत्र आंध्र प्रदेश के काकीनाड में स्थापित किया गया है।
  • भारत में सबसे ज्यादा जैव ऊर्जा का उत्पादन महाराष्ट्र राज्य करता है।
  • भू-तापीय ऊर्जा केन्द्र:
भू-तापीय ऊर्जा केन्द्र राज्य
पुगा घाटी लद्दाख
मणिकरण और ज्वालामुखी हिमाचल प्रदेश
तपोवन उत्तराखण्ड
सूरजकुंड झारखंड
तातापानी छत्तीसगढ़
तप्तपानी मध्यप्रदेश
पश्चिमी घाट महाराष्ट्र और गुजरात
  • नर्मदा नदी सोन नदी, दामोदर नदी घाटी क्षेत्र में भी भूतापीय ऊर्जा का उत्पादन होता है।
  • भविष्य का ईंधन हाइड्रोजन को कहा जाता है।
  • परमाणु ऊर्जा : परमाणु खनिज जैसे यूरेनियम, बोरेलियम, लिथियम, थोरियम इत्यादि से प्राप्त ऊर्जा को परमाणु ऊर्जा कहते हैं। भारत में परमाणु ऊर्जा का जनक डॉ० होमी जहाँगीर भाभा को माना जाता है। उन्हीं के दिशा-निर्देशन में 1948 ई॰ में परमाणु ऊर्जा विभाग का निर्माण हुआ।
  • भारत का प्रमुख नाभिकीय ऊर्जा केन्द्र : भारत का प्रथम नाभिकीय ऊर्जा केन्द्र महाराष्ट्र में स्थापित तारापुर नाभिकीय ऊर्जा केन्द्र है जिसका गठन अमेरिका की सहायता से किया गया है।
नाभिकीय ऊर्जा केन्द्र राज्य
कुडनकुलम और कलपक्कम तमिलनाडु
कैगा कर्नाटक
जैतापुर और तारापुर महाराष्ट्र
काकरापाडा गुजरात
रावतभाटा राजस्थान
नरौरा उत्तर प्रदेश
  • पेट्रोलियम पदार्थ और प्राकृतिक गैस : भारत में टरसरी युग के अवसादी चट्टानों में प्राकृतिक गैस और खनिज तेल पाया जाता है। अवसादी चट्टानों के ऊपरी भाग में प्राकृतिक गैस, मध्य भाग में खनिज तेल तथा निचले भाग में जल पाया जाता है। खनिज तेल में 90-90% हाइड्रोकार्बन पाये जाते हैं। हाइड्रोकार्बन के 70% भाग में तेल और 30% भाग से प्राकृतिक गैस निकाला जाता है। भारत में खनिज तेल एवं प्राकृतिक गैसों का अन्वेषण (निकालना) ONGC और OIL नामक संस्था के द्वारा किया जाता है। ONGC का गठन 1956 में हुआ है जबकि OIL का गठन 1959 में।
  • भारत में प्रथम तेल परिशोधनशाला का गठन 1901 में असम के डिगबोई में हुआ था।
  • स्वतंत्र भारत का प्रथम तेल परिशोधनशाला 1954 में मुंबई में स्थापित किया गया।
  • भारत में प्रथम तेल कुआँ 1867 में असम के माकुम क्षेत्र में खोजा गया लेकिन सर्वप्रथम तेल कुआँ की खुदाई 1889 ई॰ में असम के डिगबोई के क्षेत्र में हुआ।
भारत के प्रमुख तेल क्षेत्र :
असम :- डिगबोई, सुरमाघाटी, नहारकाटिया
अरूणाचल प्रदेश :- खारसंग, निग्रू क्षेत्र
नागालैंड :- बोरहोल्ला क्षेत्र
गुजरात :- अंकलेश्वर, खंभात, कलोल, मेहसाना, कोशाम्बा, सानंद, आलियाबेट, जुनेज
महाराष्ट्र :- मुंबई हाई तट, बेसिन, पन्नामुक्ता क्षेत्र
राजस्थान :- बीकानेर, जैसलमेर, बारमेर
उपर्युक्त तेल क्षेत्र के अलावा भारत के ब्रह्मपुत्र नदी घाटी, कृष्णा नदी घाटी, गोदावरी नदी घाटी, कावेरी नदी घाटी बेसिन से भी खनिज तेल का उत्पादन किया जाता है।
  • भारत में प्राकृतिक गैस का सर्वाधिक भंडारण, पूर्वी अपतट तथा सर्वाधिक उत्पादन असम राज्य के द्वारा किया जाता है।
  • भारत में सर्वाधिक पेट्रोलियम का भंडारण असम राज्य में तथा सर्वाधिक उत्पादन राजस्थान राज्य के द्वारा किया जाता है।

भारत के प्रमुख तेल परिशोधनशाला :

परिशोधनशाला राज्य
जामनगर और कोयली गुजरात
मथुरा उत्तरप्रदेश
बरौनी बिहार
मंगलुरू कर्नाटक
भटिंडा पंजाब
नुमालीगढ़ असम
तातीपाका आंध्र प्रदेश 
वीणा मध्य प्रदेश
वादिनर गुजरात
  • भारत में पेट्रोलियम पदार्थ का उत्पादन होता है लेकिन भारत की जितनी आवश्यकता है उतना उत्पादन नहीं होता है इसलिए भारत पेट्रोलियम पदार्थ को लेकर आयात पर निर्भर करता है।
  • भारत में विद्युत उत्पादन के लिए NTPC (National Thermal Power Corporation) को जाना जाता है। इसका गठन 1975 में हुआ है! 
  • कोयला (Coal) : इससे ऊष्मा एवं प्रकाश की प्राप्ति होती है। NTPC में कोयला का उपयोग कर विद्युत ऊर्जा का उत्पादन किया जाता कोयला को काला सोना, उद्योग की रोटी, उद्योगों की जननी कहा जाता है।
  • कोयला मुख्यत: चार प्रकार का होता है। जिस कोयता में कार्बन की मात्र अधिक होती है उत्तम कोटि का माना जाता है। 
    1. पिट कोयला : यह प्राथमिक कोयता माना जाता है। इसमें नमी की मात्रा अधिक होती है। इसमें कार्बन  40% से भी कम पाया जाता है।
    2. लिग्नाइट कोयला : इस कोयता में कार्बन की मात्रा 400-500 है। इसे पूरा कोणता भी कहा जाता है। यह कोयला मुख्यतः तमिलनाडु के नेवली क्षेत्र, राजस्थान का परचम, कील और गुजरात में पाया जाता है।
    3. विटुमिनस कोयता: भारत में सबसे अधिक विद्युमन कीमला पाया जाता है। इस कोयला में SAV-MAY कार्यन होता है। यह कोयला गोडवाना क्रम के चट्टान में पाया जाता है।
    4. एन्थ्रासाइट कोयला : यह सबसे उत्तम कोटि का कोयला कीता है। उप AY 95% कार्बन पाया जाता है। वह कोयला मुख्यत: जम्मू कश्मीर के रियासी क्षेत्र में मुख्यत: पाया जाता है।
  • भारत में निम्न नही आरियों में कोयला पाया जाता है-
    1. दामोदर नदी घाटी
    2. महानदी घाटी
    3. गोदावरी नहीं भाटी
    4. सोन नदी घाटी
  • सबसे अधिक कोयला दामोदर नदी घाटी में पाया जाता है।
  • झारखंड का झरिया सबसे अधिक कोयला खनन के लिए प्रसिद्ध है।
  • कोयला भंडारण के दृष्टिकोण से तीन बड़े राज्य क्रमश: निम्न है - 1. झारखंड, 2, ओडिशा, 3. छत्तीसगढ़।
  • कोयला उत्पादन के दृष्टिकोण से तीन बड़े राज्य क्रमश: निम्न है- 1. छत्तीसगढ़, 2. ओडिशा, 3. झारखंड।
कोयला पर आधारित NTPC केन्द्र :
NTPC केन्द्र राज्य
बाढ़ बिहार (पटना)
कहलगाँव भागलपुर (बिहार)
बदरपुर दिल्ली
फरक्का पश्चिम बंगाल
कोरबा छतीसगढ़
रामागुंडम तेलंगाना
रिहंद उत्तर प्रदेश
दादरी और टांडा उता प्रदेश
सिमहाद्रि आंध्र प्रदेश
खड़गाँव मध्य प्रदेश
बोगाई गाँव असम
तलचर ओडिशा
दलीपार्ली ओडिशा
सिपत छत्तीसगढ़
मौदा महाराष्ट्र
बरौनी बिहार
  • खनिज संसाधन : प्रकृति में पाये जाने वाले वैसे पदार्थ जिसमें अपना रासायनिक और भौतिक गुण मौजूद हो तथा जिसे उत्खनन प्रक्रिया द्वारा निकाला जाता हो खनिज कहलाता है। खनिज को मुख्यतः दो भागों में बाँटा गया है- 1. धात्विक खनिज 2. अधात्विक खनिज
  • धात्विक खनिज : धात्विक खनिज ऊष्णा और विद्युत का सुचालक होता है। इसके प्रमुख उदाहरण निम्न हैं जैसे- लौह अयस्क, मैंगनीज, ताँबा, टंगस्टन इत्यादि । 
  • अधात्विक खनिज : अधात्विक खनिज विद्युत और ऊष्मा का कुचालक होता है। इसके प्रमुख उदाहरण निम्न है, जैसे-चूना पत्थर, संगमरमर इत्यादि ।
  • Note : भारत में सबसे अधिक खनिज संसाधन प्रायद्वीपीय पठार में पायी जाती है।
  • भारत में पाये जाने वाले खनिज संसाधनों को चार पेटी में बाँटा गया है-
    1. उत्तरी-पूर्वी पेटी
    2. उत्तरी-पश्चिमी पेटी या अरावली पेटी
    3. मध्यवर्ती पेटी
    4. दक्षिणी-पश्चिमी पेटी
  • Note : 1. भारत में सबसे अधिक खनिज संसाधन उत्तरी-पूर्वी पेटी में पाया जाता है।
    2. छोटानागपुर के पठार को भारत का रुर प्रदेश कहा जाता है। क्योंकि यह क्षेत्र खनिज संसाधन से समृद्ध है।
    3. धारवाड़ क्रम का चट्टान धात्विक खनिज के लिए प्रसिद्ध है। 

वस्तुनिष्ठ प्रश्न एवं उत्तर

1. इनमें सबसे अधिक कठोर खनिज कौन है ?
(a) लोहा 
(b) मँगनीज
(c) हीरा
(d) सोना
2. पृथ्वी पर किस खनिज की प्रधानता है ? 
(a) धात्विक
(b) अधात्विक 
(c) कार्बनिक
(d) इनमें सभी
3. भारत का कौन- -सा क्षेत्र खनिजों में सबसे अधिक धनी है ?
(a) पश्चिमोत्तर क्षेत्र
(b) दक्षिणी क्षेत्र क्षेत्र
(c) महाराष्ट्र
(d) पूर्वोत्तर क्षेत्र
4. भारत में लगभग कितने लोग खनन कार्य में लगे हुए हैं ?
(a) लगभग 10 लाख
(b) लगभग 12 लाख
(c) लगभग 16 लाख
(d) लगभग 20 लाख
5. किस किस्म के लोहे में चुंबकीय गुण मिलता है ?
(a) हेमाटाइट
(b) सिडेराइट
(c) लाइमोनाइट
(d) मैग्नेटाइट 
6. कालाहांडी खनिज क्षेत्र किस राज्य में विद्यमान है ?
(a) झारखंड में 
(b) ओडिशा में 
(c) महाराष्ट्र में
(d) छत्तीसगढ़ में 
7. इनमें कौन अधात्विक खनिज है ?
(a) कोयला 
(b) लोहा
(c) तांबा
(d) बॉक्साइट
8. इनमें कौन अलौह खनिज है ?
(a) ताँबा 
(b) जस्ता
(c) सीसा
(d) इनमें सभी
9. इनमें कौन लौह क्षेत्र भारत के पश्चिमी भाग में है ?
(a) बस्तर 
(b) ओडिशा
(c) गोवा
(d) बेल्लारी
10. इनमें कौन कथन सही है ?
(a) विश्व में मैंगनीज का सबसे बड़ा भंडार भारत में है ।
(b) विश्व में मैंगनीज का पाँचवाँ बड़ा भंडार भारत में है।
(c) भारत में मैंगनीज का भंडार बहुत ही कम है।
(d) भारत में मैंगनीज उच्च कोटि का नहीं होता है।
11. किस क्षेत्र का अवरक सर्वश्रेष्ठ माना जाता है ?
(a) राजस्थान का 
(b) आंध्र प्रदेश का
(c) झारखंड का
(d) इनमें सभी का
12. घाटशिला का ताँबा क्षेत्र किस राज्य में हैं ?
(a) झारखंड में
(b) राजस्थान में
(c) कर्नाटक में
(d) आंध्र प्रदेश में 
13. रानीगंज का कोयला क्षेत्र किस राज्य में है ? 
(a) झारखंड में 
(b) पश्चिम बंगाल में
(c) बिहार में
(d) मध्य प्रदेश में
14. बगरू पहाड़ी क्षेत्र से कौन-सा महत्वपूर्ण खनिज प्राप्त होता है ?
(a) कायला
(b) ताँबा
(c) लोहा
(d) वॉक्साइड
15. गुजरात का कैरा क्षेत्र किस खनिज उत्पादन के लिए प्रसिद्ध है ?
(a) अवरक 
(b) बॉक्साइट
(c) मैंगनीज
(d) लोहा
16. निम्नांकित किस खनिज में भारत सुसंपन्न है ?
(a) ताँबा
(b) लोहा
(c) सोना
(d) चाँदी
17. इनमें कौन लौह अयस्क नहीं है ?
(a) हेमाटाइट
(b) मैग्नेटाइट
(c) ऐथ्रासाइट
(d) लाइमोनाइट
18. इनमें कौन लोहा का निर्यात नहीं करता है ?
(a) मुंबई
(b) कोलकाता
(c) पारादीप
(d) विशाखापट्नम
19. इनमें कौन मैगनीज का महत्वपूर्ण उत्पादक नहीं है ?
(a) ओडिशा 
(b) कर्नाटक
(c) तमिलनाडु
(d) महाराष्ट्र
20. झारखंड का घाटशिला किस खनिज उत्पपादन के लिए प्रसिद्ध है ?
(a) ताँबा
(b) बॉक्साइट
(C) अवरक 
(d) लौह अयस्क
21. किस प्रकार की चट्टानों में खनिजों का जमाव परतों में मिलता है ?
(a) आग्नेय
(b) अवसादी
(c) रूपांतरित
(d) इनमें किसी में नहीं
22. इनमें कौन परंपरागत ऊर्जा स्त्रोत नहीं है ?
(a) कोयला
(b) पवन
(c) पेट्रोलियम
(d) परमाणु खनिज
23. इनमें कौन आणविक शक्ति केंद्र है ?
(a) कोयली
(b) चंद्रपुरा
(c) तिलैया
(d) नरोरा
24. कलपक्कम किस राज्य में स्थित है ?
(a) उत्तर प्रदेश में
(b) महाराष्ट्र में
(c) राजस्थान में
(d) तमिलनाडु में
25. डिगबोई तेल क्षेत्र किस राज्य में स्थित है ?
(a) असम में
(b) गुजरात में
(c) केरल में
(d) आंध्र प्रदेश में
26. बॉम्बे हाई क्या है ?
(a) एक ऊँची सड़क
(b) एक हवाई अड्डा
(c) खनिज तेल - उत्पादक क्षेत्र
(d) औद्योगिक केंद्र
27. इनमें कौन गुजरात का तेल उत्पादन केंद्र है ?
(a) मोरन 
(b) मथुरा
(c) नहरकटिया
(d) अंकलेश्वर
28. भारत में जलविद्युत का विकास सर्वप्रथम किस क्षेत्र में हुआ ?
(a) पूर्वी भारत में
(b) दक्षिणी भारत में
(c) उत्तर-पश्चिमी भारत में
(d) मध्य भारत में
29. खनिज ऊर्जा संसाधन के संरक्षण की कौन विधि उपयुक्त है ?
(a) नए खनिज क्षेत्रों की खोज करते रहना
(b) बर्बादी से बचने के लिए खनिजों की खुदाई में सावधानी बरतना
(c) विरल या कम प्राप्य खनिजों के विकल्पों की खोज
(d) इनमें सभी
30. तारापुर परमाणु केंद्र कहाँ स्थित है ?
(a) दिल्ली के निकट
(b) मुंबई के उत्तर
(c) चेन्नई के निकट
(d) गोरखपुर के निकट
31. पायकारा जलविद्युत उत्पादन केंद्र किस राज्य में है ?
(a) तमिलनाडु में 
(b) आंध्र प्रदेश में
(c) कर्नाटक में
(d) केरल में
32. यूरेनियम और थोरियम का क्या महत्त्व है ?
(a) इनसे खाद तैयार होती है
(b) ये परमाणु खनिज है
(c) ये महत्वपूर्ण वन वृक्ष है
(d) ये पुराने पहाड़ हैं
33. सौर ऊर्जा का सबसे बड़ा संयंत्र कहाँ लगाया गया है ?
(a) पंजाब में 
(b) राजस्थान में
(c) गुजरात में
(d) उत्तर प्रदेश में
34. भारत में विद्युत की सबसे अधिक खपत किसमें होती है ?
(a) घरेलू कामों में
(b) व्यापारिक कामों में
(c) उद्योगों में
(d) कृषि कार्यों में
35. भारत में खनिज तेल का उत्पादन 2010 में कितना रहा ?
(a) 72 लाख टन
(b) 7 करोड़ टन
(c) 3 करोड़ 35 लाख टन
(d) 9 करोड़ टन
36. पेट्रोलियम किन चट्टानों में मिलता है ?
(a) आग्नेय 
(b) परतदार
(C) रूपात
(d) इनमें सभी
37. इनमें कहाँ प्राकृतिक गैस भंडार मिले हैं ?
(a) छत्तीसगढ़ 
(b) कर्नाटक
(c) त्रिपुरा
(d) मध्य प्रदेश
38. भारत में ऊर्जा का प्रमुख स्त्रोत कौन-सा है ?
(a) कोयला 
(b) पेट्रोलियम
(c) प्राकृतिक गैस
(d) जलविद्युत
39. इनमें कौन खनिज केरल तट पर बालू के रूप में मिलता है ?
(a) यूरेनियम
(b) कोयला 
(c) थोरियम
(d) सोना
हमसे जुड़ें, हमें फॉलो करे ..
  • Telegram ग्रुप ज्वाइन करे – Click Here
  • Facebook पर फॉलो करे – Click Here
  • Facebook ग्रुप ज्वाइन करे – Click Here
  • Google News ज्वाइन करे – Click Here
]]>
Wed, 10 Apr 2024 11:03:29 +0530 Jaankari Rakho
General Competition | Geography | भारत की कृषि एवं पशुपालन https://m.jaankarirakho.com/957 https://m.jaankarirakho.com/957 General Competition | Geography | भारत की कृषि एवं पशुपालन
  • कृषि राज्य सूची का विषय है।
  • अंतरिम मंत्रिमंडल और स्वतंत्र भारत के प्रथम मंत्रिमंडल में देश के खाद्य एवं कृषि मंत्री राजेन्द्र प्रसाद बने थे ।
  • विश्व का प्रमुख खाद्य फसल गेहूँ है । वही भारत का प्रमुख खाद्य फसल चावल है।
  • विश्व के वैसे क्षेत्र जहाँ चावल का उत्पादन ज्यादा होता है सामान्यतः वहाँ की जनसंख्या अधिक होती है।
  • हरित क्रांति : शब्द का प्रयोग विलियम गॉट। इसका संबंध खाद्यान्न उत्पादन से है। विश्व में हरित क्रांति लाने का श्रेय नॉर्मन बोरलॉग को माना जाता है । वही भारत में हरित क्रांति का जनक एम० एस० स्वामीनाथन को माना जाता है। भारत में हरित क्रांति 1967-68 में आयी है इसके बाद ही भारत खाद्यान्न के उत्पादन में आत्मनिर्भर बन पाया।
  • हरित क्रांति का सर्वाधिक प्रभाव गेहूँ के उत्पादन पर पड़ा है। गेहूँ के बाद चावल के उत्पादन पर वही हरित क्रांति आने से मोटे अनाज के उत्पादन में कमी हुई है।
  • 1960 ई॰ में उत्तराखण्ड के पंतनगर में भारत के प्रथम प्रधानमंत्री पंडित जवाहर लाल नेहरू ने गोविंद वल्लभ पंत कृषि विश्वविद्यालय की स्थापना की। इस विश्वविद्यालय में हरित क्रांति को सफल बनाने में अपना महत्त्वपूर्ण योगदान दिया।
  • श्वेत क्रांति : इसका संबंध दुग्ध उत्पादन से है। इस क्रांति को लाने का श्रेय डॉ. वर्गीज कुरियन को जाता है। इस क्रांति को सफल बनाने हेतु 1970 ई० में ऑपरेशन फ्लड कार्यक्रम की शुरूआत की गई है। यह कार्यक्रम राष्ट्रीय डेयरी विकास बोर्ड के द्वारा चलाया गया है। श्वेत क्रांति को सफल बनाने में सबसे अहम योगदान AMUL (Anand Milk Union Limited) का है।
  • श्वेत क्रांति से सबसे ज्यादा लाभांवित क्षेत्र पश्चिमी भारत है। खास तौर पर गुजरात सबसे ज्यादा ला नांत्रित हुआ है वही हरित क्रांति से सबसे अधिक लाभांवित क्षेत्र उत्तरी भारत रहा है वही राज्यों के दृष्टिकोण से देखें तो पंजाब में सबसे अधिक लाभांवित रहा है।
  • नीली क्रांति : इसका संबंध उत्पादन से है। इस क्रांति से सबसे धक लाभांवित क्षेत्र दक्षिण भारत है वही राज्यों के दृष्टिकोण से आंध्र प्रदेश है।
  • Note : आंध्र प्रदेश के नेल्लौर जिला को झींगा मछली का राजधानी नगर कहा जाता है।
क्रांति संबंध / क्षेत्र
लाल क्रांति टमाटर और मांस उत्पाद
गोल क्रांति आलू उत्पादन
मूक क्रांति मोटे अनाज
परामणि क्रांति भिंडी उत्पादन
पीली क्रांति तिलहन उत्पादन
काली क्रांति / कृष्ण क्रांति पेट्रोलियम उत्पादन / बायो डीजल उत्पादन
सेफ्रॉन क्रांति केसर उत्पादन
रजत क्रांति अंडा उत्पादन
अमृत क्रांति नदी जोड़ो परियोजना
सुनहरी / स्वर्ण क्रांति फल एवं सब्जी उत्पादन / बागवानी कृषि
सुनहरा रेशा क्रांति जूट उत्पादन
गुलाबी क्रांति झींगा मछली / प्याज / औषधि उत्पादन
भूरी क्रांति उर्वरक उत्पादन
  • ऋतु के आधार पर भारतीय फसल को तीन वर्गों में बाँटा गया है-
    1. रबी फसल : वैसी फसल जिसकी बुआई अक्टूबर-नवम्बर में हो तथा कटाई मार्च-अप्रैल में हो रबी फसल कहलाता है। यह शीतकालीन फसल है। पश्चिमी विक्षोभ इस फसल के लिए लाभदायक माना जाता है। जैसे-गेहूँ, चना, मटर, सरसों, आलू इत्यादि।
    2. खरीफ फसल : वैसी फसल जिसकी बुआई जून-जुलाई और कटाई अक्टूबर-नवम्बर में हों, खरीफ फसल कहलाता है। यह ग्रीष्म तथा वर्षाकालीन फसल है। इस फसल हेतु दक्षिण-पश्चिम मॉनसून लाभदायक होता है। इसके प्रमुख उदाहरण–धान, ज्वार, बाजरा, कपास, मक्का, सोयाबीन, मूँगफली, अरहर इत्यादि ।
    3. जायद फसल : वैसी फसल जिसकी बुआई अप्रैल-मई में तथा कटाई मई-जून में हो जायद फसल कहलाता है। जैसे- खीरा, ककड़ी, तरबूज, खरबूज इत्यादि ।
  • भारत में जैविक कृषि को बढ़ावा देने हेतु 10वीं पंचवर्षीय योजना के दौरान अक्टूबर, 2004 में राष्ट्रीय जैविक खेती परियोजना की शुरूआत की गई।
  • जैविक कृषि पद्धति के तहत शत-प्रतिशत खेती करने वाला भारत का प्रथम राज्य सिक्किम (2016) बना।
  • वैश्विक स्तर पर जैविक खेती को अपनाने वाला प्रथम देश क्यूबा बना ।
  • जैविक कृषि का जनक खेती अलबर्ट हावर्ड को माना जाता है।
  • बागानी कृषि पद्धति : बागान या बगीचा लगाकर फल-फूल सब्जी जैसे खाद्य पदार्थों का जो उत्पादन किया जाता है, उसे ही बागानी कृषि कहते हैं। बागानी कृषि का संबंध हॉर्टीकल्चर से है। ट्रक फार्मिंग का प्रयोग बागानी कृषि हेतु किया जाता है।
  • गहन कृषि : जीविकोपार्जन हेतु की गई कृषि को गहन कृषि कहा जाता है।
  • शस्य गहनता : किसी एक भूमि पर एक फसल वर्ष में फसल उत्पादन की बारंबारता शस्य गहनता कहलाती है। भारत में सबसे अधिक शस्य गहनता पंजाब राज्य का है वही दूसरे और तीसरे स्थान पर क्रमशः हरियाणा और पश्चिम बंगाल है।
  • मिश्रित कृषि : जब किसान फसल उत्पादन के साथ-साथ पशुपालन का भी कार्य करता है, उसे ही मिश्रित कृषि कहा जाता है।
  • युग्म पैदावार कृषि : एक साथ एक ही बार में एक ही खेत में एक साथ एक ही बार में दो फसलों का उत्पादन करना युग्म पैदावार कृषि कहलाता है। जैसे- गेहूँ और सरसो, आलू और मूली इत्यादि।
  • Note : भारतीय कृषि मॉनसून पर निर्भर होता है। अगर मॉनसून ठीक-ठाक रहता है तो कृषि में उत्पादन अच्छा होता है और अगर मॉनसून ठीक नहीं होता है तो कृषि उत्पादन अच्छा नहीं होता है इसलिए भारतीय कृषि को मॉनसून की जुआ कहा जाता है।
  • विस्तृत कृषि : यह कृषि मुख्यत: विकसित देशों में होती है। इस कृषि में कृषि कार्य आधुनिक तकनीकी मशीन से किया जाता है। इस कृषि में मानव श्रम का महत्व बहुत कम होता है। भारत में हरित क्रांति आने के बाद पंजाब और हरियाणा के क्षेत्र में इस प्रकार की कृषि की जा रही है।
  • हवा में पौधा उगाना एयरोपानिक्स कहलाता है।
  • जल में पौधा उगाना हाइडोपोनिक्स कहलाता है।
  • बागवानी उत्पादन हॉर्टीकल्चर कहलाता है।
  • रेशम कीट पालन सेरीकल्चर कहलाता है।
  • मधुमक्खी पालन एपीकल्चर कहलाता है।
  • अंगूर उत्पादन विटीकल्चर कहलाता है।
  • फूलों का उत्पादन फ्लोरीकल्चर कहलाता है।
  • केंचुआ पालन वर्मीकल्चर कहलाता है। 
  • मत्स्य पालन पीसीकल्चर कहलाता है।
  • सब्जियों का उत्पादन ओलेरीकल्चर कहलाता है।
  • फलों का उत्पादन पोमीकल्चर कहलाता है।
भारत के प्रमुख खाद्य फसल:
  • चावल : इसका वैज्ञानिक नाम Oryza Sativa है। यह एक उष्ण फसल है। इस फसल के उत्पादन हेतु 20°C-30°C तापमान तथा 100 cm-150.cm वर्षा की आवश्यकता होती है। इस फसल के उत्पादन हेतु सबसे उपयुक्त जलोढ़ मिट्टी मानी जाती है।
  • भारत के सर्वाधिक भू-भाग पर चावल की खेती की जाती है।
  • भारत के भीतर दो राज्य 1. पश्चिम बंगाल, 2. तमिलनाडु में सालों भर चावल की खेती की जाती है। ओस (शरदकालीन) अमन (शीतकालीन) बोरो (ग्रीष्मकालीन) मुख्यतः पश्चिम बंगाल में उगायी जाने वाली चावल की किस्म है।
  • धान में होने वाला प्रमुख रोग खैरा है।
  • दिश्व चावल अनुसंधान केन्द्र फिलीपींस की राजधानी मनीला में है वही राष्ट्रीय चावल अनुसंधान केन्द्र कटक (ओडिशा) में है।
  • Golden rice में विटामिन ए प्रचुर मात्रा में पाया जाता है।
  • पॉलिश किए हुए चावल में विटामिन B की कमी होती है।
  • चावल को अधिक धोने से विटामिन समाप्त हो जाती है। 
  • विश्व में चावल का सबसे ज्यादा उत्पादन चीन करता हैं तो वही भारत में चावल का सबसे ज्यादा उत्पादन पश्चिम बंगाल राज्य करता है। (जापान प्रति हेक्टेयर)
  • गेहूँ : इसका वैज्ञानिक नाम Triticum sativum है। यह एक शीतोष्ण कटिबंधीय तथा रबी फसल है। गेहूँ के उत्पादन हेतु तापमान 10°C-25°C होना चाहिए। गेहूँ की बुआई के समय तापमान 10°C - 15°C वृद्धि होने के समय तापमान 15°C 20°C तथा कटाई कं समय तापमान 20°C - 25°C होनी चाहिए। गेहूँ हेतु वर्षा 50 cm-75 cm होनी चाहिए।
  • गेहूँ के लिए दोमट एवं चारनोजम मिट्टी उपयुक्त होती है।
  • कर्नालत्रंट रोग और रस्ट रोग गेहूँ में होने वाला रोग हैं।
  • सोनालिका, कल्याण सोना, Raj 3077, अर्जुन इत्यादि गेहूँ की किस्में है ।
  • गेहूँ में 65-70% कार्बोहाइड्रेट पाया जाता है तथा 8-10% प्रोटीन पाया जाता है।
  • Note : एक आम मानव को सबसे ज्यादा कार्बोहाइड्रेट की प्राप्ति चावल से होती है।
  • विश्व में सबसे ज्यादा गेहूँ का उत्पादन चीन करता है वही भारत का स्थान इसमें दूसरा है।
  • भारत में सबसे ज्यादा गेहूँ का उत्पादन उत्तर प्रदेश राज्य करता है तथा प्रति हेक्टेयर के दृष्टिकोण से सबसे ज्यादा उत्पादन पंजाब राज्य करता है | (USA प्रति हेक्टेयर)
  • विश्व में गेहूँ की सबसे बड़ी मंडी कनाडा के विनिपेग शहर में स्थित है।
  • यूक्रेन को रोटी की डलिया कहा जाता है।
  • प्रेयरी घास के मैदान को उत्तरी अमेरिका महादेश में तथा पम्पास घास के मैदान को दक्षिणी अमेरिका महादेश में रोटी की डलिया कहा जाता है।
  • चाय : यह एक पेय पदार्थ है। इसका जन्मदाता चीन को माना जाता है। चाय की खेती के लिए सस्ता महिला श्रमिकों का होना आवश्यक है। भारत में चाय का बागान सर्वप्रथम अंग्रेजों के द्वारा 1841 ई० में पश्चिम बंगाल के दार्जिलिंग पहाड़ी में लगाया गया था।
  • चाय की खेती के लिए 25- 30°C तापमान, लैटेराइट मिट्टी, 150-200 cm वर्षा, ढालनुमा भूमि होनी चाहिए ।
  • चाय में थीन, कॉफी में कैफीन और तंबाकू में निकोटिन पाया जाता है।
  • विश्व में सबसे ज्यादा चाय का उत्पादन चीन करता है वही भारत में सबसे ज्यादा चाय का उत्पादन असम राज्य करता है।
  • भारतीय चाय बोर्ड कोलकाता में स्थित है। 
  • हमलोग चाय के पत्ती का उपयोग करते हैं तथा कॉफी के बीज का उपयोग करते हैं।
  • कॉफी / कहवा : यह भी एक पेय पदार्थ है। इसका जन्मदाता देश इथियोपिया को माना जाता है। यह सर्वप्रथम कर्नाटक के बाबाबूदान की पहाड़ी में लगाया गया था । विश्व में सबसे ज्यादा कॉफी का उत्पादन ब्राजील करता है तो वही भारत में सबसे ज्यादा कॉफी का उत्पादन कर्नाटक राज्य करता है। 
  • कॉफी उत्पादन के लिए तापमान 20-30°C, 150-200 cm वर्षा, भूमि ढालुनुमा और लैटेराइट या दोमट मिट्टी होनी चाहिए।
  • भारत में दो प्रकार की कॉफी अरेबिका और रोबास्टा का उत्पादन करता है, जिसमें भारत सबसे ज्यादा अरेबिका किस्म के कॉफी का उत्पादन करता है ।
  • Note : तमिलनाडु में स्थित नीलगिरि की पहाड़ी में चाय और कॉफी दोनों का उत्पादन किया जाता है।
  • भारतीय चाय कोलकाता बंदरगाह से सबसे ज्यादा ब्रिटेन को निर्यात किया जाता है।
  • नगदी फसल : वैसी फसल जिसको बेचने से हमें नगद रुपया की प्राप्ति गोटी है, नगदी फसल कहलाता है। जैसे - तंबाकू, गन्ना, चाय, कॉफी, मक्का, जूट etc.
  • तंबाकू : यह एक नशीला पदार्थ है इसे भारत लाने का श्रेय पुर्तगालियों को जाता है। तंबाकू के उत्पादन में विश्व में प्रथम स्थान चीन को प्राप्त है वही भारत का स्थान दूसरा है। भारत में सबसे ज्यादा तंबाकू आंध्र प्रदेश के द्वारा उगाया जाता है।
  • तंबाकू के उत्पादन हेतु न्यूनतम तापमान और कम वर्षा की आवश्यकता होती है। तंबाकू जलोढ़ तथा काली मिट्टी में उगाया जाता हैं।
  • तंबाकू के तने का प्रयोग पोटाश उर्वरक / औषधि बनाने में किया जाता है।
  • जूट : यह एक अखाद्य खरीफ फसल है। इसे भारत का स्वर्णिम तंतु (गोल्डन फाइबर ऑफ इंडिया) कहा जाता है। विश्व में सबसे ज्यादा जूट का उत्पादन बांग्लादेश करता है वही भारत में सबसे ज्यादा उत्पादन प० बंगाल राज्य करता है।
  • रेशम : विश्व में सबसे ज्यादा रेशम का उत्पादन चीन करता है वही रेशम उत्पादन में भारत का स्थान दूसरा है। विश्व का एकमात्र देश भारत है जहाँ पाँच किस्म के रेशम का उत्पादन होता है जो निम्न हैं- 1. मलबरी, 2. मूंगा, 3. इरी, 4. ट्रॉपिकल टसर, 5. ओक टसर ।
  • विश्व का एकमात्र देश भारत है जो मूँगा रेशम का उत्पादन करता है। भारत में हरित क्रांति लाने का श्रेय भारत के प्रथम प्रधानमंत्री पंडित जवाहर लाल नेहरू को जाता है वही भारत में हरित क्रांति इंदिरा गाँधी के शासनकाल में शुरू हुआ था।

वस्तुनिष्ठ प्रश्न एवं उत्तर

1. भारत की अर्थव्यवस्था का मुख्य आधार क्या है ?
(a) कृषि
(b) उद्योग
(c) खनिज उत्पादन
(d) अंतराष्ट्रीय व्यापार
2. ग्रीष्मकालीन पर्याप्त वर्षा किस फसल की खेती के लिए उपयुक्त है ?
(a) चना
(b) धान
(c) गेहूँ
(d) कपास
3. पंजाब किस फसल का सर्वप्रमुख उत्पादक है ?
(a) बाजरा का 
(b) धान का
(c) चाय का
(d) गेहूँ का 
4. इनमें कौन राज्य चाय का प्रमुख उत्पादक है ?
(a) झारखंड 
(b) महाराष्ट्र
(c) असम
(d) पश्चिम बंगाल
5. कहवा का सबसे अधिक उत्पादन किस राज्य में होता है?
(a) असम में 
(b) कर्नाटक में
(c) केरल में
(d) तमिलनाडु में
6. महाराष्ट्र किस फसल की खेती के लिए प्रसिद्ध है ?
(a) धान 
(b) कपास 
(c) कहवा 
(d) जूट
7. उत्तर- भारत का कौन- -सा राज्य गन्ना उत्पादन सर्वाधिक प्रसिद्ध है ?
(a) उत्तर प्रदेश
(b) पंजाब
(c) पश्चिम बंगाल
(d) हरियाणा
8. मूँगफली का सबसे बड़ा उत्पादक राज्य कौन है ?
(a) बिहार 
(b) महाराष्ट्र
(c) गुजरात
(d) कर्नाटक
9. रबर की खेती भारत के किस राज्य में की जाती है ?
(a) केरल 
(b) गुजरात में
(c) आंध्र प्रदेश में
(d) महाराष्ट्र
10. दक्षिण भारत में सबसे अधिक गन्ना उत्पन्न करनेवाला राज्य कौन है ?
(a ) झारखंड
(b) कर्नाटक
(c) आंध्र प्रदेश
(d) महाराष्ट्र
11. मसाले की खेती के लिए कौन-सा राज्य भारत - प्रसिद्ध है ?
(a) महाराष्ट्र
(b) केरल
(c) तमिलनाडु
(d) असम
12. भारत का कौन-सा राज्य नारियल प्रदेश के नाम से जाना जाता है ?
(a) तमिलनाडु
(b) पश्चिम बंगाल
(c) केरल
(d) आंध्र प्रदेश
13. बढ़िया नस्ल के साँड़ और भैंस के लिए भारत का कौन- -सा राज्य प्रसिद्ध है ?
(a) असम
(b) बिहार
(c) पंजाब और हरियाणा
(d) कर्नाटक और तमिलनाडु
14. ऑपरेशन फ्लड से क्या तात्पर्य है ?
(a) बाढ़ को रोकने के लिए बाँध बनाना
(b) दूध बढ़ाने के लिए क्रांतिकारी कदम उठाना
(c) खाद्यान बढ़ाने के लिए हरित क्रांति लाना
(d) मत्स्य उत्पादन बढ़ाने की योजना बढ़ाना
15. झूम खेती कहाँ की जाती है ?
(a) पूर्वी पहाड़ी राज्यों में
(b) पश्चिमी पठारी राज्यों में
(c) गंगा के मैदान में 
(d) पंजाब और हरियाणा में
16. इनमें कौन रोपण कृषि की फसल है ?
(a ) धानं 
(b) चाय
(c) गेहूँ
(d) जूट
17. इनमें कौन गेहूँ का प्रमुख उत्पादक नहीं है ?
(a) उत्तर प्रदेश 
(b) मध्य प्रदेश
(c) तमिलनाडु
(d) राजस्थान
18. किस राज्य में धान के सिंचित क्षेत्र अधिक मिलते हैं ?
(a) पश्चिम बंगाल में
(b) तमिलनाडु में
(c) आंध्र प्रदेश में 
(d) उत्तर प्रदेश में
19. भारत में ज्वार की सबसे अधिक खेती कहाँ होती है ?
(a) महाराष्ट्र में 
(b) गुजरात मं
(c) उत्तर प्रदेश में
(d) तमिलनाडु मं
20. अधिक दिनों तक वर्षा का वियोग सहना किस फसल के लिए घातक होता है ?
(a) धान 
(b) गेहूँ
(c) चाय
(d) कपास
21. किस फसल के खेती के लिए ढालू भूमि आवश्यक है ?
(a) गेहूँ 
(b) कपास
(c) कहवा
(d) गन्ना
22. कौन-सा राज्य भारत का जूट क्षेत्र कहलाता है ?
(a) असम 
(b) पश्चिम बंगाल
(c) ओडिशा
(d) तमिलनाडु
23. दक्षिण भारत में सर्वाधिक गन्ना उत्पादक राज्य कौन है?
(a) आंध्र प्रदेश 
(b) ओडिशा
(c) महाराष्ट्र 
(d) तमिलनाडु
24. ऑपरेशन फ्लड से क्या तात्पर्य है ?
(a) बाढ़ - नियंत्रण
(b) डेयरी विकास
(c) बाँध मरम्मर
(d) धान की उन्नत खेती
25. हरित क्रांति का दूसरा चरण कब स्थापित हुआ ?
(a) 1983-84 में 
(b) 1951-52 में 
(c) 2000-01 में
(d) 1914-15 में
26. दुग्ध-उत्पादन से संबंधित ऑपरेशन फ्लड 2 नामक योजना कब प्रारंभ की गई ?
(a) 1970 में 
(b) 1985 में
(c) 1981 में
(d) 1999 में 
27. जैम, जेल, स्क्वॉश इत्यादि का कच्चा माल क्या है ?
(a) सब्जी
(b) गन्ना
(c) फल
(d) मकई   
28. हारवेस्टर और थ्रेसर का प्रयोग किस कार्य में किया जाता है ?
(a) चीनी उत्पादन में 
(b) कृषि में
(c) पशुपालन में
(d) दूध-उत्पादन में
29. निम्नलिखित में से कौन-सी विशेषताएँ निर्वाह कृषि संबंधित हैं ?
1. भूमि के छोटे टुकड़ों पर आदिम कृषि औजारों के द्वारा खेती की जाती है।
2. यह कृषि प्राय: मानूसन पर निर्भर है।
3. यह श्रम प्रधान कृषि हैं।
4. फसलों की उत्पादकता मृदा की प्राकृतिक उर्वरता पर निर्भर होती है।
कूट:
(a) केवल 1, 2 और 3
(b) केवल 1 और 3
(c) केवल 2, 3 और 4
(d) उपर्युक्त सभी 
30. निम्नलिखित में से कौन उस कृषि प्रणाली को दर्शाती है, जिसमें एक ही फसल लंबे-चौड़े क्षेत्र में उगाई जाती है ?
(a) स्थानांतरी कृषि
(b) रोपण कृषि
(c) बागवानी कृषि
(d) गहन जीविका कृषि
31. भारत में शस्य प्रारूप के संबंध में नीचे दिए गए कथनों पर विचार कीजिये:
1. रबी की फसलों को शीत ऋतु में बोया जाता है। 
2. खरीफ की फसलें मानसून के आगमन के साथा बोई जाती है।
3. रबी और खरीफ फसल ऋतुओं के बीच ग्रीष्म ऋतु में बोई जाने वाली फसल को 'जायद' कहा जाता है।
उपर्युक्त में से कौन-सा/से कथन असत्य है / हैं ?
(a) केवल 1 और 2
(b) केवल 3
(c) केवल 2 और 3
(d) इनमें से कोई नहीं
हमसे जुड़ें, हमें फॉलो करे ..
  • Telegram ग्रुप ज्वाइन करे – Click Here
  • Facebook पर फॉलो करे – Click Here
  • Facebook ग्रुप ज्वाइन करे – Click Here
  • Google News ज्वाइन करे – Click Here
]]>
Wed, 10 Apr 2024 09:46:52 +0530 Jaankari Rakho
General Competition | Geography | भारत की सिंचाई एवं नहरें https://m.jaankarirakho.com/956 https://m.jaankarirakho.com/956 General Competition | Geography | भारत की सिंचाई एवं नहरें
  • सिंचाई : कृत्रिम तरीका से फसलों को नहर, नलकूप या अन्य माध्यमों से जल पहुँचाना ही सिंचाई कहलाता है। भारत में सिंचाई का सबसे प्रमुख साधन कुआँ एवं नलकूप हैं। वही सिंचाई का दूसरा प्रमुख साधन नहर है। वही दक्षिण भारत में सिंचाई का सबसे प्रमुख साधन तालाब है। दक्षिण भारत में तालाब से सबसे ज्यादा खेती तमिलनाडु राज्य में किया जाता है। उत्तर भारत - नलकूप । 
  • भारत सरकार ने जल संरक्षण को बढ़ावा देने के उद्देश्य से भारत की प्रथम राष्ट्रीय जल नीति की घोषणा 1987 ई० में किया है। इस जल नीति में पहली बार संशोधन 2002 ई० में तथा दूसरी बार संशोधन 2012 ई० में किया गया।
  • राष्ट्रीय जल संसाधन परिषद् का गठन 1983 ई० में किया गया। इस संगठन के पदेन अध्यक्ष प्रधानमंत्री होते हैं। 
  • भारत में सिंचाई परियोजना को तीन वर्गों में विभाजित किया गया हैं-
    1. वृहद या वृहतम सिंचाई परियोजना : वैसी सिंचाई परियोजना जिसमें 10,000 हेक्टेयर से अधिक कृषि योग्य भूमि शामिल हो वृहतम सिंचाई परियोजना कहलाता है। इसके अंतर्गत बहुउद्देशीय परियोजना को रखा जाता है।
    2. मध्यम सिंचाई परियोजना : वैसी सिंचाई परियोजना जिसके अंतर्गत 3. हेक्टेयर से 10,000 हेक्टेयर कृषि योग्य भूमि शामिल हो, मध्यम सिंचाई परियोजना कहलाता है। इसके अंतर्गत नहर को रखा जाता है।
    3. लघु सिंचाई परियोजना : वैसी सिंचाई परियोजना जिसके अंतर्गत 2,000 हैक्टेयर से कम कृषि योग्य भूमि शामिल हो, लघु सिंचाई परियोजना कहलाता है। इसके अंतर्गत कुआँ, नलकूप, तालाब इत्यादि आते हैं।
  • Note : 1. भारत में सबसे अधिक सिंचाई लघु सिंचाई परियोजना ( 63%) के तहत किया जाता है।
    2. विश्व में सबसे ज्यादा सिंचित भूमि चीन में है, वही भारत का स्थान दूसरा है।
  • ड्रिप सिंचाई : सिंचाई की वैसी तकनीक जिसमें पौधों के जड़ में बूंद-बूंद पानी टपका कर सिंचाई किए जाते हैं उसे ही ड्रिप सिंचाई कहते हैं। ड्रिप सिंचाई का जन्मदाता देश इजराइल है।
  • फर्टिगेशन सिंचाई : सिंचाई की वैसी तकनीक जिसमें जल के साथ घुलनशील उर्वरक भी पौधों तक पहुँचाया जाता है उसे ही फर्टिगेशन सिंचाई कहते हैं। 2. इस सिचाई का जन्मदाता दश इजराइल माना जाता है। 

वस्तुनिष्ठ प्रश्न एवं उत्तर

1. पृथ्वी पर पीने लायक मीठा जल कितना प्रतिशत है ?
(a) 2.5 
(b) 4.5
(c) 9.5
(d) 96.5
2. जल प्रदूषण को रोकने में कौन उपाय कारगार नहीं है ?
(a) कल-कारखानों के अपशिष्ट रासायनिक पदार्थ जलाशय में न गिराए जाएँ।
(b) कूड़ा-करकट मल आदि नदियों में न गिराए जाएँ।
(c) तालाबों में कीटनाशक दवाएँ छिड़की जाएँ। 
(d) सड़ी-गली वस्तुएँ तालाबों में न फेंकी जाएँ।
3. केरल में सिंचाई की आवश्यकता क्यों नहीं होती ?
(a) वहाँ खेती नहीं की जाती है।
(b) वहाँ लोग मुख्यतः मछली पकड़ते हैं।
(c) वहाँ करीब सालोंभर वर्षा हुआ करती है। 
(d) वहाँ उद्योगों का विकास हुआ है।
4. भूमिगत जल का उपयोग भारत के किस क्षेत्र में अधिक किया जाता है ?
(a) राजस्थान में
(b) महाराष्ट्र में
(c) दक्षिण भारत में
(d) उत्तर भारत में
5. भाखड़ा नांगल बाँध किस नदी पर बना है
(a) ब्यास  
(b) सतलज
(c) रावी
(d) चेनाब
6. संसार में सबसे लंबा बाँध किस नदी पर बना है ?
(a) यमुना 
(b) दामोदर
(c) महानदी
(d) कोसी
7. किस नदी परियोजना से राजस्थान और मध्य प्रदेश में सिंचाई तथा मिट्टी संरक्षण की सुविधाएँ मिली हैं ?
(a) नागार्जुन सागर परियोजना
(b) चंबलघाटी परियोजना
(c) नर्मदाघाटी परियोजना
(d) हीराकुद परियोजना
8. दक्षिण भारत की सबसे बड़ी नदीघाटी परियोजना किस राज्य में अवस्थित है ?
(a) आंध्र प्रदेश एवं तेलंगाना में
(b) ओडिशा में
(c) तमिलनाडु में
(d) मध्य प्रदेश में
9. भारत में जल-प्राप्ति का मुख्य स्रोत क्या है ?
(a) हिमनदों का पिघलना 
(b) वर्षा-जल
(c) नहरों प्राप्त जल
(d) भूमिगत जल
10. भारत के किस भाग में झीलों और तालाबों का उपयोग अधिक होता है ?
(a) असम में
(b) पश्चिम बंगाल में
(c) उत्तरी भारत में
(d) दक्षिणी भारत में 
11. किस नदी प्रणाली का जल सबसे अधिक उपयोग में आता है ?
(a) गंगा
(b) ब्रह्मपुत्र
(c) ताप्ति
(d) कावेरी
12. सोन नदी बिहार के किस भाग में बहती है ?
(a) उत्तर-पूर्वी भाग में
(b) दक्षिण- पश्चिम भाग में
(c) दक्षिण-पूर्वी भाग में 
(d) उत्तर-पश्चिम भाग में
13. इनमें राजस्थान की सबसे महत्वपूर्ण परियोजना कौन-सी है ?
(a) चंबल परियोजना
(b) नागार्जुन सागर परियोजना
(c) भाखड़ा नांगल परियोजना 
(d) इंदिरा गाँधी नहर परियोजना
14. इनमें किस परियोजना का मुख्य उद्देश्य बाढ़ नियंत्रण रहा है ?
(a) चंबल
(b) भाखड़ा
(c) कोसी
(d) हीराकुद
15. विश्व के कुल जल का कितना महासागरों में पाया जाता है?
(a) 96.5% 
(b) 90%
(c) 98%
(d) 95%
16. प्रतिव्यक्ति प्रतिवर्ष जल उपलब्धता के संदर्भ में विश्व में भारत का कौन स्थान है ?
(a) 182वाँ
(b) 150वाँ
(c) 132वाँ
(d) 100वाँ
17. वर्षा जल संग्रहण के ढाँचों को हर घर में बनाना किस राज्य में कानूनन अनिवार्य है ?
(a) बिहार
(b) तमिलनाडु
(c) पश्चिम बंगाल
(d) मध्य प्रदेश
18. भारत सरकार ने कब राष्ट्रीय पेयजल मिशन चालू किया?
(a) 1956 में
(b) 1966 में
(c) 1976 में
(d) 1986 में
19. निम्नलिखित में कौन-सा/से कथन बहुउद्देशीय नदी परियोजना के संबंध में सत्य है / है ?
1. बहुउद्देशीय परियोजनाएँ जल बहाव को नियंत्रित करके बाढ़ पर काबू पाती हैं।
2. बहुउद्देशीय परियोजनाएँ घरेलू और औद्योगिक उपयोग हेतु जल की आपूर्ति करती हैं।
3. बहुउद्देशीय परियोजनाएँ के कारण भूमि निम्नीकरण की समस्या बढ़ती है।
कूट:
(a) केवल 1 और 2
(b) केवल 2 और 3
(c) केवल 2
(d) 1, 2 और 3
20. कृष्णा नदी जल विवाद किन राज्यों के बीच है ?
(a) महाराष्ट्र, कर्नाटक, आंध्र प्रदेश, तेलंगाना
(b) महाराष्ट्र, कर्नाटक, आंध्र प्रदेश, तमिलनाडु
(c) कर्नाटक, आंध्र प्रदेश
(d) महाराष्ट्र, कर्नाटक, केरल, आंध्र प्रदेश
21. 'सलाल प्रोजेक्ट' का संबंध किस नदी से है ? 
(a) गोदावरी नदी
(b) चेनाब नदी 
(c) दामोदर नदी
(d) रावी नदी
22. 'वर्षा जल संग्रहण' के तरीके 'खादीन' और 'जोहड़' भारत के किस राज्य में प्रचलित है ? 
(a) राजस्थान 
(b) मेघालय
(c) महाराष्ट्र
(d) तमिलनाडु
23. भारत में भौमजल (Groundwater) का सर्वाधिक उपयोग करने वाले राज्य निम्नलिखित में से कौन-से हैं ? 
(a) पंजाब, हरियाणा, राजस्थान और तमिलनाडु
(b) छत्तीसगढ़, ओडिशा और केरल
(c) गुजरात, तर-प्रदेश, बिहार, त्रिपुरा, और महाराष्ट्र
(d) इनमें से कोई नहीं
24. निम्नलिखित में से किस नदी बेसिन का कुल पुनः पूर्ति योग्य भौमजल संसाधन सबसे कम हैं ? 
(a) तापी
(b) गंगा
(c) स्वर्णरेखा
(d) चंबल तथा सहायक नदियाँ
25. निम्नलिखित में से किस क्षेत्र में धरातलीय और भौमजल का उपयोग सर्वाधिक होता है ? 
(a) औद्योगिक क्षेत्र
(b) घरेलू सेक्टर
(c) कृषि
(d) इनमें से कोई नहीं
26. धरातलीय जल संसाधन के संदर्भ में निम्नलिखित कथनों पर विचार कीजिये:
1. धरातलीय जल के मुख्य स्त्रोत- नदियाँ, झीलें, तलैया और तालाब है।
2. भारत में सभी नदी बेसिनों में औसत वार्षिक प्रवाह का एक-तिहाई धरातलीय जल का ही उपयोग किया जा सकता है।
उपर्युक्त में से कौन - सा /से कथन सही है / हैं ?
(a) केवल 1 
(b) केवल 2
(c) 1 और 2 दोनों
(d) न तो 1 और न ही 2
27. निम्नलिखित कथनों में से कौन-सा/से कथन सही नहीं है/हैं ?
1. पंजाब, हरियाणा और पश्चिमी उत्तर प्रदेश में निवल बोए गए क्षेत्र का 85% भाग वर्षा आश्रित हैं।
2. महाराष्ट्र और राजस्थान में भूमिगत जल में फ्लोराइड के संकेंद्रण के बढ़ने का कारण भौमजल संसाधन का अत्यधिक उपयोग हैं।
उपर्युक्त में से कौन - सा /से कथन सही है / हैं ?
(a) केवल 1
(b) केवल 2
(c) 1 और 2 दोनों
(d) न तो 1 और न ही 2
28. निम्नलिखित कथनों पर विचार कीजिये:
1. नदियों में प्रदूषकों का संकेंद्रण ठंडी के समय अधिक कथनों पर विचार कीजिये: होता हैं।
2. भारत के नदी क्षेत्रों में सबसे अधिक प्रदूषित नदी क्षेत्र दिल्ली और इटावा के बीच यमुना नदी क्षेत्र हैं।
3. भौमजल प्रदूषण का कारण भारी विषैली धातुओं- फ्लोराइड और नाइट्रेट का संकेंद्रण हैं।
4. केंद्रीय प्रदूषण नियंत्रण बोर्ड (CPCB), राज्य प्रदूषण नियंत्रण बोर्ड (SPCB) के साथ मिलकर राष्ट्रीय जल-संसाधन की गुणवत्ता की मॉनीटरिंग करता हैं।
उपर्युक्त में से कौन-से कथन सही हैं ?
(a) केवल 1, 3 और 4
(b) 1) केवल 1, 2 और 3
(c) केवल 2, 3 और 4
(d) 1, 2, 3 और 4
29. निम्नलिखित में से कौन-सा / से अधिनियम जल प्रदूषण से संबंधित है/हैं ? 
(a) जल (प्रदूषण निवारण और नियंत्रण) अधिनिमय, 1974
(b) पर्यावरण (संरक्षण) अधिनिमय, 1986
(c) जल उपकर अधिनिमय, 1977
(d) उपर्युक्त सभी।
30. जल का पुन: चक्र और पुन: उपयोग के संदर्भ में निम्नलिखित में से कौन-सा/से कथन सही है/हैं ?
1. पुन: चक्र और पुन: उपयोग द्वारा अलवणीय जल की उपलब्धता को सुधारा जा सकता है।
2. कम गुणवत्ता वाले जल का प्रयोग उद्योगों में करने से जल पर होने वाली लागत को कम किया जा सकता है।
3. पुनः चक्रण द्वारा पुनः पूर्ति योग्य जल की उपादेयता व्यापक है।
कूट :
(a) केवल 1
(b) केवल 2 और 3
(c) केवल 1 और 2
(d) 1, 2 और 3
31. जल संभर प्रबंधन के संबंध में निम्नलिखित कथनों पर विचार कीजिये:
1. 'हरियाली' केंद्र सरकार द्वारा प्रवर्तित जल संभर प्रबंधन प्रोजेक्ट (Watershed Managemnet Project) हैं।
2. 'नीरू - मीरू' कार्यक्रम आंध्र प्रदेश में तथा 'अरवारी पानी संसद' राजस्थान में जल प्रबंधन के लिए चलाए गए कार्यक्रम हैं।
3. जल संभर से तात्पर्य, मुख्य रूप से धरातलीय और भौमजल-संसाधनों के दक्ष प्रबंधन से हैं।
कूट:
(a) केवल 1 और 2
(b) केवल 2 और 3
(c) केवल 3
(d) 1, 2 और 3
32. राष्ट्रीय जल नीति, 2012 के संबंध में निम्नलिखित कथनों पर विचार कीजिये:
1. इस नीति में एक राष्ट्रीय जल संरचना कानून बनाने, अंतर्राज्यीय नदियों तथा नदी घाटियों के इष्टतम विकास के लिये व्यापक विधान की आवश्यकता पर जोर दिया गया है।
2. जल संसाधन संरचनाओं के अभिकल्पन और प्रबंधन में जलवायु परिवर्तन के मद्देनजर अनुकूलन कार्यनीतियों को अपनाने तथा स्वीकार्य मानदंडों की समीक्षा पर जोर दिया गया है।
उपर्युक्त में से कौन - सा /से कथन सही है/ हैं ?
(a) केवल 1
(b) केवल 2
(c) 1 और 2 दोनों
(d) न तो 1 और न ही 2
33. लैगून तथा झीलों के सदंर्भ में निम्नलिखित कथनों पर विचार कीजिये:
1. लैगूनों और झीलों से प्राप्त जल संसाधन का उपयोग केरल तथा पश्चिम बंगाल राज्यों में पीने के लिए किया जाता है।
2. लैगूनों तथा झीलों के जल संसाधन का उपयोग मछली पालन और चावल को कुछ निश्चित किस्मों, नारियल आदि की सिंचाई में किया जाता है।
उपर्युक्त में से कौन-सा/से कथन सही है/हैं ?
(a) केवल 1
(b) केवल 2
(c) 1 और 2 दोनों
(d) न तो 1 और न ही 2
हमसे जुड़ें, हमें फॉलो करे ..
  • Telegram ग्रुप ज्वाइन करे – Click Here
  • Facebook पर फॉलो करे – Click Here
  • Facebook ग्रुप ज्वाइन करे – Click Here
  • Google News ज्वाइन करे – Click Here
]]>
Wed, 10 Apr 2024 09:10:28 +0530 Jaankari Rakho
General Competition | Geography | भारत की मृदा https://m.jaankarirakho.com/955 https://m.jaankarirakho.com/955 General Competition | Geography | भारत की मृदा
  • मिट्टी या मृदा अर्थात् Soil शब्द की उत्पत्ति लैटिन भाषा के Soium से हुआ है, जिसका अर्थ 'फर्श' होता है। 
  • मृदा के अध्ययन करने वाले विज्ञान को Pedology कहते हैं।
  • मृदा निर्माण की प्रक्रिया को Pedogenesis (मृदा जनन) कहा जाता है।
  • भारतीय कृषि अनुसंधान परिषद ने भारतीय मृदा को आठ वर्गों में बाँटा है। भारतीय कृषि अनुसंधान परिषद नामक संगठन की स्थापना 1929 ई० में हुई है। इसका मुख्यालय नई दिल्ली में है। यह संस्था कृषि मंत्रालय के अधीन काम करती है। इस संस्था के अनुसार चार प्रकार की मिट्टी जो भारत में वृहद स्तर पर पायी जाती है, जो निम्न है-
    1. जलोढ़ मिट्टी - 40% (43%)
    2. लाल-पीली मिट्टी - 18%
    3. काली मिट्टी - 15%
    4. लैटेराइट मिट्टी - 4% (लगभग)
  • जलोढ़ मृदा : जल के द्वारा बहाकर लायी गयी मृदा को जलोढ़ मृदा कहते हैं। इसे कांप या कछारी मृदा भी कहते हैं। यह मृदा भारत में सबसे ज्यादा क्षेत्रों में पायी जाती है तथा यह सबसे अधिक उपजाऊ मृदा है। यह मृदा दो प्रकार का होता है-
    (i) बांगरअ (ii) खादर
    Note : नयी जलोढ़ मृदा को खादर तथा पुरानी जलोढ़ मृदा को बांगर कहते हैं। इन दोनों में अधिक उपजाऊ खादर होता है।
  • भारत में जलोढ़ मृदा सबसे अधिक उर भारत के मैदानी भाग में तथा भारत के तटीय मैदान में देखने को मिलता है। हिमालय से बहने वाली नदियों के कारण उत्तरी भारत में जलोढ़ मृदा की व्यापक स्तर पर उपस्थिति देखने को मिलता है। वही प्रायद्वीपीय नदियों के कारण जलोढ़ मृदा की उपस्थिति तटीय भाग में अधिक देखने मिलती है। 
  • Note : सबसे ज्यादा जलोढ़ मिट्टी उत्तर प्रदेश राज्य में पायी जाती है।
  • जलोढ़ मृदा में गेहूँ, गन्ना, जौ, धान, मक्का, जूट इत्यादि उपजता है।
  • जलोढ़ मृदा में पोटाश सर्वाधिक मात्रा में पायी जाती है। पोटाश के बाद इस मृदा में चूना की अधिकता होती है। वही जलोढ़ मृदा में नाइट्रोजन, फॉस्फोरस और ह्यूमस की कमी होती है।
  • काली मिट्टी : अवसादी चट्टानों के अपरदन से काली मिट्टी का निर्माण होता है। इसे रेगुर मिट्टी भी कहा जाता है। यह मिट्टी कपास के उत्पादन के लिए प्रसिद्ध है। जिस कारण इसे काली कपासी मिट्टी भी कहते हैं। इसे स्वतः जुताई वाली मिट्टी भी कहा जाता है। इस मिट्टी में जल धारण करने की क्षमता अत्यधिक होती है। इस मिट्टी में बिना सिंचाई के भी कृषि कार्य किए जा सकते हैं। इस मिट्टी को अदैव मातृक मिट्टी भी कहा जाता है। इस मिट्टी को उत्तर प्रदेश में करेल के नाम से पुकारा जाता है । यह मिट्टी भारत के कुल क्षेत्रफल का 15% भूभाग पर पाया जाता है। इस मिट्टी का रंग काला टिटेनि फेरस मैग्नेटाइट की उपस्थिति के कारण होता है। इस मिट्टी में कपास के अलावे गन्ना, प्याज जैसे खाद्य पदार्थों का भी उत्पादन किया जाता है । यह मिट्टी दक्कन के पठारी क्षेत्रों में सबसे ज्यादा पायी जाती है। अगर हम राज्य विशेष की बात करें तो यह मिट्टी सबसे ज्यादा महाराष्ट्र राज्य में पायी जाती है।
  • लाल-पीली मिट्टी : इसका रंग लौह ऑक्साइड (फेरिक/फेरस ऑक्साइड) की उपस्थिति के कारण लाल होता है। यह अम्लीय प्रकृति की मिट्टी होती है। इस मिट्टी की अम्लीयता को दूर करने के लिए चूना का प्रयोग किया जाता है। यह मिट्टी दक्षिण भारतीय राज्यों में सबसे ज्यादा मात्रा में पायी जाती है। यह मिट्टी तमिलनाडु में सबसे अधिक पायी जाती है। इस मिट्टी में कॉफी, चाय, रबड़, काजू इत्यादि का उत्पादन किया जाता है।
  • अम्लीय मिट्टी : वैसी मिट्टी जिसमें चूना की कमी होती है अम्लीय मिट्टी कहलाती हैं।
  • क्षारीय मिट्टी : वैसी मिट्टी जिसमें चूना की अधिकता होती है क्षारीय मिट्टी कहलाती है।
  • लाल-पीली मिट्टी भारत के कुल क्षेत्रफल का 18% भू-भाग पर पायी जाती है। यह भारत की दूसरी सबसे प्रमुख मिट्टी है।
  • लैटेराइट मिट्टी : वैसा क्षेत्र जहाँ तापमान और वर्षा अधिक होता है उन क्षेत्रों में लैटेराइट मिट्टी पायी जाती है। यह मिट्टी भारत के कुल क्षेत्रफल का, 4% भू-भाग पर पायी जाती है। यह मिट्टी सबसे अधिक केरल राज्य में पायी जाती है। इस मिट्टी में चाय, कॉफी, रबड़ इत्यादि का उत्पादन किया जाता है।
  • गिट्टियों का क्रम (घटते क्रम में) : [ जलोढ़ > लाल-पीली काली मिट्टी > लैटेराइट]
  • मृदा अवकर्षण : मृदा के उर्वरता में ह्रास (कमी) का होना मृदा अवकर्षण कहलाता है।
  • संयुक्त राष्ट्र महासभा ने 2013 ई० में यह घोषणा किया कि मृदा के प्रति लोगों को जागरूक करने हेतु प्रत्येक वर्ष 5 दिसम्बर को विश्व मृदा दिवस मनाया जाएगा।
  • Note : 6 से 7 pH मान वाले मृदा को बेहतर मृदा माना जाता है।
    PH = Power of Hydrogen
  • पर्वतीय मृदा : यह मुख्य तौर पर भारत के पर्वतीय क्षेत्र जैसे - हिमालयी क्षेत्र, उत्तरी-पूर्वी राज्य तथा दक्षिण भारत के पहाड़ी क्षेत्र में पायी जाती है। यह मृदा पर्वतीय ढालों में विकसित हुई है। जिस कारण यह Immature (अप्रौढ़) मृदा है। इस मृदा में चाय, कॉफी, मसाला का उत्पादन किया ज है।
  • भारत के मरूस्थलीय क्षेत्रों में जो मृदा पायी जाती है उसे शुष्क अथवा मरूस्थलीय मृदा कहा जाता है। यह मृदा मुख्य तौर पर पश्चिमी राजस्थान, पश्चिमी हरियाणा, दक्षिणी पंजाब और उत्तरी गुजरात में पाया जाता है। इस मृदा में वैसे अनाज का उत्पादन होता है जिसे पानी की आवश्यकता कम होती है जैसे- ज्वार, बाजरा, रागी इत्यादि ।
  • पंजाब के क्षेत्र में लवणीय मृदा को रेह, कल्लर या उसर कहा जाता है। यह अनुर्वर मृदा होता है । लवणीय मृदा को कृषि योग्य बनाने हेतु जिप्सम या चूना का प्रयोग किया जाता है।
  • पीट एवं जैव मृदा खास तौर पर डेल्टाई इलाका में पाया जाता है।
  • मिट्टी के कणों में बालू का व्यास 2.0-1.0mm तथा मृतिका का व्यास 0.002mm से कम होता है।
  • मृदा अपरदन : मृदा के ऊपरी परतों का क्षरण या विनाश मृदा अपरदन कहलाता है। । मृदा अपरदन पाँच चरणों में होता है जिसका क्रम निम्न हैं-
    बूँद अपरदन (Splash) → परत अपरदन → रिल अपरदन → अवनालिका अपरदन → धारा चैनल अपरदन 
  • अत्यधिक पशुचारण, वनों की कटाई, झूम कृषि, वायु अपरदन, हिमानी अपरदन के कारण मृदा अपरदन होता है।
  • मृदा अपरदन को रोकने हेतु वनारोपण, कृषिवानिकी, झूम कृषि पर रोक, पशुचारण पर रोक लगाया जाता है ।
  • Note : मृदा अपरदन को रोकने हेतु भारत सरकार ने 1953 ई० में केन्द्रीय मृदा संरक्षण बोर्ड का गठन किया।

Objective ( भारत की मृदा )

  • बैसाल्ट लावा के अपक्षय के कारण काली मिट्टी का निर्माण होता है ।
  • दक्षिणी ट्रैप की रेगुड़ मिट्टियाँ काली होती है।
  • लावा मिट्टियाँ मालवा पठार में पायी जाती है।
  • मालवा पठार की प्रमुख मिट्टी काली मिट्टी है |
  • झारखंड में काली मिट्टी राजमहल के पहाड़ी प्रदेश में पायी जाती है।
  • क्रेब्स के अनुसार रेगुर मिट्टी अनिवार्य रूप से परिपक्व मिट्टी है।
  • भारत के उत्तरी मैदान की मृदा सामान्यतः तलोच्चन से बनी है।
  • जलोढ़ मिट्टी के लिए न्यूनतम उर्वरक की आवश्यकता होती है।
  • अम्लीय मिट्टी में जिप्सम का प्रयोग करके उसे फसल उगाने के उपयुक्त बनाया जाता है।

वस्तुनिष्ठ प्रश्न एवं उत्तर

1. भारत की कितनी भूमि पर कृषि की जाती है ?
(a) 37 प्रतिशत
(b) 48.83 प्रतिशत
(c) 57 प्रतिशत
(d) 62 प्रतिशत
2. भूमि ह्रास रोकने का क्या उपाय किया जा सकता है ?
(a) उपजाऊ भूमि पर कल-कारखाने न खड़े करना
(b) जलमग्न भूमि का उद्धार करना
(c) पहाड़ी भागों में सीढ़ीनुमा खेत बनाना
(d) इनमें सभी
3. रवेदार और रूपांतरित चट्टानों के टूटने से किस प्रकार की मिट्टी बनती है ?
(a) वाहित मिट्टी
(b) दोमट मिट्टी
(c) बलुई मिट्टी
(d) लाल मिट्टी
4. किस मिट्टी में लोहे के ऑक्साइड मिला करते हैं ?
(a) लाल मिट्टी
(b) लैटेराइट मिट्टी
(c) काली मिट्टी
(d) जलोढ़ मिट्टी
5. भूमि और मिट्टी का कटाव कैसे रोका जा सकता है ?
(a) फसल चक्र अपनाकर
(b) पशुओं की चराई पर नियंत्रण कर
(c) वनों की अंधाधुंध कटाई
(d) इनमें सभी
6. विश्व में कृषि कार्य के लिए जितनी भूमि उपलब्ध है,उसमें भारत का हिस्सा कितना है ?
(a) 6.9 प्रतिशत 
(b) 11.9 प्रतिशत
(c) 19.9 प्रतिशत
(d) 16.5 प्रतिशत
7. विहार में प्राय: कैसी मिट्टी पाई जाती है ?
(a) लाल मिट्टी 
(b) काली मिट्टी
(c) जलोढ़ मिट्टी
(d) लैटेराइट मिट्टी
8. भारत के किस क्षेत्र में मरुस्थलीय मिट्टी पाई जाती है ?
(a) पूर्वोत्तर भारत में
(b) भारत के पश्चिमी शुष्क भाग में
(c) दक्षिण भारत के नीलगिरि क्षेत्र में
(d) झारखंड में
9. इनमें कौन लाल मिट्टी का क्षेत्र है ?
(a) पंजाब का मैदान
(b) छोटानागपुर का पठार
(c) पूर्वतटीय मैदान
(d) हिमाचल प्रदेश
10. कृषि कार्य के लिए भारत की कौन-सी मिट्टी सबसे अधिक महत्वपूर्ण है ?
(a) काली मिट्टी
(b) लाल मिट्टी
(c) जलोढ़ मिट्टी
(d) पहाड़ी मिट्टी
11. इनमें कौन मिट्टी पौधों के लिए सबसे अधिक उपयुक्त है ?
(a) मोटे कणोंवाली मिट्टी
(b) सबसे महीन कणोंवाली चिकनी मिट्टी
(C) दोमट (loam), अर्थात् मध्यम आकार के कणोंवाली मिट्टी
(d) बलुई मिट्टी
12. इनमें कौन कारक मृदा-निर्माण में सहायक है ?
(a) स्थानीय जलवायु
(b) पूर्ववर्ती चट्टानें
(c) वनस्पतियाँ और जीव
(d) इनमें सभी
13. इनमें काली मिट्टी का क्षेत्र कौन है ?
(a) छोटानागपुर
(b) महाराष्ट्र
(c) गंगाघाटी
(d) अरुणाचल प्रदेश
14. प्रायद्वीपीय भारत की नदी घाटियों में कौन-सी मिट्टी मिलती है ?
(a) काली
(b) लाल
(c) रेतीली
(d) जलोढ़
15. भारत में चरागाह के अन्तर्गत (2008-09 में ) कितनी भूमि थी ?
(a) 3.38%
(b) 12%
(c) 19%
(d) 26%
16. इनमें कौन उपाय भूमि - ह्रास के संरक्षण में उपयुक्त हो सकता है ?
(a) भूमि को जलमग्न बनाए रखना
(b) बाढ़ नियंत्रण
(c) जनसंख्या - वृद्धि की दर में तेजी
(d) इनमें कोई नहीं
17. भारत में पर्वत का प्रतिशत क्या है ?
(a) 10
(b) 27
(c) 30
(d) 48
18. पंजाब में भूमि - ह्रास का मुख्य कारण क्या है ?
(a) वनोन्मूलन
(b) सघन कृषि
(c) अत्यधिक सिंचाई
(d) अत्यधिक पशुचारण
19. किस राज्य में सीढ़ीनुमा खेती प्रचलित है ?
(a) पंजाब में 
(b) हरियाणा में
(c) उत्तराखंड में
(d) बिहार में
20. निम्नलिखित में से कौन-से कारक मृदा निर्माण को प्रभावित करते हैं ?
1. उन्चावच
2. वनस्पति
3. जलवायु
4. समय
कूट :
(a) केवल 1 और 2
(b) केवल 1, 2 और 3
(c) केवल 2 और 3
(d) 1, 2, 3 और.. 4
21. निम्न में से कौन-से मदा के घटक हैं ?
1. खनिज
2. ह्यूमस
3. जल
4. वायु
कूट :
(a) केवल 1 और 2
(b) केवल 1, 2 और 3
(c) केवल 1, 3 और 4
(d) उपर्युक्त सभी
22. मृदा की विभिन्न परतों (संस्तरों) के संबंध में नीचे दिये गए कथनों पर विचार कीजिये:
1. पौधों में वृद्धि के लिए आवश्यक जैव पदार्थों का खनिज पदार्थ, पोषक तत्व तथा जल में संयोग सबसे ऊपरी परत 'क' संस्तर में होता है।
2. मृदा निर्माण की प्रक्रिया में सबसे नीचे की परत सबसे पहले बनती है।
उपर्युक्त में से कौन-सा/से कथन सत्य है/हैं ?
(a) केवल 1 
(b) केवल 2
(c) 1 और 2 दोनों
(d) न तो 1 और न ही 2
23. भारत में मृदा का सबसे व्यापक और सर्वाधिक उपजाऊ प्रकार कौन-सा है ?
(a) काली मृदा
(b) जलोढ़ मृदा
(c) लैटेराइट मृदा
(d) शुष्क मृदा
24. निम्नलिखित कथनों में से कौन-सा/से कथन सत्य है/हैं ? 
1. जलोढ़ मृदा प्रायद्वीपीय प्रदेश में पूर्वी तट की नदी डेल्टाओं और नदियों की घाटियों में पाई जाती है।
2. जलोढ़ मृदा में पोटाश की मात्रा अधिक पाई जाती है।
3. प्रतिवर्ष बाढ़ों के द्वारा निक्षेपित होने वाली नई जलोढ़ मृदा को 'खादर' कहते हैं।
कूट :
(a) केवल 1 
(b) केवल 1 और 2
(c) केवल 2 और 3
(d) उपर्युक्त सभी
25. निम्न में से किस मृदा को 'रेगुर' कहते हैं ?
(a) काली मृदा 
(b) लैटेराइट मृदा
(c) जलोढ़ मृदा
(d) पीट मृदा
26. निम्नलिखित में से किन-किन राज्यों में काली मृदा पाई जाती है ?
1. गुजरात
2. महाराष्ट्र
3. आंध्र प्रदेश 
4. पंजाब
कूट :
(a) केवल 1 और 2
(b) केवल 1, 2 और 3
(c) केवल 1, 3 और 4
(d) 1, 2, 3 और 4
27. निम्न में से कौन-सी विशेषताएँ काली मृदा से संबंधित है ?
1. काली मृदा लंबे समय तक नमी को बनाए रखती है।
2. काली मृदा में फॉस्फोरस, नाइट्रोजन और जैव तत्वों की अधिकता होती है। 
कूट :
(a) केवल 1
(b) केवल 2
(c) 1 और 2 दोनों
(d) न तो 1 और न ही 2
28. लाल मृदा का रंग लाल होने का कारण है :
(a) जैव पदार्थों की अधिकता
(b) फॉस्फोरस की अधिकता
(c) ह्यूमस की कमी
(d) लोहे के ऑक्साइड की उपस्थिति
29. लाल मृदा के संबंध में नीचे दिये गए कथनों पर विचार कीजिये:
1 लाल मृदा का विकास उन क्षेत्रों में हुआ, जहाँ रवेदार आग्नेय चट्टानें पाई जाती हैं।
2. लाल मृदा जलयोजित होने पर पीली दिखाई देने लगती है।
उपर्युक्त में से कौन - सा /से कथन सत्य है / हैं ?
(a) केवल 1
(b) केवल 2
(c) 1 और 2 दोनों
(d) न तो 1 और न ही 2
30. निम्नलिखित में से कौन-सा कथन लैटेराइट मृदा के संबंध में सत्य नहीं है ?
(a) यह मृदा उच्च तापमान और भारी वर्षा के क्षेत्रों में विकसित होती है।
(b) ये बहुत ही अपक्षालित मिट्टियाँ हैं।
(c) ये मृदाएँ प्रायद्वीपीय के ऊँचे क्षेत्रों में पाई जाती है।
(d) ये मृदाएँ कृषि के लिए पर्याप्त उपजाऊ है।
हमसे जुड़ें, हमें फॉलो करे ..
  • Telegram ग्रुप ज्वाइन करे – Click Here
  • Facebook पर फॉलो करे – Click Here
  • Facebook ग्रुप ज्वाइन करे – Click Here
  • Google News ज्वाइन करे – Click Here
]]>
Wed, 10 Apr 2024 08:30:28 +0530 Jaankari Rakho
General Competition | Geography | भारत की जलवायु https://m.jaankarirakho.com/954 https://m.jaankarirakho.com/954 General Competition | Geography | भारत की जलवायु
  • भारत की जलवायु उष्ण कटिबंधीय मॉनसूनी जलवायु है।
  • जलवायु : किसी भी क्षेत्र में लंबे समय तक जो मौसम की स्थिति होती है, उसे ही जलवायु कहते हैं।
  • मौसम : किसी भी क्षेत्र में कुछ घंटा, कुछ दिन या कुछ सप्ताह तक जो वायुमंडलीय दशा होती है, उसे ही मौसम कहा जाता है।
  • मॉनसून : मॉनसून अरबी भाषा के मोसिम से बना है जिसका शाब्दिक अर्थ ऋतु होता है।
  • मॉनसूनी पवन : मॉनसूनी पवन उस पवन को कहते हैं जो ऋतु में परिवर्तन के साथ अपनी दिशा में परिवर्तन कर लेती है । भारत में दो प्रकार की मॉनसूनी पवन हैं-
    1. दक्षिणी-पश्चिमी मॉनसून
    2. उत्तरी-पूर्वी मॉनसून
    1. दक्षिणी-पश्चिमी मॉनसून : इस मॉनसून की उत्पत्ति भारत के दक्षिणी-पश्चिमी भाग से होती है जिस कारण इसे दक्षिणी-पश्चिमी मॉनसून कहते हैं । इस पवन की उत्पत्ति ग्रीष्म ऋतु में हिंद महासागर अर्थात् जलीय भाग से होती है जिस कारण इस पवन में नमी की मात्रा होती है इसलिए यह पवन भारत में वर्षा कराती है। 
  • Note : भारत में सर्वाधिक वर्षा दक्षिणी-पश्चिमी मॉनसून से ही होता है।
  • मॉनसून प्रस्फोट (मॉनसून ब्रस्ट ) : भारत में मॉनसून का आगमन जून के प्रथम सप्ताह में केरल के तट पर होता है इसी घटना को मॉनसून प्रस्फोट कहते हैं।
  • मॉनसून विच्छेद : वर्षा ऋतु में वर्षा का रुक-रुक कर होना मॉनसून विच्छेद कहलाता है ।
  • उत्तरी-पूर्वी मॉनसून : इसकी उत्पत्ति भारत के उत्तरी-पूर्वी भाग से होती है जिस कारण इसे उत्तरी-पूर्वी मॉनसून कहते हैं। इसकी उत्पत्ति हिमालय पर्वत से होती है अर्थात् स्थलीय भाग से होती है जिस कारण इसमें नमी नहीं होता है इसलिए यह मॉनसूनी पवन वर्षा नहीं कराती है। 
  • Note : 1. उत्तरी-पूर्वी मॉनसून या लौटता हुआ मॉनसून के कारण ही तमिलनाडु के तट पर जाड़े के दिनों में वर्षा होती है।
    2. दक्षिण-पश्चिम मॉनसून की दिशा उत्तर- पूर्व की ओर होती है तथा उत्तरी-पूर्वी मॉनसून की दिशा दक्षिण-पश्चिम की ओर होती है।
  • मॉनसून निवर्तन : मॉनसून के लौटने की घटना को मानसून निवर्तन कहते है। 
  • दक्षिणी-पश्चिमी मॉनसून जलीय भाग से स्थलीय भाग की ओर चलती है तथा उत्तरी-पूर्वी मॉनसून स्थलीय भाग से जलीय भाग की ओर चलती है।
  • भारत में शरद ऋतु या शीत ऋतु में वर्षा दो क्षेत्रों में होता है।
    1. उत्तरी-पश्चिमी भारत
    2. दक्षिणी-पूर्वी भारत ।
  • पश्चिमी विक्षोभ (जेट प्रवाह) : इसकी उत्पत्ति भूमध्य सागर से होती है। यह पूरब की ओर बहती है तथा काला सागर और कैस्पियन सागर से नमी प्राप्त करती है। यह भारत में प्रवेश उत्तरी-पश्चिमी भाग से करती है। यही मॉनसूनी पवन शरद या शीत ऋतु में पंजाब, राजस्थान, हरियाणा और दिल्ली जैसे क्षेत्रों में वर्षा कराती है। पश्चिमी विक्षोभ के कारण ही जम्मू-कश्मीर के क्षेत्र में शरद ऋतु में हिमपात होता है ।
  • Note : पश्चिमी विक्षोभ रबी के फसल (गेहूँ, चना, मटर, सरसों, आलू) के लिए लाभदायक होता है।
  • तमिलनाडु : शरद ऋतु में तमिलनाडु के तट पर वर्षा उत्तरी-पूर्वी मॉनसून या लौटते हुए मॉनसून के कारण होता है।
  • हिमालय पर्वत : हिमालय पर्वत के कारण ही दक्षिणी-पश्चिमी मॉनसून से भारत में वर्षा होती है। अगर हिमालय पर्वत नहीं होता तो भारत में सूखा की स्थिति होती । हिमालय पर्वत ध्रुवीय अथवा साइबेरियाई क्षेत्र से आने वाली ठंडी पवनों को रोक लेती है और भारत के नागरिकों को कड़ाके की ठंड से बचाती है।
  • Note : हिमालय पर्वत भारतीय जलवायु को प्रभावित करती हैं तथा जलवायु विभाजक के रूप में काम करती है।
  • जो स्थान समुद्र के जितना नजदीक होता है वहाँ उतना अधिक बारिश होता है तथा जो स्थान समुद्र के जितना दूर होता है वहाँ उतना कम बारिश होता है । जैसे- कोलकाता में 119 cm वर्षा, पटना में 105 cm वर्षा, प्रयागराज ( इलाहाबाद) में 76 cm वर्षा, दिल्ली में 56 cm वर्षा ।
  • भारत तथा विश्व में सर्वाधिक वर्षा मेघालय के मॉसिनराम में होता है। मॉसिनराम में वर्षा दक्षिणी-पश्चिमी मॉनसून के बंगाल की खाड़ी शाखा से होता है । ( 1141 cm)
  • भारत में औसत वार्षिक वर्षा 125 cm होता है ।
  • भारत में सबसे कम वर्षा लद्दाख के लेह में होता है। ( 30-40 cm)
  • भारत का सबसे गर्म स्थान राजस्थान के बीकानेर का ब्रियावली है।
  • भारत का सबसे ठंडा स्थान द्रास ( लद्दाख ) है ।
  • भारत का सबसे ठंडा मरूस्थल - सियाचीन
    भारत का सबसे ठंडा मरूस्थल - थार मरूस्थल
    भारत का सबसे सफेद मरूस्थल - कच्छ का रन
  • एलनीनो और भारतीय मॉनसून : एलनीनो एक गर्म जलधारा है इसकी उत्पत्ति दक्षिणी प्रशांत महासागर में पेरू के तट पर प्रत्येक चार से पाँच वर्ष की अवधि पर क्रिसमस के आसपास होती है। एलनीनो का शाब्दिक अर्थ बालकईशा होता है। एलनीनो जलधारा की उत्पत्ति होने से भारतीय मॉनसून पर निम्न प्रभाव पड़ता है-
    1. भारत में सूखा की स्थिति उत्पन्न होती है।
    2. भारत में मॉनसून का आगमन 10 से 15 दिन देर से पहुँचता है।
  • लानीनो : यह एक ठंडी जलधारा है इसकी उत्पत्ति प्रशांत महासागर से होती है। इस जलधारा का भारतीय मॉनसून पर प्रभाव पड़ता है। इसके कारण भारत में बाढ़ की स्थिति उत्पन्न होती है।
  • समुद्र देर से ठंडा और देर से गर्म होता है।
  • स्थल तुरंत ठंडा और तुरंत गर्म होता है।
  • वायु हमेशा उच्च दाब से निम्न दाब (L.P.) की ओर चलती है।
  • जलधारा दो प्रकार की होती है-.
    1. गर्म
    2. ठंडी।
  • गर्म जलधारा : विषुवत रेखीय क्षेत्र में ध्रुव की ओर चलने वाली जलधारा को गर्म जलधारा कहते हैं।
  • ठंडी जलधारा : ध्रुव से विषुवत रेखा की ओर चलने वाली जलधारा को ठंडी जलधारा कहते हैं।
  • गर्म जलधारा अपने मार्ग में आने वाले जल को गर्म बना देती है और ठंडी जलधारा अपने मार्ग में आने वाले जल को ठंडी बना देती है।

वस्तुनिष्ठ प्रश्न एवं उत्तर

1. भारतीय जलवायु की क्या विशेषता है ?
(a) जाड़ा और गर्मी की दो स्पष्ट ऋतुओं का होना
(b) वर्षा का मुख्य रूप से ग्रीष्मऋतु में होना
(c) जलवायु का मौसमी हवाओं से प्रभावित होना
(d) इनमें सभी
2. इनमें कौन-सा कथन सही नहीं है ?
(a) राजस्थान भारत का सबसे गर्म क्षेत्र है
(b) मेघालय भारत का सबसे अधिक वर्षा
(c) केरल में सालोभर समान तापमान मिलता है,
(d) बिहार में वर्षा जाड़े की ऋतु हुआ करती का क्षेत्र है
3. भारत का कौन-सा भाग वृष्टि - छाया में पड़ता है ?
(a) पश्चिम बंगाल
(b) पश्चिमी घाट का सम्मुख भाग
(c) पश्चिमी घाट का विमुख भाग
(d) छोटानागपुर
4. अक्टूबर से मध्य नवंबर तक कौन ऋतु मिलती है ?
(a) वसंत 
(b) शरद
(c) वर्षा
(d) ग्रीष्म
5. इनमें सबसे कम वर्षा का क्षेत्र कौन है ?
(a) बिहार
(b) झारखंड
(c) पंजाब
(d) केरल
6. गर्मी के दिनों में सबसे अधिक तापमान इनमें कहाँ मिलता है ?
(a) चेन्नई
(b) जोधपुर
(c) भोपाल
(d) पटना
7. भारत में औसतन कितनी वार्षिक वर्षा होती है ?
(a) 118 सेंटीमीटर
(b) 320 सेंटीमीटर
(c) 720 सेंटीमीटर
(d) 1,392 सेंटीमीटर
8. चेरापूँजी कहाँ स्थित है ?
(a) हिमाचल प्रदेश में 
(b) पश्चिम बंगाल में
(c) मेघालय में
(d ) अरुणाचल प्रदेश में
9. जून से सितंबर के बीच भारत में कौन-सी ऋतु रहती है ?
(a) शरद 
(b) ग्रीष्म
(c) वर्षा
(d) शीत
10. जाड़े में किस दिशा से चक्रवात आकर भारत में वर्षा करते हैं ?
(a) उत्तर- पूर्व से
(b) उत्तर-पश्चिम से
(c) दक्षिण - पूर्व से
(d) दक्षिण-पश्चिम से
11. इनमें सबसे अधिक वर्षा कहाँ होती है ?
(a) मंगलुरु में 
(b) चेरापूँजी में
(c) मासिनराम में
(d) निकोबार द्वीप में
12. भारत में उष्ण कटिबंधीय चक्रवात प्राय: कहाँ आते हैं ?
(a) गुजरात में
(b) असम में
(c) ओडिशा में
(d) पंजाब में
13. इनमें कौन अत्यल्प वर्षा का क्षेत्र हैं ?
(a) लेह
(b) केरल
(c) नागपुर
(d) कन्याकुमारी
14. तमिलनाडु में जाड़क में वर्षा कराने का श्रेय किसे है ?
(a) दक्षिण-पश्चिमी मॉनसून को 
(b) चक्रवात को 
(c) लौटते मॉनसून को
(d) इनमें कोई नहीं
15. उत्तर भारत में चलनेवाली धूलभरी आंधी को क्या कहा जाता है ?
(a) काल बैसाखी
(b) लू
(c) चक्रवात
(d) व्यापारिक भवन 
16. भारत में औसतन वार्षिक वर्षा कितनी होती है ?
(a) 150 सेंटीमीटर 
(b) 200 सेंटीमीटर
(c) 118 सेंटीमीटर
(d) 300 सेंटीमीटर
17. मानसून के संबंध में नीचे दिये गए कथनों पर विचार कीजिये:
1. मानसून शब्द की उत्पत्ति अरबी शब्द से हुई है। 
2. मानसून का अर्थ, एक वर्ष के दौरान वायु की दिशा में ऋतु के अनुसार परिवर्तन है।
उपर्युक्त में से कौन-सा/से कथन सत्य है / हैं ?
(a) केवल 1 
(b) केवल 2
(c) 1 और 2 दोनों
(d) न तो 1 और न ही 2
18. नीचे दिये गए कथनों को ध्यानपूर्वक पढ़िये-
1. मौसम एक विशेष समय में एक क्षेत्र के वायुमंडल की अवस्था को बताता है।
2. जलवायु अपेक्षाकृत लंबे समय की एक विशाल क्षेत्र की मौसम की अवस्थाओं तथा विविधताओं का औसत योग है।
3. मौसम तथा जलवायु दोनों के तत्त्व एक ही होते हैं। उपर्युक्त में से कौन-सा /से कथन सत्य है / हैं ?
(a) केवल 1
(b) केवल 1 और 2
(c) केवल 2 और 3
(d) उपर्युक्त सभी
19. एशिया महाद्वीप में मानसूनी जलवायु मुख्यतः पाई जाती है- 
1. दक्षिण एशिया में
2. पश्चिमी एशिया में
3. दक्षिण - पूर्वी एशिय में
कूट :
(a) केवल 1
(b) केवल 1 और 2
(c) केवल 1 और 3
(d) 1, 2 और 3
20. भारतीय वर्षण की प्रादेशिक विविधताओं के संबंध में नीचे दिये गए कथनों पर विचार कीजिये:
1. हिमालय में वर्षण मुख्यतः हिमपात के रूप में होता है।
2. तमिलनाडु के तटीय प्रदेशों में वर्षा शरद ऋतु के आरंभ में होती है।
3. उत्तरी मैदान में वर्षा की मात्रा पश्चिम से पूर्व की ओर घटती जाती है। Raush
निम्नलिखित में से कौन-से कथन सत्य हैं ?
(a) केवल 1 और 2 
(b) केवल 2 और 3
(c) केवल 1 और 3 
(d) उपर्युक्त सभी
21. किसी क्षेत्र की जलवायु को नियंत्रित करने वाले प्रमुख कारक हैं-
1. अक्षांश
2. समुद्र से दूरी
3. देशांतर
4. वायुदाब
5. तुंगता (ऊंचाई)
कूट :
(a) केवल 1, 2 और 5
(b) केवल 2, 3, 4 और 5 
(c) कंवल 1, 2, 4 और 5
(d) उपर्युक्त सभी
22. निम्नलिखित कथनों पर विचार कीजिये :
1. मानसूनी जलवायु की क्षेत्रीय विभिन्नता अनेक प्रकार की फसलों को उगाने में सहायक है।
2. मानसून का अचानक प्रस्फोट देश के व्यापक क्षेत्रों में मृदा अपरदन की समस्या उत्पन्न कर देता है।
3. भारत की जलवायु की विभिन्नता के कारण यहाँ के भोजन, वस्त्र और आवास में विविधता उजागर होती है।
उपर्युक्त में से कौन-सा/से कथन सत्य है/हैं ?
(a) केवल 1
(b) केवल 1 और 2
(c) केवल 2 और 3
(d) उपर्युक्त सभी
23. निम्नलिखित में कौन-सा/से कथन सत्य है / हैं ?
1. भारत की जलवायु में उष्ण कटिबंधीय एवं शीतोष्ण कटिबंधीय दोनों-जलवायु की विशेषताएँ उपस्थित हैं।
2. भारत में कर्क रेखा के उत्तरी भाग में कम दैनिक एवं वार्षिक तापांतर पाया जाता है।
3. भारत के दक्षिणी भाग में उच्च दैनिक तथा वार्षिक तापांतर के साथ विषम जलवायु पाई जाती है।
कूट:
(a) केवल 1
(b) कंवल 1 और 2
(c) केवल 2 और 3
(d) केवल 1 और 3
24. भारतीय जलवायु के संबंध में हिमालय पर्वत की भूमिका से संबंधित नीचे दिये गए कथनों पर विचार कीजिये :
1. हिमालय पर्वत एक जलवायु विभाजक की भाँति कार्य करता है।
2. हिमालय पर्वत भारतीय उपमहाद्वीप में वर्षा का कारण बनता है।
उपर्युक्त में से कौन-सा/से कथन सत्य है / हैं ?
(a) केवल 1
(b) केवल 2 
(c) 1 और 2 दोनों 
(d) न तो 1 और न ही 2
25. निम्नलिखित में से कौन-सा कारण भारत के उत्तर-पश्चिम भाग में शीत ऋतु में होने वाली वर्षा के लिए उत्तरदायी है ?
(a) चक्रवाती अवदाब
(b) मानसून की वापसी
(c) दक्षिण-पश्चिम मानूसन
(d) पश्चिमी विक्षोभ
26. पश्चिमी विक्षोभ के संदर्भ में नीचे दिये गए कथनों पर विचार कीजिये.
1. पश्चिमी विक्षोभ कैस्पियन सागर में उत्पन्न होते हैं।
2. भारत में इनका प्रवेश पश्चिमी जेट प्रवाह द्वारा होता है।
उपर्युक्त में से कौन - सा /से कथन सत्य है / हैं ?
(a) केवल 1
(b) केवल 2 
(c) 1 और 2 दोनों
(d) न तो 1 और न ही 2
27. ये पवनें तिब्बत के पठार के समानांतर हिमालय के उत्तर में एशिया महाद्वीप पर चलती हैं। तिब्बत की उच्चभूमि इन पवनों के प्रवाहों के मार्ग में अवरोधक का काम करती है, जिसके परिणामस्वरूप ये पवनें दो भागों में बँट जाती हैं। इसकी एक शाखा तिब्बत के पठार के उत्तर में तथा दूसरी शाखा हिमालय के दक्षिण में पूर्व की ओर बहती है।
उपर्युक्त विवरण संबंधित है-
(a) जेट प्रवाह से
(b) पश्चिमी विक्षोभ से
(c) दक्षिण-पश्चिम मानसून से
(d) इनमें से कोई नहीं
28. अंतः उष्णकटिबंधीय अभिसरण क्षेत्र ( आई. टी. सी. जेड ) के संबंध में नीचे दिये गए कथनों पर विचार कीजिये :
1. यह विषुवतीय अक्षांशों में विस्तृत गर्त एवं निम्न दाब का क्षेत्र होता है।
2. सूर्य की स्थिति में परिवर्तन के साथ यह क्षेत्र उत्तर या दक्षिण की ओर खिसकता है। 
उपर्युक्त में से कौन-सा/से कथन सत्य है/हैं ?
(a) केवल 1
(b) केवल 2
(c) 1 और 2 दोनों
(d) न तो 1 और न ही 2
29. निम्नलिखित में से कौन-सा/से कारक भारतीय जलवायु को प्रभावित करता है / करते हैं ?
1. एल-नीनो
2. पश्चिमी जेट धाराएँ
3. तिब्बत का पठार
नीचे दिये गए कूट का प्रयोग कर सही उत्तर चुनिये:
(a) केवल 1 और 3 
(b) केवल 2 और 3
(c) केवल 2
(d) उपयुक्त सभी 
30. भारत में मानसून के प्रस्फोट ( फटने) के लिए निम्नलिखित में से किसे जिम्मेदार माना जाता है ?
(a) पश्चिमी जेट धाराएँ 
(b) पूर्वी जेट प्रवाह
(c) व्यापारिक पवनें
(d) ला-नीना
31. भारतीय मानसून के संदर्भ में नीचे दिये गए कथनों पर विचार कीजिये :
1. मानसूनी पवनें नियमित पवनें होती हैं।
2. भारत में वर्षा लाने वाला एकमात्र तंत्र अरब सागर से उठने वाली दक्षिण-पश्चिम मानूसन धारा है।
3. भारतीय द्वीपों पर मानसून की सबसे पहली वर्षा होती है। तथा मानसून की वापसी भी सबसे बाद में होती है।
उपर्युक्त में से कौन-सा/से कथन सत्य है / हैं ?
(a) केवल 1
(b) केवल 1 और 2 
(c) केवल 1 और 3
(d) केवल 3
32. भारतीय मानसून की अरब सागर शाखा एवं बंगाल की खाड़ी शाखा मिलती है-
(a) नीलगिरी की पहाड़ियों पर
(b) पूर्वी हिमालयी क्षेत्र में
(c) गंगा के मैदान के उत्तर-पश्चिमी भाग में
(d) दोनों शाखाएँ कभी आपस में नहीं मिलती हैं
33. मराठवाड़ा, पुणे और बंगलूरू मे कम वर्षा होने के प्रमुख कारण हैं-
(a) यहाँ वनों का प्रतिशत बहुत कम है।
(b) ये क्षेत्र पश्चिमी घाट के वृष्टि छाया क्षेत्र में स्थित हैं।
(c) इन क्षेत्रों में प्रदूषण का अत्यधिक होना।
(d) कारण स्पष्ट नहीं है।
34 वर्षा ऋतु में तमिलनाडु तट के शुष्क रहने का / के कारण है/हैं ?
1. तमिलनाडु का तट बंगाल की खाड़ी की मानसूनी पवनों के समानांतर पड़ता है।
2. यह दक्षिण-पश्चिम मानसून की अरब सागर शाखा के दृष्टि छाया क्षेत्र में स्थित है।
नीचे दिये गए कूट का प्रयोग कर सही उत्तर चुनिये :
(a) केवल 1
(b) केवल 2
(c) 1 और 2 दोनों
(d) न तो 1 और न ही 2
35. मानसूनी वर्षा की विशेषताओं के संदर्भ में नीचे दिये गए कथनों पर विचार कीजिये :
1. दक्षिण-पश्चिम मानसून से प्राप्त होने वाली वर्षा मौसमी है।
2. मा वर्षा मुख्य रूप से उच्चवाच अथवा भू-आकृति द्वारा निर्यात होती है।
3. समुद्र से दूरी बढ़ने के साथ मानसूनी वर्षा की मात्रा घटने लगती है।
4. मानसूनी वर्षा का स्थानिक वितरण एकसमान है।
उपर्युक्त में से कौन-से कथन सत्य हैं ?
(a) केवल 1 और 3 
(b) केवल 1. 2 और 3
(c) केवल 2, 3 और 4
(d) केवल 2 और 4
हमसे जुड़ें, हमें फॉलो करे ..
  • Telegram ग्रुप ज्वाइन करे – Click Here
  • Facebook पर फॉलो करे – Click Here
  • Facebook ग्रुप ज्वाइन करे – Click Here
  • Google News ज्वाइन करे – Click Here
]]>
Wed, 10 Apr 2024 07:16:40 +0530 Jaankari Rakho
General Competition | Geography | भारत की नदियाँ झील, जलप्रपात एवं बहुउद्देशीय परियोजना https://m.jaankarirakho.com/953 https://m.jaankarirakho.com/953 General Competition | Geography | भारत की नदियाँ झील, जलप्रपात एवं बहुउद्देशीय परियोजना
  • भारत की नदियाँ : भारत में सैकड़ों की संख्या में नदियाँ हैं, भारत की नदियाँ अपना जल बंगाल की खाड़ी या अरब सागर में गिराती हैं। भारत की नदियाँ अपनी कुल प्रवाहित जल का 77% बंगाल की खाड़ी में गिराती हैं तथा 23% अरब सागर में गिराती हैं।
  • बंगाल की खाड़ी में जल गिराने वाली नदियाँ : गंगा, ब्रह्मपुत्र, स्वर्णरेखा, महानदी, गोदावरी, कृष्णा, कावेरी इत्यादि । 
  • अरब सागर में जल गिराने वाली नदियाँ : साबरमती, नर्मदा, ताप्ती नदी इत्यादि ।
  • उद्गम स्थल : नदियाँ जहाँ से निकलती हैं उसे उद्गम स्थल कहा जाता है। जैसे -गोदावरी का उद्गम स्थल महाराष्ट्र के नासिक का त्रयम्बक पहाड़ी है।
  • संगम या मुहाना : नदियाँ जहाँ समुद्र या सागर में मिलती हैं उसे संगम या मुहाना कहते हैं। जैसे- गोदावरी का संगम या मुहाना बंगाल की खाड़ी है।
  • अपवाह क्षेत्र या परवाह क्षेत्र : उद्गम स्थल से लेकर संगम या मुहाना तक के क्षेत्र को अपवाह क्षेत्र या परवाह क्षेत्र कहते हैं।
  • माही नदी : भारत में बहने वाली यह नदी कर्क रेखा को दो बार काटती है।
  • कांगो नदी या जायरे नदी : अफ्रीका महादेश में बहने वाली यह नदी विषुवत रेखा को दो बार काटती है।
  • नील नदी : यह विश्व की सबसे लंबी नदी है। यह नदी अफ्रीका महादेश के सबसे बड़े झील विक्टोरिया झील से निकलती है। विक्टोरिया झील से होकर विषुवत रेखा गुजरती है। नील नदी विक्टोरिया झील से निकलने के बाद उत्तर की ओर बहती हुई कर्क रेखा को पार करते हुए अपना जल भूमध्य सागर में गिराती है। हम यह कह सकते हैं कि नील नदी का संबंध विषुवत रेखा और कर्क रेखा दोनों से है।
  • लिम्पोपो नदी : अफ्रीका महादेश में बहने वाली यह नदी मकर रेखा को दो बार काटती है।
  • बारमेजो नदी : दक्षिण अमेरिका महादेश में बहने वाली यह नदी मकर जिससे होकर कर्क रेर को तीन बार काटती है।
  • Note : एकमात्र महादेश अफ्रीका महादेश है जिससे होकर कर्क रेखा, विषवुत रेखा और मकर रेखा तों गुजरती है।
  • कर्क रेखा तीन महादेश से होकर गुजरती है- 1. उत्तरी अमेरिका महादेश, 2. अफ्रीका महादेश, 3. एशिया महादेश |
  • विषुवत रेखा तीन महादेश से होकर गुजरती है- 1. दक्षिण अमेरिका महादेश, 2. अफ्रीका महादेश, 3. एशिया महादेश |
  • मकर रेखा तीन महादेश से होकर गुजरती है- 1. दक्षिण अमेरिका महादेश, 2. अफ्रीका महादेश, 3. ऑस्ट्रेलिया महादेश |
  • यूरोप महादेश और अंटार्कटिका महादेश ऐसा महादेश है जिसका संबंध कर्क, विषुवत और मकर रेखा किसी से भी नहीं है।
  • मकर रेखा ऑस्ट्रेलिया को लगभग दो बराबर भाग में बाँटती है।
  • नदियों का वर्गीकरण :
    1. पूर्ववर्त्ती नदी : वैसी नदी जो हिमालय के निर्माण के पूर्व से बहती है, पूर्ववर्ती नदी कहलाती है। जैसे—सिंधु, सतलज, ब्रह्मपुत्र नदी इत्यादि ।
    2. अनुगामी / अनुवर्त्ती नदी : वैसी नदियाँ जो ढ़ाल का अनुसरण करते हुए बहती है, अनुगामी नदी कहलाती है। जैसे- गोदावरी, कृष्णा, कावेरी इत्यादि ।
  • प्रायद्वीपीय भारत की नदियाँ सामान्यतः पश्चिम से पूर्व की ओर बहती हैं क्योंकि प्रायद्वीपीय भारत का पश्चिमी भाग अधिक ऊँचा है जबकि पूर्वी भाग की ऊँचाई कम हैं।
  • भारत की नदियों को मूल तौर पर दो भागों में बाँटकर पढ़ते हैं-
1. हिमालय की नदियाँ, 2. प्रायद्वीपीय भारत की नदियाँ ।
हिमालय की नदियाँ प्रायद्वीपीय भारत की नदियाँ
(i) हिमालय की नदियाँ पूर्ववर्ती नदी का उदाहरण हैं। (i) प्रायद्वीपीय भारत की नदियाँ अनुवर्ती या अनुगामी नदी के उदाहरण हैं।
(ii) यह नदियाँ बारहमासी/ सततवाहिनी / सदानीरा होती हैं क्योंकि इन नदियों को जल की प्राप्ति बर्फ और वर्षा दोनों से होता है। (ii) यह नदी मौसमी होती है क्योंकि इन नदियों को जल की प्राप्ति सिर्फ वर्षा से होती है।
(iii) यह नदी अपनी युवावस्था में है। (iii) यह नदी अपनी प्रौढ़ावस्था में है।
(iv) यह नदी अपरदन ( तोड़ना - फोड़ना) करती है । (iv) प्रायद्वीपीय भारत की कुछ नदियाँ डेल्टा का निर्माण करती हैं जैसे- महानदी, गोदावरी, कृष्णा, कावेरी, लेकिन प्रायद्वीपीय भारत की कुछ नदियाँ ऐसी हैं जो डेल्टा का निर्माण नहीं करती हैं जैसे- नर्मदा और ताप्ती ।
(v) हिमालय की नदियाँ डेल्टा का निर्माण करती हैं।
(vi) सामान्यतः हिमालय की नदियाँ अपने संगम या मुहाना पर डेल्टा का निर्माण करती है।
  • Note :
    1. हिमालयीय क्षेत्र को तीसरे ध्रुव की संज्ञा दी जाती है।
    2. विश्व का सबसे बड़ा डेल्टा सुंदरवन का डेल्टा है जिसका निर्माण गंगा और ब्रह्मपुत्र नदी की धारा करती है ।
    3. नर्मदा और तात्पी एश्चुअरी का निर्माण करती है।
    4. प्रायद्वीपीय भारत की नदियों में कावेरी नदी बारहमासी/ सततवाहिनी/सदानीरा होती है ।
  • गंगा नदी क्षेत्र : गंगा भारत की सबसे लंबी और बड़ी नदी है। हिन्दू धार्मिक ग्रंथों में इसे सबसे पवित्र नदी बताया गया है। पवित्रता के कारण उत्तर भारत के लोग इस नदी के जल का प्रयोग धार्मिक अनुष्ठान में करता है। इस नदी की लम्बाई 2525 km हैं (कुछ स्रोतों में गंगा की लम्बाई 2510km बतायी यी गयी है)- 2008 है) 2008 ई० में गंगा का भारत का राष्ट्रीय नदी घोषित किया गया है। 2017 ई॰ में उत्तराखण्ड उच्च न्यायालय (नैनीताल में) ने गंगा के साथ-साथ यमुना को भी जीवित मानव का दर्जा दिया है।
  • बद्रीनाथ के संतोपथ हिमनद ( ग्लेशियर) से अलकनंदा निकलती है।
  • केदारनाथ से मंदाकिनी नदी निकलती है।
  • गंगोत्री से भागीरथी नदी निकलती है।
  • धौलीगंगा, नंदाकिनी, पिंडार, मंदाकिनी, भागीरथी इत्यादि अलकनंदा की सहायक नदी है।
  • विष्णुप्रयाग : यह धौलीगंगा नदी अलकनंदा से आकर मिलती है।
  • नंदप्रयाग : यहाँ नंदाकिनी नदी अलकनंदा से आकर मिलती है।
  • कर्णप्रयाग : यहाँ पिंडार नदी अलकनंदा से आकर मिलती है।
  • रूद्रप्रयाग : यहाँ मंदाकिनी नदी अलकनंदा से आकर मिलती है।
  • देवप्रयाग : यहाँ भागीरथी नदी अलकनंदा से आकर मिलती है।
  • गंगा का उद्गम : उत्तराखण्ड के संतोपथ हिमानी से अलकनंदा नदी निकलती हैं तथा गोमुख (गंगोत्री) हिमानी से भागीरथी नदी निकलती है। आगे चलकर यह दोनों नदी उत्तराखण्ड के देवप्रयाग में आपस में मिल जाती हैं तत्पश्चात् ही अलकनंदा और भागीरथी की संयुक्त धारा को गंगा कहते हैं। गंगा शुरूआती दौर में उत्तराखण्ड के पर्वतीय भाग में बहती है। हरिद्वार के बाद गंगा पर्वतीय भाग से मैदानी भाग में प्रवेश करती है। उत्तराखण्ड के पश्चात् गंगा नदी बिजनौर जिला से उत्तर प्रदेश में प्रवेश करती है। गंगा नदी उत्तर प्रदेश के 28 जिलों से होकर बहती है। गंगा नदी के तट पर बसा हुआ सबसे बड़ा शहर कानपुर है। गंगा नदी बलिया जिला के बाद बिहार में प्रवेश करती है। बिहार में यह नदी बक्सर से प्रवेश करती है। बिहार के 12 जिला से होकर यह नदी गुजरती है जो निम्न हैं- 1. बक्सर, 2. भोजपुर ( आरा), 3. सारण, 4. पटना, 5. वैशाली, 6. समस्तीपुर, 7. बेगुसराय, 8 लखीसराय, 9. मुंगेर, 10. खगड़िया, 11. भागलपुर, 12. कटिहार।
    • गंगा बिहार के लगभग बीचोंबीच से होकर गुजरती हैं तथा बिहार को लगभग दो बराबर भाग उत्तरी बिहार और दक्षिणी बिहार में बाँटती है। बिहार की राजधानी पटना गंगा नदी के तट पर बसा हुआ है जो दक्षिणी बिहार के अंतर्गत आता है। गंगा नदी कटिहार के बाद बिहार से बाहर निकलती है। यह नदी झारखंड के साथ सीमा बनाती हुई पश्चिम बंगाल में प्रवेश करती है। पश्चिम बंगाल में गंगा की धारा दो भागों में विभक्त हो जाती है। एक धारा पूर्व की ओर बहती है जिसे भागीरथी कहते हैं तथा दूसरी धारा दक्षिण की ओर बहती है जिसे हुगली कहते हैं। हुगली नदी के तट पर ही कोलकाता स्थित है। ब्रिटिश काल में जॉब चारनॉक ने सुतानती, कालीकाटा और गोविन्दपुर नामक तीन गाँव को मिलाकर कोलकाता शहर की स्थापना की। गंगा नदी को दो राज्य उत्तराखण्ड और पश्चिम बंगाल में भागीरथी के नाम से जाना जाता है। पश्चिम बंगाल के बाद यह नदी बांग्लादेश में प्रवेश करती है। बांग्लादेश में इसे पद्मा कहते हैं। आगे चलकर गंगा नदी बांग्लादेश में मेघना से मिल जाती है और मेघना के नाम से ही अपना जल बंगाल की खाड़ी में गिराती है।
    • राष्ट्रीय जलमार्ग संख्या-1 (NW-1 ) : इसका निर्माण गंगा नदी पर 1986 ई. में करवाया गया है। इसकी लंबाई 1620 km है। इसका विस्तार उत्तर प्रदेश के प्रयागराज से पश्चिम बंगाल के हल्दिया तक देखने को मिलता है।
    • गंगा की सहायक नदी: रामगंगा, गोमती, घाघरा, गंडक, कोसी, महानंदा इत्यादि नदी गंगा से बायाँ तट पर मिलती है, तो वही यमुना, सोन दायाँ तट पर मिलती है।
    • यमुना : यह गंगा की सबसे बड़ी सहायक नदी है। इसका उद्गम स्थल उत्तराखंड के यमुनोत्री हिमानी है जबकि इसका संगम या मुहाना गंगा है। यह नदी गंगा में प्रयागराज में मिलती है। गंगा और यमुना के संगम पर ही प्रयागराज नामक शहर बसा हुआ है। प्रयागराज को पहले इलाहाबाद कहा जाता था। इलाहाबाद नामक शहर को मुगल बादशाह अकबर ने स्थापित किया था। यमुना की लंबाई 1326km है। यमुना को प्रदूषित होने के कारण खुला नाला की भी संज्ञा दी जाती है। यमुना नदी के तट पर दिल्ली, आगरा, मथुरा, ओरैया, इटावा इत्यादि शहर स्थित हैं।
    • सोन : यह नदी मध्य प्रदेश के अमरकंटक पहाड़ी से निकलती है तथा उत्तर-पूर्व की ओर बहती हुई पटना के निकट दानापुर में गंगा में मिल जाती है। सोन नदी गंगा नदी में दक्षिणी भाग से मिलती है।
    • रामगंगा नदी : यह नदी उत्तराखण्ड के गढ़वाल पहाड़ी से तथा चलकर यह नदी उत्तर प्रदेश में कन्नौज के समीप गंगा से मिल जाती है। रामगंगा नदी भारत के प्रथम राष्ट्रीय उद्यान जिम कार्बेट राष्ट्रीय उद्यान से गुजरती है। यह राष्ट्रीय उद्यान 1935 ई० में स्थापित हुआ है। इसे हेली नेशनल पार्क के नाम से जाना जाता था। वर्तमान में रामगंगा नेशनल पार्क है।
    • गोमती नदी : यह नदी उत्तर प्रदेश के पीलीभीत जिला के फुल्हर झील से निकलती है तथा आगे चलकर यह लखनऊ से गुजरती हुई गाजीपुर के समीप गंगा से मिल जाती है। लखनऊ गोमती नदी के तट पर बसा हुआ है।
    • घाघरा नदी : यह तिब्बत के पठार के मापचा चुंग हिमनद से निकलती है और आगे चलकर यह बिहार के छपरा में गंगा से मिल जाती है।
    • गंडक नदी : यह नदी नेपाल हिमालय से निकलती है आगे चलकर यह नदी सोनपुर के निकट गंगा से मिल जाती है। गंडक नदी से ही त्रिवेणी नहर निकलता है । अर्थात् त्रिवेणी नहर को जल की प्राप्ति गंडक नदी से ही होती है।
    • कोसी नदी : नेपाल के गोसाईंस्थान चोटी से यह नदी निकलती है। कोसी नदी की मुख्य धारा को अरूणा के नाम से जाना जाता है। कोसी नदी कटिहार के कुरसेला के समीप गंगा से मिल जाती है। विश्व में सबसे ज्यादा मार्ग परिवर्तन करने वाली नदी कोसी नदी है। कोसी नदी के जलस्तर में वृद्धि होने के कारण ही प्रत्येक वर्ष उत्तरी बिहार बाढ़ के चपेट में आता है। बाढ़ के कारण भारी जानमाल की क्षति होती है। इसलिए कोसी नदी को बिहार का शोक कहा जाता है।
    • महानंदा नदी : यह नदी पश्चिम बंगाल के दार्जिलिंग पहाड़ी से निकलती है और आगे चलकर फरक्का के समीप गंगा में मिल जाती है।
    • ब्रह्मपुत्र नदी : ब्रह्मपुत्र नदी तिब्बत के मानसरोवर झील के चेमयुंग दुंग हिमनद से निकलती है तथा पूर्व की ओर बहती हुई अरूणाचल प्रदेश के बाद यह असम में बहती हुई बांग्लादेश में प्रवेश करती है। बांग्लादेश में ही यह अपना जल मेघना के नाम से बंगाल की खाड़ी में गिराती है। इस नदी की लंबाई 2900 km है। भारत में इसकी लंबाई 916 km है। इस नदी को अलग-अलग दोनों में अलग-अलग नामों से जाना जाता है। जैसे-तिब्बत में इसे सांग्पो, अरूणाचल प्रदेश में दिहांग या दिबांग, असम में ब्रह्मपुत्र तथा बांग्लादेश में जमुना कहा जाता है।
    • राष्ट्रीय जलमार्ग संख्या-2 : इसका विस्तार असम में ब्रह्मपुत्र नदी पर सदिया से धुबरी के बीच है। इसकी लंबाई 891 km है। इसका निर्माण 1988 ई० में किया गया है।
    • सिंधु नदी तंत्र : यह नदी तिब्बत के मानसरोवर झील के सिंगी खंमान हिमनद से निकलती है तथा पश्चिम की ओर बहती हुई "आरत के एकमात्र केन्द्रशासित प्रदेश लद्दाख से होते हुए पाकिस्तान में प्रवेश करती है और आगे चलकर यह नदी अपना जल अरब सागर में गिराती है। इसकी लंबाई 2880 km है। सिंधु नदी को पाकिस्तान की जीवन रेखा कहा जाता है। सिंधु की कई सारी सहायक नदियाँ हैं जैसे-झेलम, चिनाब, रावी, व्यास, सतलज इत्यादि है। इन सहायक नदियों में व्यास एक ऐसी सहायक नदी हैं जो सिर्फ भारत में बहती है। व्यास के अलावे सिंधु की अन्य सहायक नदियाँ भारत और पाकिस्तान दोनों देशों में बहती है ।
    • सिंधु नदी जल समझौता : सिंधु तथा उसकी सहायक नदियाँ झेलम, चिनाब, रावी, व्यास, सतलज इत्यादि के जल को लेकर 1960 ई० में भारत और पाकिस्तान के बीच विश्व बैंक की अध्यक्षता में समझौता हुआ यह समझौता कराची में हुआ था। इस समझौता में भारत के प्रधानमंत्री पंडित जवाहर लाल नेहरू तथा पाकिस्तान के राष्ट्रपति अयूब खाँ ने भाग लिया। इस समझौता के तहत सिंधु, झेलम और चिनाब पर पाकिस्तान का अधिकार बताया गया जबकि रावी, व्यास और सतलज पर भारत का अधिकार बताया गया। इस समझौता में यह भी निर्धारित किया गया कि सिंधु नदी के 20 जल का उपयोग भारत करेगा तो वही 80% जल का उपयोग पाकिस्तान करेगा।
    • Note : 1. सिंधु नदी लद्दाख और जॉस्कर पर्वत श्रेणी के बीच से होकर गुजरती है।
      2. झेलम, चिनाब, रावी, व्यास, सतलज इन पाँचों नदियों को पंचनद की संज्ञा दी जाती है। 
    • सिंधु की सहायक नदियाँ :
    • झेलम नदी : यह जम्मू-कश्मीर के बेरिनाग के निकट स्थित शेषनाग झील से निकलती हैं तथा जम्मू-कश्मीर के बुलर झील से होकर बहती हुई पाकिस्तान में प्रवेश करती है। यह नदी पाकिस्तान में चिनाब से जाकर मिलती है। इस नदी पर पाकिस्तान में मंगलाडैम बाँध का निर्माण किया गया है। इसका प्राचीन नाम वितास्ता है । इस नदी पर ही जम्मू-कश्मीर की राजधानी नगर श्रीनगर स्थित है। श्रीनगर मौर्य शासक अशोक के द्वारा बसाया गया था।
    • चिनाब नदी : यह हिमाचल प्रदेश के बारालाचा दर्रा से निकलती है और चलकर पाकिस्तान में सतलज से मिल जाती है। यह सिंधु की सबसे बड़ी सहायक नदी है।
    • रावी नदी : यह हिमाचल प्रदेश के रोहतांग दर्रा से निकलती है तथा आगे पाकिस्तान में चिनाब नदी से मिल जाती है।
    • सतलज नदी : यह नदी तिब्बत के पठार के मानसरोवर झील के राकसताल से निकलती है और पश्चिम की ओर बहती हुई शिपकीला दर्रा से भारत के हिमाचल प्रदेश राज्य में प्रवेश करती है। हिमाचल प्रदेश के बाद यह पंजाब में बहती हुई पाकिस्तान में प्रवेश करती है। पाकिस्तान में यह चिनाब से मिल जाती है।
    • Note : झेलम, चिनाब, रावी, व्यास, सतलज की संयुक्त जलधारा मिठानकोट के समीप पाकिस्तान में सिंधु नदी से मिलती हैं।
    • व्यास नदी : यह नदी रोहतांग दर्रा के समीप व्यासकुंड से निकलती है तथा आगे चलकर सतलज से मिल जाती हैं। यह सिंधु की एक ऐसी सहायक नदी है जो सिर्फ भारत में बहती है।
    • चिनाब नदी को हिमाचल प्रदेश में चन्द्रा तथा भागा के नाम से जाना जाता है।
    नदी उद्गम संगम
    झेलम शेषनाग चिनाब
    चिनाब बारालाचा सतलज
    रावी रोहतांग दर्रा चिनाब
    व्यास व्यासकुंड सतलज
    सतलज राकसताल चिनाब
    • प्रायद्वीपीय भारत की नदियाँ :
      1. दामोदर नदी : यह झारखंड के छोटानागपुर पठार से निकलती है आगे चलकर यह पश्चिम बंगाल में हुगली से मिल जाती है। यह हुगली की सबसे बड़ी सहायक नदी है। दामोदर नदी को पश्चिम बंगाल का शोक कहा जाता है। स्वतंत्र भारत की प्रथम नदी घाटी परियोजना दामोदर नदी घाटी परियोजना है। इसका निर्माण 1948 ई० में हुआ है । यह परियोजना अमेरिका के टेनेसी नदी घाटी परियोजना के आधार पर स्थापित हुआ है। इस परियोजना के अंतर्गत दामोदर नदी पर पंचेत बाँध, बराकर नदी पर मैथन, बाला पहाड़ी बाँध, कोनारी नदी पर कोनारी बाँध का निर्माण किया गया है।
    • Note : भारत के प्रथम प्रधानमंत्री पंडित जवाहर लाल नेहरू ने नदी घाटी परियोजना ( बहुउद्देशीय परियोजना) को आधुनिक भारत के मंदिर की संज्ञा दी है।
    • स्वर्णरेखा नदी : यह छोटानागपुर के पठार से निकलती है तथा पूरब की ओर बहती हुई अपना जल बंगाल की खाड़ी में गिराती है। इस नदी पर जमशेदपुर नामक शहर बसा हुआ है। इस नदी को छोटानागपुर की गंगा कहा जाता है ।
    • महानदी : यह नदी छत्तीसगढ़ के सिंधावा पर्वत श्रेणी से निकलती है और ओडिशा राज्य में बहती हुई अपना जल बंगाल की खाड़ी में गिराती है। इसकी प्रमुख सहायक नदियाँ मांड, ईब, जोंक, तेल, शिवनाथ इत्यादि है। इस नदी पर ओडिशा राज्य में भारत का सबसे लंबा बाँध हीराकुंड बाँध का निर्माण किया गया है।
    • महानदी डेल्टा का निर्माण करती है जिसे महानदी डेल्टा कहते हैं। यह डेल्टा ओडिशा राज्य में स्थित है।
    • गोदावरी नदी : यह महाराष्ट्र के नासिक के त्रयम्बक पहाड़ी से निकलती है तथा पूरब की ओर बहती हुई महाराष्ट्र, तेलंगाना और आंध्र प्रदेश से गुजरती है तथा आंध्र प्रदेश के तट पर गोदावरी डेल्टा का निर्माण करते हुए अपना जल बंगाल की खाड़ी में गिराती है। इसकी लंबाई 1465 km है। यह दक्षिण भारत की सबसे लम्बी नदी है। इसलिए इसे दक्षिणी गंगा कहा जाता है। इसकी प्रमुख सहायक नदी पैनगंगा, वैनगंगा, वर्धा, मंजरा, इन्द्रावती, प्राणहिता इत्यादि है।
    • कृष्णा नदी : यह महाराष्ट्र में स्थित महाबालेश्वर की पहाड़ी से निकलती है तथा पूरब की ओर बहती हुई महाराष्ट्र, कर्नाटक, तेलंगाना, आंध्र प्रदेश होते हुए अपना जल बंगाल की खाड़ी में गिराती है। यह नदी बंगाल की खाड़ी में जल गिराने से पूर्व आंध्र प्रदेश के तट पर कृष्णा नदी डेल्टा का निर्माण करती है। इसकी प्रमुख सहायक नदी भीमा, तुंगभद्रा, घाटप्रभा, मालप्रभा इत्यादि है। इसकी लंबाई 1400 km है। यह दक्षिण भारत की दूसरी सबसे लम्बी नदी है।
    • कोवरी नदी : यह कर्नाटक के ब्रह्मगिरि पहाड़ी से निकलती है तथा पूरब की ओर बहती हुई तमिलनाडु के तट पर अपना जल बंगाल की खाड़ी में गिराती है। यह नदी तमिलनाडु के तट पर कावेरी नदी डेल्टा का निर्माण करती है। इसकी लंबाई 800 km है। दक्षिण भारत के लोग इसे सबसे पवित्र नदी मानता है। जिस प्रकार उत्तर भारत में यज्ञ और धार्मिक अनुष्ठान में गंगा नदी के जल का प्रयोग किया जाता है उसी प्रकार दक्षिण भारत में यज्ञ और धार्मिक अनुष्ठान में कावेरी नदी के जल का प्रयोग करता है। पवित्रता के कारण कावेरी नदी को दक्षिण भारत की गंगा कहते हैं।
    • Note : कावेरी नदी के जल को लेकर विवाद कर्नाटक और तमिलनाडु के बीच है।
    • लूनी नदी : यह नदी राजस्थान के अरावली पर्वत श्रेणी से निकलती हैं तथा गुजरात में बहती हुई कच्छ के रन में लुप्त हो जाती है। यह नदी समुद्र तक नहीं पहुँच पाती हैं इसे रेगिस्तान की गंगा भी कहते हैं। इसकी प्रमुख सहायक नदी सरसुती, जुबई इत्यादि है। इसे लवण नदी भी कहा जाता है।
    • साबरमती नदी : यह नदी राजस्थान के अरावली पहाड़ी से निकलती है तथा गुजरात से होकर बहती हुई अपना जल खंभात की खाड़ी में गिराती है। इस नदी के तट पर ही अहमदाबाद शहर है। अहमदाबाद में ही महात्मा गाँधी ने 1916 ई॰ में साबरमती आश्रम की स्थापना की, जिसे सत्याग्रह आश्रम भी कहा जाता है।
    • नर्मदा नदी : यह नदी विन्ध्याचल और सतपुड़ा पर्वत श्रेणी के मध्य में स्थित अमरकंटक की पहाड़ी से निकलती है तथा इन दोनों पर्वत श्रेणियों के मध्य से होकर गुजरती हुई गुजरात राज्य के तट पर अपना जल खंभात की खाड़ी में गिराती है। इस नदी की लंबाई 1312 km है। यह नदी अपने उद्गम सील से पश्चिम की ओर बहती है। इसकी प्रमुख सहायक नदी ओरसंग, तवा, बंजर, शेर इतयादि है।
    • Note : अमरकंटक की पहाड़ी से निकलने वाली नमंदा और सोन नदी की धारा ठीक एक-दूसरे के विपरीत हैं जहाँ एक तरफ नर्मदा नदी पश्चिम की ओर बहती है तो वह दूसरी तरफ सोन नदी उत्तर - पूर्व की ओर बहती है।
    • ताप्ती नदी : यह नदी सतपुड़ा पर्वत श्रेणी के दक्षिणी भाग से होकर गुजरती है। यह नदी मध्य प्रदेश के बैतूल जिले के महादेव पहाड़ी से निकलती है तथा पश्चिम की ओर बहती हुई अपना जल गुजरात के तट पर खंभात की खाड़ी में गिराती है। इसकी लंबाई 730 km है। इसकी प्रमुख सहायक नदी अरुणावती, मूरना, गिरना इत्यादि है।
    • Note : 1. नर्मदा और ताप्ती प्रायद्वीपीय भारत की ऐसी नदियाँ हैं जो डेल्टा नहीं बनाती हैं बल्कि ये नदियाँ एश्चुअरी का निर्माण करती हैं।
      2. सतपुड़ा के उत्तर से होकर नर्मदा नदी बहती है तो वही सतपुड़ा के दक्षिण से होकर ताप्ती नदी बहती है।
    • माही नदी : यह मध्य प्रदेश के विन्ध्याचल पर्वत श्रेणी में स्थित मेहंद झील से निकलती है तथा कर्क रेखा को दो बार काटती हुई अपना जल खंभात की खाड़ी में गिराती है।
    • पेरियार नदी : यह नदी अन्नामलाई की पहाड़ी से निकलती है तथा अपना जल अरब सागर में गिराती है। यह केरल की सबसे लंबी नदी है।
    • मांडबा नदी -: यह नदी कर्नाटक के बेलगाँव से निकलती है तथा गोवा में बहती हुई अपना जल अरब सागर में गिराती है। गोवा- की राजधानी नगर पणजी मांडवी नदी के तट पर स्थित है।
    • शरावती नदी : यह कर्नाटक के शिमोगा जिले से निकलती हैं तथा अपना जल अरब सागर में गिराती है। यह कर्नाटक की एक प्रमुख नदी हैं। इस नदी पर ही जोग या गरसोप्पा जलप्रपात है।
    • बहुउद्देशीय परियोजना : नदियों के ऊपर बाँध बनाकर एक से अ अधिक 3 उद्देश्य जैसे- बाढ़ नियंत्रण, विद्युत उत्पादन, सिंचाई सुविधा उपलब्ध करवाया जाना इत्यादि उद्देश्यों की पूर्ति की जाती है जिसे बहुउद्देशीय परियोजना कहा जाता है। भारत के प्रथम प्रधानमंत्री पंडित जवाहर लाल नेहरू ने बहुउद्देशीय परियोजना को आधुनिक भारत के मंदिर की संज्ञा दी है।
    • दामोदर नदी घाटी परियोजना : यह स्वतंत्र भारत की प्रथम नदी घाटी परियोजना है। इसका निर्माण 1948 ई० में अमेरिका के टेनेसी नदी घाटी परियोजना के आधार पर किया गया है। इस परियोजना के अंतर्गत दामोदर नदी पर पंचेत बाँध बराकर नदी पर मैथन और बाला पहाड़ी बाँध और तिलैया बाँध तथा कोनारी नदी पर कोनारी बाँध का निर्माण किया गया है।
    • सलाल परियोजना : इस परियोजना का निर्माण जम्मू-कश्मीर में चिनाब नदी पर किया गया है।
    • बगलिहार परियोजना : इस परियोजना का निर्माण जम्मू-कश्मीर में चिनाब नदी पर किया गया है।
    • दुलहस्ती परियोजना : इस परियोजना का निर्माण जम्मू-कश्मीर में चिनाब नदी पर किया गया है।
    • तुलबुल परियोजना और ऊरी परियोजना : इस परियोजना का निर्माण झेलम नदी पर किया गया है।
    • थीन सागर परियोजना और रंजीत सागर परियोजना : इस परियोजना का निर्माण पंजाब में रावी नदी पर किया गया है।
    • पोंग परियोजना और मंडी परियोजना : इस परियोजना कापल नि प्रदेश में व्यास नदी पर किया गया है।
    • चमेरा बाँध परियोजना : यह परियोजना रावी नदी पर हिमाचल प्रदेश राज्य में स्थित है।
    • नाथपा झाकरी परियोजना : यह परियोजना हिमाचल प्रदेश राज्य में सतलज नदी पर निर्मित हैं।
    • भांखड़ा नांगल परियोजना : यह परियोजना सतलज नदी पर पंजाब में स्थित हैं। यह देश की सबसे बड़ी बहुउद्देशीय परियोजना है इसमें लाभांवित होने वाले राज्य पंजाब, हरियाणा, हिमाचल प्रदेश, दिल्ली और राजस्थान है।
    • इंदिरा गाँधी नहर : यह देश की सबसे लम्बी नहर है ! इस नहर की लम्बाई 648 km है | यह नहर सतलज और व्यास के संगम पर स्थित हरिके बैराज से निकलती है तथा मुख्य तौर पर राजस्थान राज्य में बहती है। इस नहर से सर्वाधिक लाभांवित राज्य राजस्थान होता है जिस कारण इसे राजस्थान नहर भी कहा जाता है ! राजस्थान के मरूस्थलीय इलाका में इस नहर की उपस्थिति के कारण ही हरियाली देखने को मिलती है जिस कारण इस नहर को मरूगंगा भी कहा जाता है। इस नहर की नींव 1958 ई० में भारत के गृहमंत्री गोविन्द वल्लभ पंत के द्वारा रखा गया था। उत्तर प्रदेश के प्रथम मुख्यमंत्री गोविन्द वल्लभ पंत थे।
    • नर्मदा घाटी परियोजना : इस परियोजना के अंतर्गत मध्य प्रदेश राज्य में नर्मदा सागर बाँध का निर्माण किया गया है तो वही गुजरात राज्य में नर्मदा नदी पर सरदार सरोवर बाँध का निर्माण किया गया है। नर्मदा घाटी परियोजना से सर्वाधिक लाभ प्राप्त करने वाला राज्य गुजरात है तो वही सर्वाधिक दुष्प्रभावित राज्य मध्य प्रदेश है। इस परियोजना से लाभांवित होने वाले प्रमुख राज्य मध्य प्रदेश, गुजरात, महाराष्ट्र, राजस्थान है।
    • Note : नर्मदा नदी दो बच्चों से होकर गुजरती है - 1. मध्य प्रदेश, 2. गुजरात ।
    • उकाई परियोजना : यह गुजरात राज्य में ताप्ती नदी पर स्थित है।
    • काकरपाड़ा परियोजना : यह गुजरात राज्य में ताप्ती नदी पर स्थित है।
    • Note : ताप्ती नदी तीन राज्य से होकर गुजरती है - 1. मध्य प्रदेश, 2. महाराष्ट्र, 3. गुजरात।
    • टिहरी बाँध परियोजना : यह परियोजना उत्तराखण्ड राज्य में भागीरथी और भीलंगना नदी के संगम पर स्थित है।
    • हीराकुंड बाँध : यह भारत तथा विश्व का सबसे लंबा मानव निर्मित बाँध है। इसकी लंबाई 25.4 km है। यह बाँध ओडिशा राज्य में महानदी के तट पर स्थित है। इस बाँध से लाभांवित होने वाला राज्य मुख्यत: ओडिशा और छत्तीसगढ़ है। इसके अलावे झारखंड और मध्य प्रदेश है।
    • रिहंद परियोजना : यह परियोजना उत्तर प्रदेश राज्य में रिहंद नदी पर स्थित है। इस परियोजना के ठीक पीछे रिहंद नदी पर एक मानव निर्मित झील बनाया गया है जिसे गोविन्द वल्लभ पंत सागर झील कहते हैं।
    • Note : पंजाब में सतलज नदी पर भांखड़ा नांगल के ठीक पीछे गोविन्द सागर झील स्थित है। ये भी मानव निर्मित झील है।
    • फरक्का बैराज : यह बाँध पश्चिम बंगाल राज्य में गंगा नदी के तट पर स्थित है। इससे लाभांवित होने वाले राज्य पश्चिम बंगाल और झारखण्ड है।
    • रेणुका बाँध परियोजना : यह हिमाचल प्रदेश राज्य में गिरी नदी के तट पर स्थित है।
    • कोलडैम बाँध परियोजना : यह परियोजना हिमाचल प्रदेश राज्य में  सतलज नदी के तट पर स्थित है।
    • काल्पोंग परियोजना : यह काल्पोंग नदी पर अंडमान निकोबार द्वीप समूह में स्थित है।
    • कोसी परियोजना : यह कोसी नदी पर स्थित है। इस परियोजना से लाभांवित होने वाले क्षेत्र नेपाल और बिहार है। मूलत: कोसी नदी पर बाँध का निर्माण बाढ़ को नियंत्रित करने के लिए किया गया है।
    • गंडक परियोजना : यह गंडक नदी पर स्थित है। यह बिहार और उत्तर प्रदेश की संयुक्त परियोजना है।
    • कोयना परियोजना : यह परियोजना कोयना नदी पर महाराष्ट्र राज्य में स्थित है।
    • Note : महाराष्ट्र के महाबलेश्वर पहाड़ी से मूलत: पाँच नदियाँ निकलती है और महाराष्ट्र राज्य में बहती हैं जो निम्न हैं- 1. कृष्णा, 2. कोयना, 3. वेन्ना, 4. सावित्री, 5. गायत्री ।
    • तुंगभद्रा परियोजना : यह परियोजना कर्नाटक राज्य में तुंगभद्रा नदी पर स्थित है।
    • घाटप्रभा परियोजना : यह परियोजना कर्नाटक राज्य में घाटप्रभा नदी पर स्थित है।
    • चंबल परियोजना : यह परियोजना चंबल नदी पर स्थित है। इससे लाभांवित वाला राज्य मध्य प्रदेश और राजस्थान है।
    • Note : चंबल नदी पर मध्य प्रदेश राज्य में गाँधी सागर परियोजना स्थित है तो वही राजस्थान राज्य में चंबल नदी पर राणा प्रताप सागर परियोजना, जवाहर सागर परियोजना और कोटा बैराज परियोजना स्थित हैं।
    • कृष्ण सागर बाँध कावेरी नदी पर स्थित है।
    • मयूराक्षी परियोजना : यह मयूराक्षी नदी पर पश्चिम बंगाल में स्थित है। इससे लाभांवित होने वाले राज्य पश्चिम बंगाल और झारखंड है।
    • माताटीला परियोजना : यह परियोजना बेतवा नदी पर स्थित है। इससे लाभांवित होने वाले राज्य मध्य प्रदेश और उत्तर प्रदेश है।
    • शिवसमुद्रम परियोजना : यह कावेरी नदी पर है। इससे लाभांवित होने वाला राज्य मुख्यतः तमिलनाडु है ।
    • Note : श्री नागार्जुन सागर, श्री सेलम तथा तेलुगु गंगा परियोजना कृष्णा नदी पर स्थित है। इससे लाभांवित होने वाले राज्य मुख्यतः आंध्र प्रदेश और तेलंगाना है।
    • मेट्टूर बाँध परियोजना : महाराष्ट्र राज्य में गोदावरी नदी के तट पर जायकावाड़ी और पोचम्पाद परियोजना स्थित है।
    • इडुक्की परियोजना : यह केरल राज्य में पेरियार नदी पर स्थित है।
    • मेटुर बाँध कावेरी नदी पर तमिलनाडु राज्य में स्थित है।
    • ब्रह्मपुत्र नदी नामचाबारबा में U-टर्न लेती है और भारत में प्रविष्ट होती है।
    • भारत की सबसे लंबी नदी गोदावरी है जिसका उद्गम तथा मुहाना दोनों भारतीय क्षेत्र में अवस्थित है।
    • दामोदर नदी अपने भ्रंश घाटी प्रवाह के लिए प्रसिद्ध है।
    • अमरावती कृष्णा की सहायक नदी नहीं है। (कावेरी का )
    • लूनी नदी के ऊपरी मार्ग में मीठा पानी परंतु निचले भाग में खारे जल का प्रवाह मिलता है।
    • काली नदी धांधर, साबरमती पश्चिम की ओर बहने वाली नदी है ।
    • वैतरणी पश्चिम की ओर बहने वाली नदी है।

    भारत के झील

    • झील : चारों तरफ से स्थलीय भाग से घिरे हुए जल के शांत क्षेत्र को झील कहते हैं। झील कई प्रकार के होते हैं जो निम्न हैं-
      1. अनुप झील या लैगून झील : सामान्यतः लैगुन झील समुद्र तटीय भाग में पाया जाता है। मुख्य समुद्री जल से जल का कुछ भाग बालू, कंकड़, पत्थर की उपस्थिति के कारण जब अलग होता है तो उससे जिस झील का निर्माण होता है उसे अनुप या लैगून झील कहते हैं।
    • कुछ महत्वपूर्ण लैगून झील :
    • चिल्का झील : यह एक लैगून झील है, जो ओडिशा राज्य में समुद्र तट पर स्थित है। यह झील झींगा मछली उत्पादन के लिए प्रसिद्ध है। इस झील में कई सारे द्वीप हैं। जैसे- नालावान द्वीप।
    • पुलिकट झील : यह एक लैगून झील है, जो आंध्र प्रदेश और तमिलनाडु राज्य के सीमा पर स्थित है। श्री हरिकोटा द्वीप इस झील को बंगाल की खाड़ी अलग करता है।
    • वेम्बनाड झील : यह एक लैगून झील है। यह केरल में स्थित है। यह भारत की सबसे लम्बी झील है । इस झील में कई सारे द्वीप हैं। जैसे- वेलिंगटन द्वीप, बल्कारपदम द्वीप इत्यादि है। इस झील में नौकायन प्रतियोगिता का आयोजन होता है।
    • अष्टमुडी झील : यह एक लैगून झील है। यह केरल राज्य में स्थित एक लैगून झील है।
    • भारत की मीठे पानी की सबसे बड़ी झील बुलर झील है जो जम्मू-कश्मीर राज्य में स्थित है।
    • भारत की खारे पानी की सबसे बड़ी झील सांभर है जो कि राजस्थान राज्य में स्थित है।
    • भारत के सबसे अधिक ऊँचाई पर स्थित झील चोलामु है जो सिक्किम राज्य में स्थित है।
    • भारत की सबसे बड़ी समुद्र तटीय झील चिल्का झील है जो खारे पानी की झील है।
    • भारत की सबसे बड़ी मानव निर्मित झील गोविंद वल्लभ पंत सागर झील है उत्तर प्रदेश में रिहंद नदी के तट पर है।

    मानव निर्मित झील

    झील राज्य नदी
    गोविन्द सागर झील हिमाचल प्रदेश और पंजाब के सीमा पर सतलज नदी
    गाँधी सागर झील मध्य प्रदेश चंबल नदी
    राणा प्रताप सागर झील और जवाहर सागर झील राजस्थान चंबल नदी
    नागार्जुन सागर झील आंध्र प्रदेश कृष्णा नदी
    स्टेनली झील तमिलनाडु कावेरी नदी
    पेरियार झील केरल पेरियार नदी

    भारत के महत्त्वपूर्ण झील

    राज्य झील
    असम दिपोर बिल झील, सोनबिल झील
    मेघालय उमियम झील
    मिजोरम पाला झील
    मणिपुर लोकटक झील
    लद्दाख पागौग सो तथा सोमोरिरि झील लद्दाख में स्थित है।
    जम्मू-कश्मीर डल झील, बुलर झील, नागिन झील, शेषनाग झील, बेरिनाग झील, मानसबब झील इत्यादि ।
    उत्तराखण्ड भीमताल, नैनीताल, कुछियाताल, खुरपाताल, रूपकुण्ड झील इत्यादि ।
    पंजाब कांजलि झील, रोपड़ तथा हरिकेय झील
    हरियाणा सूरजकुण्ड झील
    हिमाचल प्रदेश रेणुका झील, सूरजताल, चन्द्रताल
    राजस्थान सांभर, डीडवाना, राजसमद, जयसमंद, पुष्कर झील
    गुजरात नल सरोवर, नारायण सरोवर झील
    तेलंगाना हुसैन सागर, उस्मान सागर, दस्मान सागर झील
    महाराष्ट्र लोनार झील, सलीम अली सरोवर झील
    मध्य प्रदेश तवा झील, भोंज झील
    आंध्र प्रदेश कोलेरू झील
    तमिलनाडु उटी और कोडइकनाल झील
    अरूणाचल प्रदेश साला झील
    हरियाणा बड़खल झील
    तमिलनाडु कालीवेला झील
    • लोकटक : लोकटक मणिपुर में स्थित एक मीठे जल की झील है जो पूर्वोत्तर भारत के सबसे बड़ी झील है। विश्व का एकमात्र तैरता हुआ राष्ट्रीय उद्यान केबुल लामजाओ राष्ट्रीय उद्यान है जो मणिपुर के लोकटक झील में स्थित है।
    • केबुल लामजाओ एकमात्र राष्ट्रीय उद्यान है जहाँ संगाई हिरण पायी जाती है। संगाई हिरण संकटग्रस्त प्राणी है जो लगभग विलुप्त होने के कगार पर है।
    • रेड डाटा बुक : रेड डाटा बुक की संकल्पना सर पीटर स्कॉट ने किया था। पहली बार रेड डाटा बुक 1964 में प्रकाशित हुआ था। इस बुक में विलुप्तप्राय या संकटग्रस्त जानवरों को शामिल किया जाता है।
    • गुजरात में सुदर्शन झील वस्तुतः एक - कृत्रिम जलाशय हैं, जो मौर्यों के शासन के दौरान बनाया गया था।
    • 'सास्थामकोट्टा' झील केरल राज्य में स्थित है।

    भारत के जलप्रपात

    • पहाड़ी पठारी क्षेत्रों में ऊँचाई से तीव्र गति से नीचे गिरते हुए जल को जलप्रपात कहते हैं।
    • विश्व की सबसे ऊँची जलप्रपात एंजिल है जो वेनेजुएला में कोरोनी नदी पर स्थित है। ( ऊँचाई 979 m)
    • भारत की सबसे ऊँची जलप्रपात कर्नाटक राज्य में कुचीकल है जिसकी ऊँचाई 455m है। यह जलप्रपात बराही नदी पर है।
    जलप्रपात नदी राज्य
    चुलिया चंबल राजस्थान
    दुग्धधारा, कपिलधारा, धुआँधार जलप्रपात नर्मदा मध्य प्रदेश
    चचाई जलप्रपात बिहर नदी मध्य प्रदेश
    दूध सागर जलप्रपात मांडवी गोवा
    जोग या गरसोप्पा जलप्रपात शरावती नदी कर्नाटक (255m )
    गोकक जलप्रपात घाटप्रभा नदी कर्नाटक
    हुंडरू जलप्रपात स्वर्णरेखा नदी झारखंड
    डुडुमा जलप्रपात मुचकुंड नदी ओडिशा
    चित्रकोट जलप्रपात इन्द्रावती नदी छत्तीसगढ़
    साडनी जलप्रपात शंख नदी झारखंड
    गौतमधारा जलप्रपात रारु नदी झारखंड
    दसम जलप्रपात काँची नदी झारखंड
    ककोलत जलप्रपात सराही नदी नवादा (बिहार)
    • भारत का सबसे बड़ा जलप्रपात हुंडरू है।
    • भारत का नियाग्रा चित्रकोट जलप्रपात को कहा जाता है।
    • नियाग्रा एक जलप्रपात है जो USA और कनाडा की सीमा पर सेंट लॉरेंस नदी पर स्थित है।
    • जोग या गरसोप्पा जलप्रपात का नया नाम महात्मा गाँधी जलप्रपात है ।

    नदी जल विवाद

    • भारतीय संविधान के अनुच्छेद 262 अंतर्राज्यीय नदी जल विवाद से संबंधित प्रावधान की चर्चा करता है।
      1. कावेरी नदी जल विवाद इस नदी के जल को लेकर विवाद मुख्यतः कर्नाटक और तमिलनाडु राज्य के बीच है।
    • नर्मदा नदी जल विवाद : इस नदी के जल को लेकर विवाद मुख्यतः मध्य प्रदेश और गुजरात के बीच है।
    • कृष्णा नदी जल विवाद : इस नदी के जल को लेकर विवाद मुख्यतः कर्नाटक और आंध्र प्रदेश राज्य के बीच है।
    • सोन नदी जल विवाद : इस नदी के जल को लेकर विवाद मुख्यतः बिहार, मध्य प्रदेश और उत्तर प्रदेश के बीच है।
    • कुचीकल जलप्रपात की ऊँचाई 1493 फीट है।

    वस्तुनिष्ठ प्रश्न एवं उत्तर

    1. अपवाह का क्या अर्थ है ?
    (a) समुद्र के किनारे का बंदरगाह
    (b) नदी-पत्तन
    (c) नदी और अपवहन तंत्र
    (d) नदीघाटी
    2. इनमें कौन-सा कथन सही है ?
    (a) ढाल के अनुरूप गमन करनेवाली नदी को अनुवर्ती नदी कहते हैं।
    (b) भारत की कुछ नदियाँ ढाल के अनुरूप नहीं हैं।
    (c) भारत की कुछ नदियाँ पहाड़ के आर-पार बहती हैं।
    (d) उपर्युक्त सभी कथन सही हैं।
    3. दो नदी - द्रोणियों को अलग करनेवाली उच्चभूमि को क्या कहते हैं?
    (a) पहाड़
    (b) पठार
    (c) जलविभाजक
    (d) चरागाह
    4. इनमें कौन-सी नदी पूर्ववर्ती है ?
    (a) महानदी 
    (b) हगली
    (c) चंबल
    (d) सिंधु
    5. हिमालय में सबसे गहरी नदीघाटी (5,100 मीटर) कहाँ मिलती है ?
    (a) शिमला के निकट
    (b) गिलगिट के निकट
    (c) गंगोत्री के निकट
    (d) नामचा बारवा के निकट
    6. गंगा की सबसे बड़ी सहायक नदी कौन है ?
    (a) यमुना 
    (b) गंडक
    (c) कोसी
    (d) सरयू
    7. इनमें कौन-सी नदी भ्रंशघाटी में बहती है ?
    (a) गोदावरी 
    (b) नर्मदा
    (c) कृष्णा
    (d) कावेरी
    8. इनमें कौन-सी नदी अपने मुहाने पर डेल्टा नहीं बनाती है?
    (a) कावेरी 
    (b) कृष्णा
    (c) गोदावरी
    (d) नर्मदा
    9. निम्नांकित में कौन-सी नदी अरब सागर में गिरती है ?
    (a) गोदावरी 
    (b) कृष्णा
    (c) नर्मदा
    (d) कावेरी
    10. लूनी किसकी सहायक नदी है ?
    (a) गंगा की 
    (b) यमुना की
    (c) चंबल की
    (d) इनमें किसी की नहीं
    11. इनमें कौन झील जम्मू और कश्मीर में है ?
    (a) चिल्का 
    (b) कोलेरू
    (c) पुलीकट
    (d) डल
    12. लोनार झील कहाँ अवस्थित है?
    (a) राजस्थान में
    (b) मध्य प्रदेश में 
    (c) महाराष्ट्र में
    (d) कर्नाटक में
    13. इनमें कौन-सी नदी प्रायद्वीपीय भारत की सबसे लंबी नदी है?
    (a) महानदी
    (b) कृष्णा
    (c) तापी
    (d) गोदावरी
    14. इनमें कौन गंगा की सहायक नदी नहीं है?
    (a) बेतवा 
    (b) सोन
    (c) लोहित
    (d) चंबल
    15. निम्न कथनों पर विचार कीजिए तथा नीचे दिए गए कूट की सहायता से सही उत्तर का चयन कीजिए:
    1. भारत में भाकड़ा ( Bhakra) सबसे ऊंचा बांध है।
    2. भारत में चिल्का झील सबसे बड़ी खारे पानी (Brakish Water) की झील है।
    3. भारत में लोकटक सबसे बड़ी मीठे पानी (Fresh Water ) की झील है।
    कूट :
    (a) 1 तथा 2 सही हैं।
    (b) 1 तथा 3 सही हैं।
    (c) 2 तथा 3 सही हैं।
    (d) 1, 2 तथा 3 सही हैं।
    16. निम्न में कौन-सी नदी सबसे लंबी है ?
    (a) चिनाब
    (b) झेलम
    (c) रावी
    (d) सतलूज
    17. निम्न में से कौन-सा जल प्रपात तमिलनाडु में नहीं है ?
    (a) अय्यानार (Ayyanar)
    (b) कुर्तालम (Kurtalam)
    (c) लोढ़ (Lodh)
    (d) किलियर (Kiliyur)   
    18. निम्न में से कौन-सा जल प्रपात कर्नाटक में नहीं है? 
    (a) इरूपू (Erupu) 
    (b) गोकाक (Gokak )
    (c) हुण्डरू (Hundru )
    (d) जोग (Jog)
    19. निम्न में से किस झील से नमक का अधिक उत्पादन होता है ?
    (a) चिल्का (Chilka)
    (b) कोल्लेरू (Kolleru)
    (c) पुलीकट (Pulicat)
    (d) साम्भर ( Sambhar)
    20. निम्न में से कौन-सा जल प्रपात सबसे ऊंचा है? 
    (a) बर्काना (Barkana) 
    (b) कन्डाधार (Kandadhar)
    (c) लोढ़ (Lodh)
    (d) बुन्डला (Bundla)
    21. निम्न कथनों पर विचार कीजिए तथा नीचे दिए गए कूट की सहायता से सही उत्तर का चयन कीजिए:
    1. झीलें स्थिर जल का संचय होती हैं।
    2. झीलें चारों ओर थल से अवरुद्ध होती हैं।
    3. कैस्पियन सागर विश्व की सबसे बड़ी झील है।
    4. चिल्का झील भारत की सबसे बड़ी झील है।
    कूट:
    (a) 1, 2 तथा 3 सही हैं । 
    (b) 1, 2 तथा 4 सही हैं।
    (c) 1, 3 तथा 4 सही हैं ।
    (d) 1, 2, 3 तथा 4 सही हैं।
    22. इनमें से कौन-सी नदी पूर्व से पश्चिम दिशा में बहती है ? 
    (a) दिबांग 
    (b) लोहित
    (c) मानस
    (d) सुबानसीरी
    23. निम्नांकित कथनों पर विचार कीजिए:
    1. रूपनारायण पश्चिम बंगाल की एक नदी है
    2. रूपनारायण झारखंड की पहाड़ियों से निकलती है
    3. रूपनारायण रारह मैदानी हिस्से से गुजरती है
    उपरोक्त में से कौन-सा कथन सही है ?
    (a) केवल 1 
    (b) 2 और 3
    (c) 1 और 3
    (d) 1, 2 और 3
    24. निम्न नदियों को पश्चिम से पूर्व के क्रम में सजाएं और नीचे दिए कूट की सहायता से सही उत्तर का चयन कीजिए:
    1. बागमती (Bagmati )
    2. गंडक ( Gandak)
    3. घाघरा (Ghagra
    4. राप्ती ( Rapti ).
    कूट:
    (a) 3, 4, 2 और 1
    (b) 3, 4, 1 और 2
    (c) 1, 2, 4 और 3
    (d) 1, 2, 3 और 4
    25. निम्नांकित नदियों पर विचार कीजिए:
    1. बनास
    2. काली सिंध
    3. पार्वती
    4. सिंध (Sindh)
    इन नदियों को पश्चिम से पूर्व के क्रम में सजाएं और नीचे दिए गए कूट की सहायता से सही उत्तर का चयन कीजिए:
    (a) 1, 2, 3 और 4
    (b) 1,3, 2 और 4
    (c) 3, 1, 4 और 2
    (d) 3, 4, 1 और 2
    26. नर्मदा पश्चिम की ओर बहती है, जबकि अधिकतर प्रायद्वीपीय नदी पूर्व की ओर बहती है, ऐसा क्यों है?
    1. यह विंध्य और सतपुड़ा के बीच बहती है
    2. यह एक रिफ्ट घाटी ( Rift Valley) से बहती है
    3. मध्य भारत के बाद धरातल का पश्चिम की ओर हो जाता है
    नीचे दिए गए कूट की सहायता से सही उत्तर का चयन कीजिए:
    (a) 2 और 3
    (b) 1 और 2
    (c) 1 और 3
    (d) केवल 2
    27. अलकनन्दा तथा भागीरथी नदियों का संगम निम्न में से किस स्थान पर है?
    (a) देव प्रयाग
    (b) कर्ण-प्रयाग
    (c) रुद्र - प्रयाग
    (d) विष्णु प्रयाग
    28. निम्न में से कौन-सी नदी, गंगा की सबसे बड़ी सहायक नदी है ?
    (a) गण्डक
    (b) घाघरा
    (c) कोसी
    (d) यमुना
    29. निम्न में से कौन-सी नदी रिफ्ट - वैली (Rift Valley) में नहीं बहती ?
    (a) गोदावरी
    (b) महानदी
    (c) नर्मदा
    (d) तापी
    हमसे जुड़ें, हमें फॉलो करे ..
    • Telegram ग्रुप ज्वाइन करे – Click Here
    • Facebook पर फॉलो करे – Click Here
    • Facebook ग्रुप ज्वाइन करे – Click Here
    • Google News ज्वाइन करे – Click Here
    ]]>
    Wed, 10 Apr 2024 05:38:56 +0530 Jaankari Rakho
    General Competition | Geography | भारत के पर्वत, पठार एवं मैदान https://m.jaankarirakho.com/952 https://m.jaankarirakho.com/952 General Competition | Geography | भारत के पर्वत, पठार एवं मैदान
    • भारत के कुल क्षेत्रफल के 10.7% भू-भाग पर पर्वत का, 18.6% भू-भाग पर पहाड़ का, 27.7% भू-भाग पर पठार का तथा 43% भू-भाग पर मैदान का विस्तार है।
    • हिमालय पर्वत : भारत के उत्तर में यह पर्वत स्थित है। हिमालय पर्वत के उत्तर में तिब्बत का पठार है तथा दक्षिण में गंगा का मैदान या उत्तर भारत का विशाल मैदान है। हिमालय पर्वत पश्चिम में सिंधु नदी से लेकर पूर्व में ब्रह्मपुत्र नदी तक 2400 km में फैला हुआ है। मुख्य तौर पर हिमालय पर्वत पश्चिम से पूर्व में फैला हुआ है लेकिन अरूणाचल प्रदेश राज्य में यह दक्षिण की ओर मुड़ जाती है। इसका विस्तार म्यांमार में भी देखने को मिलता है। म्यांमार में इसे अराकान योमा कहा जाता है। हिमालय पर्वत 2400 km में फैला हुआ है। इसका आकार लगभग धनुषाकार है । सर्वप्रथम सर सिडनी बुराई नामक विद्वान ने हिमालय पर्वत को प्रादेशिक तौर पर चार भागों में बाँटा है-
      1. पंजाब हिमालय : सिंधु नदी से लेकर सतलज नदी तक जो हिमालय फैला हैं उसे पंजाब हिमालय कहते हैं। इसकी लंबाई 560 km है। इसके अंतर्गत काराकोरम, लद्दाख, जास्कर, पीरपंजाल, धौलाधर जैसी पर्वत श्रेणियाँ आती हैं।
      2. कुमायूँ हिमालय : सतलज नदी से काली नदी तक फैली हुई हिमालय को कुमायूँ हिमालय कहते हैं। अर्थात् दूसरे शब्दों में कहे तो उत्तराखंड में फैले हुए हिमालय को कुमायूँ हिमालय कहते हैं। इसके अंतर्गत कामेत, त्रिशुल, नंदा देवी जैसी पर्वत चोटियाँ आती हैं। इसकी लंबाई 320 km है।
      3. नेपाल हिमालय : काली नदी से तीस्ता नदी तक फैले हुए हिमालय को नेपाल हिमालय कहते हैं। इसकी लंबाई 800 km है। विश्व की ऊँची पर्वत चोटियाँ नेपाल हिमालय के अंतर्गत ही आता है। जैसे- माउंट एवरेस्ट, मकालु, अन्नपूर्णा, कंचनजंघा इत्यादि।
      4. असम हिमालय : तीस्ता नदी से ब्रह्मपुत्र नदी तक फैले हिमालय को असम हिमालय कहते हैं। इसकी लंबाई 750 km है। नामचाबरवा पर्वत इसी हिमालय के अंतर्गत आता है।
    • भारतीय राज्य जिसमें हिमालय पर्वत का विस्तार हैं। भारत के कुल 11 राज्य है। 2 केन्द्रशासित प्रदेश जम्मू-कश्मीर और लद्दाख हैं जहाँ हिमालय पर्वत का स्व हैं, जहां हिमालय पर्वत कर पश्चिम बंगाल, अरूणाचल प्रदेश, असम, मेघालय, विस्तार हैं। 2 केन्द्रशासित प्रदेश में हिमालय पर्वत का विस्तार विस्तार है। हिमाचल प्रदेश, उत्तराखण्ड, सिक्किम, स्त्री मासज्य है जहाँ हिमालय पर्वत का विस्तार हैं।
    • भारत के उत्तर में जम्मू-कश्मीर तथा लद्दाख केन्द्रशासित प्रदेश में फैले हुए पर्वत श्रेणियों का उत्तर से दक्षिण में क्रम निम्न हैं- 1. काराकोरम, 2. लद्दाख 3. जास्कर, 4. पीरपंजाल, 5. धौलाधर ।
      Trick : कल जाएँगें PDS लाने ।
    • काराकोरम लद्दाख और जास्कर लद्दाख में है | पीरपंजाल पर्वत श्रेणी जम्मू-कश्मीर में तथा धौलाधर पर्वत श्रेणी हिमाचल प्रदेश में मुख्यतः हैं।
      उपर्युक्त या ऊपर लिखे हुए पर्वत श्रेणियों का जो उत्तर से दक्षिण में विस्तार हैं वही क्रम पूरब से पश्चिम का भी है।
    • भौगोलिक तौर पर हिमालय पर्वत को चार वर्गों में बाँटा गया है, जो निम्न हैं- 1. ट्रांस हिमालय, 2. वृहद हिमालय, 3. लघु हिमालय, 4. शिवालिक ।
    1. ट्रांस हिमालय : इसका निर्माण हिमालय के निर्माण के पूर्व हुआ हैं। भूगोलवेत्ताओं के अनुसार यह मुख्य हिमालय के अंतर्गत नहीं आता है। इसका विस्तार भारत के एकमात्र केन्द्रशासित प्रदेश लद्दाख में देखने को मिलता है तथा इसका सबसे ज्यादा विस्तार तिब्बत में देखा जाता है जिस कारण इसे तिब्बती हिमालय कहते हैं। इसके अंतर्गत काराकोरम, लद्दाख, जास्कर, कैलाश इत्यादि पर्वत श्रेणी आता है।
    • काराकोरम पर्वत श्रेणी की सबसे ऊँची पर्वत चोटी K, या गॉडविन ऑस्टिन है जो कि भारत की सबसे ऊँची चोटी है। इस चांटी की ऊँचाई 8611 km है।
    • काराकोरम पर्वत श्रेणी में ही लद्दाख में सियाचिन ग्लेशियर स्थित है जो भारत की सबसे बड़ी ग्लेशियर है। यह 76 km में फैला हुआ है।
    • बलतोरो ग्लेशियर, रेमो ग्लेशियर, बियाफो ग्लेशियर, पुनमा ग्लेशियर, हिस्पर ग्लेशियर, बतूरा ग्लेशियर इत्यादि काराकोरम पर्वत श्रेणी के अंतर्गत लद्दाख में स्थित है।
    2. वृहद हिमालय : इसे कई नाम जैसे - आंतरिक हिमालय, महान हिमालय, उच्च हिमालय, मुख्य हिमालय, हिमाद्रि इत्यादि से जाना जाता है। प्राचीन काल में हिमालय का यह भाग हमेशा बर्फ से ढँका रहता था जिस कारण इसे हिमाद्रि कहा जाता है। विश्व की ऊँची-ऊँची पर्वत चोटियाँ इस हिमालय के अंतर्गत आता है जिस कारण इसे उच्च हिमालय भी कहा जाता है। अधिकतर भूगोलवेत्ता वृहद हिमालय को मुख्य हिमालय मानता है। इस हिमालय के अंतर्गत आने वाली प्रमुख पर्वत चोटियाँ निम्न हैं-
    (i) कंचनजंघा         8598m       सिक्किम
    (ii) मकालु             8481m       नेपाल
    (iii) माउंट एवरेस्ट   8848m       नेपाल
    (iv) चोयु                8201m       नेपाल
    (v) गौरीशंकर         7145m       नेपाल
    (vi) मानासालु         8156m       नेपाल 
    (vii) अन्नपूर्णा          8078m       नेपाल
    (viii) धौलागिरि       8172m       नेपाल
    • विश्व की सबसे ऊँची पर्वत चोटी माउंट एवरेस्ट हैं जो कि मूलतः नेपाल में स्थित है। सीमा के संदर्भ में यह पर्वत नेपाल और तिब्बत के सीमा पर है। नेपाल में इसे सागरमाथा तथा तिब्बत में इसे चोमोलंगमा कहा जाता है। इसकी ऊँचाई 8848m है।
    • हिमालय में स्थित भारत की सबसे ऊँची पर्वत चोटी कंचनजंघा है जो कि सिक्किम में स्थित है। सीमा के संदर्भ में यह पर्वत सिक्किम और नेपाल के सीमा पर स्थित है। इसकी ऊंचाई 8598m है। यह विश्व की तीसरी सबसे ऊँची पर्वत चोटी है।
    • भारत की सबसे ऊँची पर्वत चोटी K2 या गॉडविन ऑस्टिन है जिसकी ऊँचाई 8611 m है। यह ट्रांस हिमालय के अंतर्गत काराकोरम पर्वत श्रेणी की पर्वत है। यह लद्दाख में स्थित है। यह भारत की ऊँची पर्वत चोटी है तथा यह विश्व की दूसरी सबसे ऊँची पर्वत चोटी है।
    • धौलाधर पर्वत श्रेणी हिमाचल प्रदेश है जबकि धौलागिरि नेपाल में है ।
    • ट्रांस हिमालय को वृहत हिमालय से ITSZ (Indus Tsangpo Sulture Zone) करता है।
    • लघु हिमालय : इसे कई नाम जैसे - हिमाचल हिमालय, मध्य हिमालय से जाना जाता है। यह वृहद हिमालय के समानांतर उसके दक्षिणी भाग में स्थित है। यह छोटे-छोटे पर्वत श्रेणियों से मिलकर बना है। इसी के अंतर्गत पीरपंजाल, धौलाधर, मसूरी, महाभारत इत्यादि पर्वत श्रेणी आते हैं।
      पीर पंजाल - जम्मू-कश्मीर
      धौलाधर - हिमाचल प्रदेश
      मसूरी - उत्तराखण्ड
      महाभारत - नेपाल
    • वृहद हिमालय और लघु हिमालय के बीच में कई घाटियाँ मिलती हैं। जैसे- पश्चिम में कश्मीर घाटी तथा पूर्व में काठमांडू घाटी, इसके अलावे और भी कई घाटियाँ जैसे- कांगड़ा घाटी, कुल्लू घाटी, लाहुल और स्फीतिघाटी |
      कश्मीर घाटी - जम्मू-कश्मीर
      कांगड़ा, कुल्लू घाटी, लाहुल स्फीति - हिमाचल प्रदेश
      काठमांडू घाटी - नेपाल 
    • लघु हिमालय के हाल पर छोटे-छोटे घास के मैदान पाये जाते हैं जिसे कश्मीर में मर्ग जैसे- गुलमर्ग, सोनमर्ग तथा उत्तराखण्ड में बुग्याल या पयार कहते हैं।
    • घाटी (Valley) : दो पहाड़ियों के बीच के गहरे भाग को घाटी कहते हैं।
    • शिवालिक हिमालय : यह हिमालय का सबसे नवीनतम (नया) तथा दक्षिणी भाग है। इसके दक्षिण में उत्तर भारत का विशाल मैदान है। इसे कई नाम जैसे - बाह्य हिमालय और निम्न हिमालय के नाम से भी जाना जाता है। लघु हिमालय और शिवालिक के बीच समतल संरचनात्मक मैदान पाया जाता है जिसे पश्चिम में दून जैसे - देहरादून तथा पूरब में द्वार जैसे- हरिद्वार कहा जाता है।
    • ट्रांस हिमालय : तिब्बत के पठार और वृहद हिमालय के बीच में है ।
    • वृहद हिमालय : ट्रांस हिमालय और लघु हिमालय के बीच में हैं। इसके अंतर्गत आने वाली पर्वत चोटियों की औसत ऊँचाई 6100m हैं। ट्रांस हिमालय को वृहद हिमालय से अलग ITSZ (Indus Tsangpo Sulture Zone) करता है।
    • लघु हिमालय : यह वृहद हिमालय और शिवालिक के बीच में स्थित हैं। इसके अंतर्गत आने वाली पर्वत चोटियाँ की औसत -ऊँचाई 3000m – 45000 m होती है। वृहद हिमालय को लघु हिमालय से अलग MCT (Main Central Thrust) करता है |
    • शिवालिक : लघु हिमालय और उत्तर भारत का विशाल मैदान के बीच शिवालिक हैं। इसके अंतर्गत आने वाले पर्वत श्रेणियों की औसत ऊँचाई 600m - 1500m होती है। शिवालिक को उत्तर भारत के विशाल मैदान से अलग HFF (Himalayan Frontiers Fault) करता है।
    • Note : लघु हिमालय को शिवालिक हिमालय से अलग MBF (Main Boundary Fault) करता है। 
    • पूर्वांचल हिमालय या उत्तरी पूर्वी हिमालय : हिमालय पर्वत भारत के सबसे पूरब में स्थित अरूणाचल प्रदेश राज्य से दक्षिण की ओर मुड़ जाती है तथा भारत के उत्तर - पूर्व में स्थित राज्यों में विस्तारित होती है। उसे ही पूर्वांचल हिमालय या उत्तरी-पूर्वी हिमालय कहते हैं ।
    • पूर्वांचल हिमालय के अंतर्गत अलग-अलग पहाड़ियों का विस्तार अलग-अलग राज्यों में है जो निम्न हैं- 
      1. पश्चिम बंगाल : दार्जिलिंग पहाड़ी और टाइगर पहाड़ी पश्चिम बंगाल राज्य में स्थित है।
      2. सिक्किम: कंचनजंघा, संडकफू पर्वत, छोमोयुमो पर्वत, पौहुनरी पर्वत इत्यादि सिक्किम में स्थित है।
      3. अरूणाचल प्रदेश : दाफला, मीरी, अबोर, मिशमी और पटकाई बुम अरूणालच प्रदेश में स्थित है। इन पहाड़ियों का पश्चिम से पूर्व में यही क्रम है। अरूणाचल प्रदेश राज्य के पूर्व में स्थित पटकाई बुम पर्वत श्रेणी भारत को म्यांमार से अलग करती है। कांगटो पर्वत अरूणाचल प्रदेश में स्थित है। सीमा के संदर्भ में यह अरूणाचल प्रदेश और तिब्बत के सीमा पर स्थित है। (7090m)
      4. असम : मिकिर पहाड़ी, रेंगमा पहाड़ी, भुवन पहाड़ी इत्यादि असम में स्थित है।
      5. मेघालय : मेघालय में स्थित पहाड़ियों का पश्चिम से पूर्व क्रम निम्न हैं-
        गारो -खासी - जैनतिया, नोकरेक पर्वत, काइलस पर्वत और चेरापूँजी पर्वत मेघालय में ही स्थित है।
      6. नागालैंड : नागा पहाड़ी नागालैंड राज्य में स्थित है। नागालैंड की सबसे ऊँची पर्वत चोटी सारामती ( 3826m) है।
      7. मणिपुर : मणिपुरी पहाड़ी तथा लैमातोल पहाड़ी मणिपुर में स्थित है। 
      8. मिजोरम : मिजो पहाड़ी अर्थात् लुशाई की पहाड़ी मिजोरम में स्थित हैं। इसकी सबसे ऊँची चोटी ब्लू माउंटेन (नीला पर्वत) है।
      9. त्रिपुरा : त्रिपुरी पहाड़ी त्रिपुरा में स्थित हैं।
    • ब्रेल / बराइल : यह एक ऐसी पर्वत श्रेणी है जिसका विस्तार तीन अलग-अलग राज्य असम, नागालैंड, मणिपुर में है।
    • दक्षिण का पर्वतीय क्षेत्र :
      पश्चिमी घाट पर्वत श्रेणी : भारत के दक्षिणी पश्चिमी भाग में गुजरात से लेकर तमिलनाडु तक यह पर्वत श्रेणी 1600km में फैली हुई है। हिमालय पर्वत श्रेणी के बाद यह भारत की दूसरी सबसे लम्बी पर्वत श्रेणी है। यह भारत के छ: राज्य गुजरात, महाराष्ट्र, गोवा, कर्नाटक, केरल, तमिलनाडु राज्य में फैली हुई है। पश्चिम घाट पर्वत श्रेणी को महाराष्ट्र में सह्याद्रि कहते हैं। (कर्नाटक)
    • गुजरात : गुजरात में स्थित पहाड़ियों का उत्तर से दक्षिण में क्रम निम्न है-
      कच्छ की पहाड़ी, माण्डव की पहाड़ी, बरदा पहाड़ी, गिरनार पहाड़ी, गिर पहाड़ी ।
    • कच्छ प्रायद्वीप को काठियावाड़ प्रायद्वीप से अलग कच्छ की खाड़ी करता है।
    • कांडला बंदरगाह तथा ओखा बंदरगाह कच्छ की खाड़ी में स्थित है। कांडला बंदरगाह भारत का सबसे बड़ा बंदरगाह है, जो कि गुजरात में स्थित है। आजादी के बाद यह पहला विकसित बंदरगाह है। कांडला बंदरगाह का नाम बदलकर पंडित दीनदयाल उपाध्याय बंदरगाह रखा गया है।
    • दीव और दमन के बीच खंभात की खाड़ी स्थित है।
    • काठियावाड़ प्रायद्वीप को गुजरात की मुख्य भूमि से अलग खंभात की खाड़ी करता है ।
    • गुजरात की राजधानी गाँधीनगर कर्क रेखा के दक्षिण में स्थित है।
    • नीलगिरि पहाड़ी : पश्चिमी घाट पर्वत श्रेणी तथा पूर्वी घाट पर्वत श्रेणी आपस में एक-दूसरे से नीलगिरि क्षेत्र में मिलती है। नीलगिरि क्षेत्र मूलत: तमिलनाडु राज्य में स्थित है। लेकिन इसका विस्तार तमिलनाडु के साथ-साथ कर्नाटक और केरल के क्षेत्र में भी देखने को मिलता है।
    • नीलगिरि की सबसे ऊँची चोटी डोडाबेटा है जिसकी ऊँचाई 2636m है। यह दक्षिणी भारत की दूसरी सबसे ऊँची पर्वत चोटी है।
    • अनाईमुडी : यह दक्षिण भारत काऊची पर्वत चोटी है। इसकी ऊँचाई 2695m है। अन्नाईमुडी तीन पहाड़ियों का केन्द्र बिन्दु है। इसके उत्तर में अन्नामलाई पहाड़ी, दक्षिण में इलायची की पहाड़ी तथा पूर्व में पालनी की पहाड़ी है।
    • पालगी की पहाड़ी तमिलनाडु राज्य में स्थित है।
    • Note : नीलगिरि पहाड़ी और अन्नामलाई पहाड़ी के बीच में पालघाट दर्रा स्थित है।
    • नीलगिरि क्षेत्र में टोडा नामक जनजाति पायी जाती है जो मूलत: पशुपालक जनजाति होती है। इस जनजाति में शिशु वध प्रथा का प्रचलन देखने को मिलता है। इस जनजाति में बहुपति विवाह का भी प्रचलन देखने को मिलता है।
    • पश्चिमी घाट पर्वत श्रेणी के अंतर्गत आने वाले पहाड़ियों का उत्तर से दक्षिण में क्रम निम्न है -
      कालसुबाई, महाबलेश्वर - महाराष्ट्र
      बाबा बूदन की पहाड़ी - कर्नाटक
      नीलगिरि पहाड़ी - तमिलनाडु
      अन्नामलाई पहाड़ी - केरल और तमिलनाडु
      इलायची पहाड़ी, कार्डमम पहाड़ी - केरल
    • पूर्वी घाट पर्वत श्रेणी के अंतर्गत आने वाले पर्वतों का उत्तर से दक्षिण में क्रम निम्न है-
      महेन्द्रगिरि - ओडिशा
      नल्लालाई, वेलीकोण्डा, पालकोण्डा, नगारी - आंध्र प्रदेश
      जवादी, शेवरॉय, पंचमलाई, सिरूमलाई - तमिलनाडु
    • महाराष्ट्र : सतमाला पहाड़ी, अजन्ता पर्वत श्रेणी, सह्याद्रि पर्वत श्रेणी, हरिश्चन्द्र पर्वत श्रेणी, बालाघाट पर्वत श्रेणी इत्यादि महाराष्ट्र राज्य में स्थित है।
    • महाराष्ट्र की सबसे ऊँची पर्वत चोटी कालसुबाई है, जिसकी ऊँचाई 1646m है।
    • ओडिशा : गढजात पहाड़ी, टिकरपाड़ा पर्वत श्रेणी, अस्सीया पहाड़ी, नयागढ़ पहाड़ी इत्यादि ओडिशा राज्य में स्थित है।
    • Note : टिकरपाड़ा पर्वत श्रेणी और नयागढ़ पहाड़ी के बीच से होकर महानदी गुजरती है।
    • सतमाला पहाड़ी और हरिश्चन्द्र पर्वत श्रेणी के बीच महाराष्ट्र राज्य में थालघाट दर्रा है। जो मुंबई को नासिक से जोड़ता है। (गोदावरी नदी)
    • हरिश्चन्द्र पर्वत श्रेणी और महाबलेश्वर के बीच में भोरघाट दर्रा है जो मुंबई को पुणे से आपस में जोड़ता है। (भीमा नदी)

             

    • पालघाट दर्रा केरल के कोच्चि को तमिलनाडु के कोयम्बटूर से आपस में जोड़ता है। यह दर्रा नीलगिरि पहाड़ी और अन्नामलाई पहाड़ी के बीच में स्थित है।
    • सेनकोटा घाट दर्रा : यह केरल के तिरूवनंतपुरम को तमिलनाडु के मदुरै से आपस में जोड़ता है। यह दर्रा कार्डमम पहाड़ी और वरूशनाऊ पहाड़ी के बीच में स्थित है।
    • Note : थालघाट दर्रा, भोरघाट दर्रा, पालघाट दर्रा, सेनकोटा घाट दर्रा पश्चिमी घाट पर्वत श्रेणी की दर्रा है।
    • दर्रा (Pass / Gap) : पर्वतीय भागों में एक स्थान से दूसरे स्थान तक जाने वाले मार्ग को दर्रा (Pass / Gap) कहते हैं। दर्रा खास तौर पर पर्वतीय राज्यों में देखने को मिलता है। जैसे हिमाचल प्रदेश, उत्तराखण्ड, सिक्किम इत्यादि।
    • तुजु दर्रा : यह मणिपुर में स्थित है। यह दर्रा मणिपुर को म्यांमार से आपस में जोड़ता है।
    • बोमाडिला दर्रा : यह अरूणाचल प्रदेश राज्य में स्थित है। यह दर्रा ल प्रदेश को तिब्बत से आपस में जोड़ता है।
    • दिफु दर्रा : यह अरूणाचल प्रदेश राज्य में स्थित है। यह अरूणाचल प्रदेश को म्यांमार से आपस में जोड़ता है ।
    • पांगसाऊ दर्रा : यह अरूणाचल प्रदेश राज्य में स्थित है। यह अरूणाचल प्रदेश को म्यांमार से जोड़ता है।
    • यांग्याप दर्रा : यह अरूणाचल प्रदेश राज्य में स्थित है। यह अरुणाचल प्रदेश को म्यांमार से जोड़ता है ।
    • नाथुला दर्रा : यह सिक्किम राज्य में स्थित है। यह दर्रा सिक्किम को तिब्बत से जोड़ता है। भारत-चीन युद्ध 1962 के दौरान इसी दर्रा से होकर चीनी आक्रमणकारी भारत में प्रवेश किया था।
    • जेलेप्ला दर्रा : यह सिक्किम राज्य में स्थित है। यह दर्रा सिक्किम को तिब्बत से आपस में जोड़ता है ।
    • माना दर्रा : यह उत्तराखण्ड राज्य में स्थित है। यह दर्रा उत्तराखंड को तिब्बत से जोड़ता है।
    • नीति दर्रा : यह दर्रा उत्तराखण्ड राज्य में स्थित है। यह उत्तराखंड को तिब्बत से आपस में जोड़ता है।
    • लिपुलेख दर्रा : यह दर्रा उत्तराखण्ड राज्य में स्थित है। यह उत्तराखंड को तिब्बत से आपस में जोड़ता है।
      Trick : मनीला उत्तराखण्ड |
    • मुलिंग्ला दर्रा : यह उत्तराखण्ड राज्य में स्थित है। यह दर्रा उत्तराखंड को तिब्बत से आपस में जोड़ता है ।
    • शिपकीला दर्रा : यह हिमाचल प्रदेश राज्य में स्थित है। यह दर्रा हिमाचल प्रदेश को तिब्बत से आपस में जोड़ता है।
    • रोहतांग दर्रा : यह हिमाचल प्रदेश राज्य में स्थित है। यह हिमाचल प्रदेश के लाहुल शहर को स्फीति से आपस में जोड़ता है।
    • बारालाचा दर्रा : यह हिमाचल प्रदेश राज्य में स्थित है ! यह हिमाचल प्रदेश के मनाली को लद्दाख के लेह से आपस में जोड़ता है।
      Trick : शराब - हिमाचल प्रदेश 
    • देवसा दर्रा : हिमाचल प्रदेश
    • जोजिला दर्रा : यह जम्मू कश्मीर में स्थित हैं। यह दर्रा श्रीनगर को लेह से आपस में जोड़ता है।
    • बुर्जिल दर्रा : यह जम्मू कश्मीर राज्य में स्थित है। यह दर्रा श्रीनगर को गिलगित से आपस में जोड़ता है।
    • बनिहाल दर्रा : यह जम्मू कश्मीर राज्य में स्थित है। यह दर्रा जम्मू को श्रीनगर से आपस में जोड़ता है।
    • पीरपंजाल दर्रा : यह जम्मू कश्मीर राज्य में स्थित है। यह जम्मू को कश्मीर से आपस में जोड़ता है।
    • काराकोरम, लानकला, देप्सला, अघील, चांगला, बारा ला, खार्दंगला, पेन्सिला इत्यादि दर्रा लद्दाख में स्थित है। ये सभी दर्रा लद्दाख को चीन से आपस में जोड़ता है। इन सभी दरों में सबसे अधिक ऊँचाई पर स्थित दर्रा काराकोरम है।
    • भारतीय मरूस्थल / रेगिस्तान (Desert ) : भारतीय मरूस्थल को थार मरूस्थल भी कहा जाता है। यह मरूस्थल सामान्यतः भारत के उत्तर-पश्चिम में स्थित भारतीय राज्य राजस्थान में विस्तृत है। इसका विस्तार पाकिस्तान में भी देखने को मिलता है। इसे पाकिस्तान में चोलिस्तान कहा जाता है। यह एक गर्म प्रकृति का मरूस्थल है। पूरे विश्व में सबसे ज्यादा मानव जनघनत्व ताला मरूस्थल थार मरूस्थल है। विश्व के कुल क्षेत्रफल के 1/5 भू-भाग या 20% भू-भाग पर मरूस्थल का विस्तार है।
    • विश्व का सबसे बड़ा मरूस्थल सहारा मरूस्थल है, जो अफ्रीका महादेश में स्थित है। यह भी एक गर्म प्रकृति का मरूस्थल है।
    • वैसा भू-भाग जहाँ 25 cm से कम वर्षा होती है वह भू-भाग मरूस्थल में तब्दील हो जाता है।
    • भारतीय द्वीप : भारतीय द्वीप को हमलोग दो भागों में बाँटकर पढ़ते हैं। (a) बंगाल की खाड़ी में स्थित अंडमान निकोबार द्वीप समूह (b) अरब सागर में स्थित लक्षद्वीप समूह |
    • (a) अंडमान निकोबार द्वीप समूह : यह भारत के दक्षिण-पूर्व में स्थित बंगाल की खाड़ी में स्थित है। इसमें द्वीपों की संख्या लगभग 572 है। जिसमें बड़े द्वीप मात्र 7 है। बड़े द्वीपों में चार बड़ा द्वीप अंडमान में जबकि 3 बड़े द्वीप निकोबार द्वीप समूह मैं स्थित है।
      1. उत्तरी अंडमान : यह अंडमान निकोबार द्वीप समूह का एक द्वीप है । इस द्वीप के पूर्वी भाग में नारकोनडम द्वीप है जो कि भारत का एकमात्र सुसुप्त ज्वालामुखी द्वीप है। अंडमान निकोबार द्वीप समूह की सबसे ऊँची पर्वत चोटी सैडलपीक (738m) है। इसी द्वीप पर स्थित है।
      2. मध्य अंडमान : यह भारत का सबसे बड़ा द्वीप है। इस द्वीप के पूर्वी भाग में भारत का एकमात्र सक्रिय ज्वालामुखी द्वीप बैरन द्वीप स्थित है।
      3. दक्षिणी अंडमान : अंडमान निकोबार द्वीप समूह की राजधानी पोर्ट ब्लेयर इसी द्वीप पर स्थित है। सेलुलर जेल इसी द्वीप पर स्थित है। 
      4. ग्रेट निकोबार: यह भारत का सबसे दक्षिणी द्वीप है। इसी द्वीप पर इंदिरा प्वाइंट स्थित है जो कि भारत का सबसे दक्षिणी बिंदु है। अंडमान निकोबार द्वीप समूह की दूसरी सबसे ऊँची पर्वत चोटी माउंट धुलियर (642m) इसी द्वीप पर स्थित है।
    • लैंडफॉल द्वीप : यह अंडमान निकोबार द्वीप समूह के सबसे उत्तर में स्थित द्वीप है !
    • न्यूमूर द्वीप : यह बंगाल की खाड़ी में स्थित है । इस द्वीपको लेकर भारत और बांग्लादेश के बीच विवाद है ।
    • प्रिपेरिस द्वीप : यह बंगाल की खाड़ी में स्थित है। इस पर म्यांमार का अधिकार है।
    • ग्रेट कोको द्वीप : यह बंगाल की खाड़ी में स्थित है। इस पर म्यांमार का अधिकार है।
    • टेरासा द्वीप, केमोराटा द्वीप, नंकौरी द्वीप, कट्चल्ल द्वीप इत्यादि निकोबार द्वीप समूह में स्थित है।
    • माजुली द्वीप : यह द्वीप असम में स्थित है। यह द्वीप का ब्रह्मपुत्र नदी द्वारा किया गया है। यह द्वीप अपने जैव विविधता के लिए प्रसिद्ध है। माजुली एक नदीय द्वीप है जो विश्व का सबसे बड़ा नदीय द्वीप है। हाल के दिनों में माजुली को असम के 33वें जिला का दर्जा दिया गया है। यह सबसे बड़ा नदी द्वीपीय जिला है। 
    • ह्वीलर द्वीप : यह ओडिशा राज्य में स्थित है। इसे वर्तमान समय में ए० पी० जे० अब्दुल कलाम द्वीप के रूप में जाना जाता है।
    • श्री हरिकोटा द्वीप : यह आंध्र प्रदेश राज्य में स्थित है। यह द्वीप पुलिकट झील को बंगाल की खाड़ी से अलग करता है। सतीश श्रवन अंतरिक्ष केन्द्र श्री हरिकांय द्वीप पर ही स्थित है।
    • पम्बन द्वीप : यह आदम ब्रिज या रामसेतु का ही एक भाग है। यह द्वीप मन्नार की खाड़ी में स्थित है । इस द्वीप पर रामेश्वरम् स्थित है। पम्बन द्वीप या पम्बन चैलन रामेश्वरम् द्वीप को भारत की मुख्य भूमि से अलग करता है।
    • काचा तीवू द्वीप : इस द्वीप को लेकर भारत और श्रीलंका के बीच विवाद है।
    • 30 दिसम्बर, 2018 ई० में अंडमान निकोबार द्वीप समूह के कुछ द्वीपों के नाम परिवर्तित किया गया है जो निम्न हैं-
      1. रॉस द्वीप : इसका नाम बदलकर सुभाष चन्द्र बोस द्वीप किया गया है। सुभाष चन्द्र बोस के जन्मदिवस पर 23 जनवरी को पराक्रम दिवस के रूप में मनाया जाता है।
      2. नील द्वीप : इसका नाम बदलकर शहीद द्वीप किया गया I
      3. हैवलॉक द्वीप : इसका नाम बदलकर स्वराज द्वीप किया गया है।
    • Note: ये तीनों द्वीप अंडमान द्वीप के अंतर्गत आते हैं।
    • (b) अरब सागर में स्थित लक्षद्वीप समूह :
    • लक्षद्वीप : यह भारत के दक्षिण-पश्चिम में स्थित अरब सागर में स्थित एक प्रवाल द्वीप है । लक्षद्वीप में द्वीपों की संख्या 36 है, जिसमें मात्र 60 द्वीप पर मानव निवास करता है। लक्षद्वीप का सबसे दक्षिणी द्वीप मिनीकॉय है। जो कि लक्षद्वीप का सबसे बड़ा द्वीप है। (कुछ अन्य स्रोतों के अनुसार लक्षद्वीप का सबसे बड़ा द्वीप आण्ड्रोट है ) । लक्षद्वीप का आकार घोड़े के नाल जैसा होता है।
    • मिनीकॉय द्वीप : इस द्वीप को लेकर भारत और मालदीव के बीच विवाद है । इस द्वीप पर भारत का अधिकार है।
    • आलियाबेट और खालियाबेट द्वीप : यह द्वीप खंभात की खाड़ी में स्थित है। नर्मदा नदी के मुहाने पर आलियाबेट द्वीप जबकि ताप्ती नदी के मुहाने पर खालियाबेट द्वीप स्थित हैं।
    • Note: भारत का सबसे उत्तर में स्थित नदी द्वीप माजुली द्वीप है जो असम में स्थित है।
    • भारत की प्रमुख घाटियाँ  :
    • घाटी (Valley) : दो पहाड़ियों के बीच के गहरे भाग को घाटी कहते हैं। घाटी सामान्यतः पर्वतीय राज्यों में मिलता है। घाटी का आकार सामान्यतः यू (U) आकार का होता है। घाटियों से नदियाँ बहा करती है। खास तौर पर वृहद हिमालय और लघु हिमालय के बीच में घाटी देखने को मिलता है।
    राज्य घाटी
    लद्दाख गलवान घाटी, मर्खा घाटी, सुरू घाटी, नुब्रा घाटी
    हिमाचल प्रदेश कांगड़ा घाटी, कुल्लु घाटी, लाहुल घाटी, स्फीति घाटी, पार्वती घाटी, किन्नौर घाटी, मालना घाटी, चंबा घाटी
    उत्तराखण्ड दून घाटी, फूलों की घाटी, जोहर घाटी, नेलांग घाटी, धर्मा पाटी
    सिक्किम चुंबी घाटी और युथांग घाटी
    नागालैंड जुकु घाटी
    आंध्र प्रदेश अराकु घाटी
    तमिलनाडु कंबम घाटी
    केरल साइलैंट वैली / मौन घाटी / शांत घाटी
    • पीरपंजाल और धौलाधर पर्वत श्रेणी के बीच में कुल्लू घाटी है। इसे देवताओं की घाटी भी कहा जाता है।
    • मालना घाटी छोटा यूनान के रूप में प्रसिद्ध है। 
    • धर्मशाला शहर कांगड़ा घाटी में स्थित है।
    • बौद्धों का प्रसिद्ध ताबो मठ स्फीति घाटी में स्थित है।
    • चन्द्रा और भागा नदी लाहुल घाटी से निकलती है।
    • चुंबी घाटी के समीप भारत, भूटान और चीन की सीमा मिलती हैं।
    • साइलैंट वैली अपनी जैव विविधता के लिए प्रसिद्ध हैं।
    • Line of Actual Control (LAC) के समीप गलवान घाट स्थित है।
    • न्योरा घाटी पश्चिम बंगाल में स्थित है।
    • सिलिकॉन वैली कर्नाटक राज्य में स्थित है।
    • पठार : धरातल से ऊँचा उठा हुआ वैसा स्थल जिसका आधार विस्तृत हो शीर्ष सपाट या मेजाकार हो, पठार कहलाता है। इसकी औसत ऊँचाई धरातल से 500-600 m होता है। भारत का दक्षिणी भाग जिसे हमलोग प्रायद्वीपीय भारत कहते हैं। यह मुख्यतः पठारी भाग ही है। जिस कारण इसे प्रायद्वीपीय पठार भी कहते हैं । प्रायद्वीपीय पठार कई पठारों से निर्मित है इसलिए इसे पठारों का पठार भी कहा जाता है। प्रायद्वीपीय पठार का आकार त्रिभुजाकार है। इसके अंतर्गत दक्कन का पठार, छोटानागपुर का पठार, दण्डकारण्य का पठार इत्यादि आता है।
    • छोटानागपुर का पठार : यह मूलत: झारखंड राज्य में है। यह पठार हजारीबाग के पठार, राँची का पठार तथा राजमहल की पहाड़ी से मिलकर बना है।
    • छोटानागपुर के पठार से कई सारी नदियाँ निकलती हैं जिसमें छोटानागपुर के पठार की सबसे बड़ी नदी दामोदर नदी है जो कि हजारीबाग के पठार को राँची के पठार से अलग करता है।
    • छोटानागपुर के पठार से स्वर्णरेखा नदी निकलती है जो दूसरी सबसे लम्बी नदी है। पवित्रता के दृष्टिकोण से इस नदी को छोटानागपुर की गंगा कहते हैं।
    • पारसनाथ की पहाड़ी : यह छोटानागपुर पठार की सबसे ऊँची चोटी है। इसकी ऊँचाई 1366m है। यह पहाड़ी जैन धर्म के लिए प्रसिद्ध है।
    • भारत का सबसे प्राचीनतम और प्राथमिक चट्टान आर्कियन क्रम का चट्टान है।
    • भारत का सबसे आधुनिकतम या नया चट्टान गोंडवाना क्रम का चट्टान है।
    • भारत का 98% कोयला गोंडवाना क्रम के चट्टान में ही पाया जाता है।
    • चट्टान निर्माण का क्रम : आर्कियन, धारवाड़, कुडप्पा, विंध्य ।
    • हिमालय वलित पहाड़ से संबंधित है।
    • इन्द्रावती पर्वत - छत्तीसगढ़
      मोल्लेम पर्वत - गोवा
      कलेसर पर्वत - हरियाणा
      बेतला - झारखण्ड

    वस्तुनिष्ठ प्रश्न एवं उत्तर

    1. हिमालय की सर्वोच्च श्रेणी कौन है ?
    (a) हिमाचल 
    (b) हिमाद्रि
    (c) शिवालिक
    (d) सह्याद्रि 
    2. इनमें कोन हिमालय के उत्तर में स्थित है ?
    (a) हिंदूकुश 
    (b) पटकाई बुम
    (c) काराकोरम
    (d) अरावली
    3. निम्नलिखित में कौन-सी चोटी भारत में स्थित नहीं है ?
    (a) के2
    (b) नंदादेवी
    (c) कंचनजंघा
    (d) माउंट एवरेस्ट
    4. महेंद्रगिरि कहाँ स्थित है ?
    (a) झारखंड में
    (b) ओडिशा में
    (c) आंध्र प्रदेश में
    (d) कर्नाटक में
    5. इनमें कौन केरल को तमिलनाडु से मिलाता है?
    (a) थालघाट 
    (b) भोरघाट
    (c) पालघाट
    (d) शिपकीला
    6. किस पहाड़ी पर लावा निक्षेप मिलते हैं?
    (a) राजमहल 
    (b) अरावली
    (c) नीलगिरि
    (d) लुशाई
    7. अरावली और विंध्याचल के बीच कौन पठार अवस्थित है?
    (a) कर्नाटक
    (b) छोटानागपुर
    (c) मालवा
    (d) बघेलखंड
    8. 'ऊटी' क्या है?
    (a) राजस्थान का एक प्रमुख पशु
    (b) दक्षिण भारत का एक पर्वतीय नगर
    (c) नदी
    (d) पहाड़ी दर्श
    9. थार मरुस्थल कहाँ स्थित है?
    (a) अरावली के पश्चिम में
    (b) अरावली के पूर्व में
    (c) विंध्याचल के उत्तर में
    (d) विंध्याचल के दक्षिण में
    10. भारत के उत्तरी मैदान का पूर्व-पश्चिम विस्तार कितने किलोमीटर की लंबाई में है ?
    (a) 900 किलोमीटर
    (b) 12.00 किलोमीटर
    (c) 24,00 किलोमीटर
    (d) 36,00 किलोमीटर
    11. भारत के उत्तरी मैदान का पश्चिमी भाग किस नाम से जाना जाता है?
    (a) गंगा का मैदान
    (b) यमुना का मैदान
    (c) पंजाब का मैदान
    (d) ब्रह्मपुत्र का मैदान
    12. ब्रह्मपुत्र के मैदान की लंबाई कितनी है ?
    (a) 600 किलोमीटर 
    (b) 800 किलोमीटर
    (c) 1000 किलोमीटर
    (d) 400 किलोमीटर
    13. मेघना क्या है ?
    (a) दक्षिण भारत की एक नदी
    (b) गंगा और ब्रह्मपुत्र की संयुक्त धारा
    (c) उत्तर बिहार की बाढ़ग्रस्त भूमि
    (d) सतलुज की सहायक नदी
    14. सतपुरा का सबसे ऊँचा भाग कौन है?
    (a) यमरकटक पहाड़
    (b) अजंता
    (c) जबलपुर 
    (d) धूपगढ़ चोटी
    15. खंभात की खाड़ी कहाँ है?
    (a) भारत और श्रीलंका के बीच
    (b) केरल में
    (c) गुजरात में 
    (d) ओडिशा में 
    16. इनमें कौन मुख्य हिमालय में की श्रेणी है? 
    (a) पटकाई 
    (b) जयंतिया 
    (c) किरथर 
    (d) शिवालिक
    17. निम्न में से कौन-सा शिखर सही सुमेलित नहीं है? 
    (a) मेहासनी - ओडिशा 
    (b) कुद्रेमुख - कर्नाटक 
    (c) कलसुबाई - महाराष्ट्र 
    (d) गोरखनाथ - उत्तर प्रदेश
    18. त्रिशूल शिखर किस राज्य में स्थित है
    (a) हिमाचल प्रदेश
    (b) ओडिशा
    (c) उत्तराखंड
    (d) उत्तर प्रदेश
    19. निम्न में से कौन-सा कथन सही नहीं है?
    (a) अघिल (Aghil) दर्रा लद्दाख को तिब्बत से जोड़ता है।
    (b) शिपकीला दर्रा सतलुज गज (Satluj Gorge) से गुजरता है।
    (c) रोहतांग दर्रा हिमाचल प्रदेश में है।
    (d) जोजीला दर्रा श्रीनगर को लद्दाख से जोड़ता है।
    20. निम्न में से किस दर्रे कौ ऊंचाई सबसे अधिक है। 
    (a) बाल्चा धुरा 
    (b) बानिहाल दर्श
    (c) खरडूंगला दर्रा
    (d) पीरपंजाल दर्श
    21. निम्नांकित पर विचार कीजिए : 
    1. सारामती पर्वत नागालैंड में स्थित है
    2. मकुती पहाड़ी त्रिपुरा में स्थित है
    3. मिकिर पहाड़ी असम में स्थित है
    उपरोक्त में से कौन-सा कथन सही है ?
    (a) 1 और 2 सही हैं
    (b) 2 और 3 सही हैं
    (c) 1 और 3 सही हैं
    (d) 1, 2 और 3 सही हैं
    22. निम्न में से किस घाटी में करेवा (Karewa) पाए जाते हैं?
    (a) ब्रह्मपुत्र
    (b) भद्रवाह
    (c) दून वैली
    (d) फूलों की घाटी
    23. कैलाश शिखर (Kylas Peak) निम्न में से किस राज्य में स्थित है ?
    (a) अरुणाचल प्रदेश 
    (b) हिमाचल प्रदेश
    (c) मेघालय
    (d) उत्तर प्रदेश
    24. वाल्चा - धुरा दर्रा किस-किस को जोड़ता है ?
    (a) जम्मू को तिब्बत से
    (b) कश्मीर को हिमाचल प्रदेश से
    (c) सिक्किम को तिब्बत से
    (d) कुमाऊँ (उत्तराखण्ड) को तिब्बत से
    25. दिफू दर्रा निम्न में से किस-किस को जोड़ता है ? 
    (a) अरुणाचल प्रदेश को चीन से
    (b) अरुणाचल प्रदेश को म्यांमार से
    (c) मणिपुर को म्यांमार से
    (d) नागालैंड को म्यांमार से
    हमसे जुड़ें, हमें फॉलो करे ..
    • Telegram ग्रुप ज्वाइन करे – Click Here
    • Facebook पर फॉलो करे – Click Here
    • Facebook ग्रुप ज्वाइन करे – Click Here
    • Google News ज्वाइन करे – Click Here
    ]]>
    Tue, 09 Apr 2024 10:38:57 +0530 Jaankari Rakho
    General Competition | Geography | भारत की भूगर्भिक संरचना https://m.jaankarirakho.com/951 https://m.jaankarirakho.com/951 General Competition | Geography | भारत की भूगर्भिक संरचना
    • पैंजिया और पैंथालासा : आज से करोड़ों वर्ष पूर्व विश्व के सभी महादेश अर्थात् सभी स्थलखण्ड आपस में एक-दूसरे के साथ सटे हुए थे। इस स्थलखण्ड को अल्फ्रेड वेगनर ने पैंजिया नाम दिया। पैंजिया के चारों तरफ जो जलीय / सागरीय / महासागरीय भाग था उसे पैंथालासा नाम दिया गया।

    • भारतीय भूगर्भिक संरचना का विकास पैंजिया के अंगारालैंड (लॉरेंशिया) तथा गोंडवानालैंड के विभाजन से प्रारंभ होता है।

    पैंजिया का विभाजन तथा विभिन्न स्थलखंडों का निर्माण

    अंगारालैंड ( लॉरेंशिया ) : यूरोप, एशिया, उत्तरी अमेरिका, ग्रीनलैंड गोंडवानालैंड : प्रायद्वीपीय भारत, अफ्रीका, ऑस्ट्रेलिया, दक्षिण अमेरिका, अंटार्कटिका, मेडागास्कर

     भारत के भूगर्भिक संरचनाओं का समय मापक्रम

    • समय के विभिन्न कालखंडों में गोंडवानालैंड के विभाजन तथा उसके एक भाग के उत्तर में प्रवाह के कारण भारतीय भूगर्भिक संरचना का विकास हुआ।
    • किसी भी देश की भूगर्भिक संरचना के द्वारा, उस देश की विभिन्न भागों में मिलने वाली चट्टानों की प्रकृति एवं उसके स्वरूप की जानकारी प्राप्त होती हैं। भूगर्भिक संरचना की दृष्टि से भारत को तीन स्पष्ट भागों में विभाजित किया जा सकता है-
      1. दक्षिण का प्रायद्वीपीय पठार
      2. उत्तर की विशाल पर्वतमाला
      3. उत्तर भारत का विशाल मैदानी भाग

    दक्षिण का प्रायद्वीपीय पठार

    • यह गोंडवानालैंड का ही एक भाग है। प्री-कैम्ब्रियन काल के बाद से ही यह भाग कभी भी पूर्णत: समुद्र में नहीं डूबा। यह आर्कियन युग के आग्नेय चट्टानों से निर्मित है जो अब नीस व सिस्ट के रूप में अत्यधिक रूपांतरित हो चुकी हैं । प्रायद्वीपीय भारत की संरचना में चट्टानों के निम्न क्रम मिलते हैं-
    1. आर्कियन क्रम की चट्टानें: ये प्राचीनतम एवं प्राथमिक चट्टानें हैं जो नीस व सिस्ट के रूप में रूपांतरित हो चुकी हैं। बुंदेलखंड नीस व बेल्लारी नीस इनमें सबसे प्राचीन हैं। बंगाल नीस व नीलगिरि नीस भी इन्हीं चट्टानों के उदाहरण हैं। इनमें जीवाश्म नहीं पाये जाते हैं।
    2. धारवाड़ क्रम की चट्टानें : ये आर्कियन क्रम की प्राथमिक चट्टानों के अपरदन व निक्षेपण से निर्मित परतदार चट्टानें हैं। ये अत्यधिक रूपांतरित हो चुकी हैं एवं इसमें जीवाश्म नहीं मिलते। कर्नाटक के धारवाड़ एवं बेल्लारी जिला, अराबली श्रेणियाँ, बालाघाट, रीवा, छोटानागपुर आदि क्षेत्रों में ये चट्टानें मिलती हैं। भारत की सर्वाधिक खनिज भंडार इसी क्रम के चट्टानों में मिलते हैं। लौह-अयस्क, तांबा और स्वर्ण इन चट्टानों में पाये जाने वाले महत्वपूर्ण खनिज हैं।
    3. कुड़प्पा क्रम की चट्टानें : इनका निर्माण धारबाड़ क्रम की चट्टानों के अपरदन व निक्षेपण से हुआ है। इस प्रकार ये भी परतदार चट्टानें हैं, जो अपेक्षाकृत कम रूपांतरित हैं परन्तु इनमें भी जीवाश्मों का अभाव मिलता हैं। कृष्णा घाटी, नल्लामलाई पहाड़ी क्षेत्र, पापाघानी व चेयार घाटी आदि में ये चट्टानें मिलती हैं।
    4. विंध्य क्रम की चट्टानें : कुडप्पा क्रम की चट्टानों के बाद ये चट्टानें निर्मित हुई हैं। इनका विस्तार राजस्थान के चित्तौड़गढ़ से बिहार के सासाराम क्षेत्र तक है। विंध्य क्रम के परतदार चट्टानों में बलुआ पत्थर मिलते हैं। इन चट्टानों का एक बड़ा भाग दक्कन ट्रैप से ढँका है।
    5. गोंडवाना क्रम की चट्टानें : ऊपरी कार्बोनीफेरस युग से लेकर जुरैसिक युग तक इन चट्टानों का निर्माण अधिक हुआ है। ये चट्टानें कोयले के लिए विशेष महत्वपूर्ण है। भारत का 98% कोयला गोंडवाना क्रम की चट्टानों में मिलता है। ये परतदार चट्टानें हैं एवं इनमें मछलियों व रेंगनेवाले जीवों के अवशेष मिलते हैं। दामोदर, महानदी, गोदावरी व उसकी सहायक नदियों में इन चट्टानों का सर्वोत्तम रूप मिलता है।
    6. दक्कन ट्रैप : इसका निर्माण मेसोजोइक महाकल्प के क्रिटैशियस कल्प में हुआ था। इस समय विदर्भ क्षेत्र में ज्वालामुखी के दरारी उद्भेदन से लावा का वृहद उद्गार हुआ वं लगभग : वर्ग किमी. का क्षेत्र इससे आच्छादित हो गया। ला इस क्षेत्र में 600 से 1500 मी. एवं कहीं-कहीं तो 3000 मी. की मोटाई तक बेसाल्टिक लावा का जमाव मिलता है। यह प्रदेश दकन ट्रैप कहलाता है। राजमहल ट्रैप का निर्माण इससे भी पहले जुरैसिक कल्प में हो गया था ।

    उत्तर की विशाल पर्वतमाला

    भारत के उत्तर में स्थित हिमालय पर्वत का निर्माण एक लम्बे भू-गर्भिक ऐतिहासिक काल से गुजरकर सम्पन्न हुआ है। इसके निर्माण के संबंध में कोबर का भू-सन्नति सिद्धान्त (Geo- Syncine Theory) एवं मोर्गन और आइजैक का प्लेट विवर्तनिकी सिद्धांत सर्वाधिक मान्य हैं। कोबर ने भू-सन्नतियों को पर्वतों का पालना (Cradle of Mountains) कहा है। ये भूसन्नतियाँ लंबे, संकरे व छिछले जलीय भाग है। उनके अनुसार आज से 7 करोड़ वर्ष पूर्व हिमालय के स्थान पर टेथिस (Tethys) भू-सन्नति थी जो उत्तर के अंगारालैंड को दक्षिण के गोंडवानालैंड से पृथक करती इन दोनों भूमि के अवसाद टेथिस भू-सन्नति में क्रमिक रूप से जमा होते रहे एवं इन अवसादों का क्रमशः अवतलन होता रहा, जिसके परिणामस्वरूप दोनों संलग्न अग्रभूमियों में दबाव जनित भू-संचलन उत्पन्न हुआ जिससे क्युनलुन एवं हिमालय - काराकोरम श्रेणियों का निर्माण हुआ। वलन से अप्रभावित या अल्प प्रभावित मध्यवर्ती क्षेत्र तिब्बत का के नाम से जाना गया।

    वर्तमान में मोर्गन और आइजैक के द्वारा प्रतिपादित प्लेट विवर्तनिकी सिद्धांत हिमालय की उत्पत्ति की उपयुक्त व्याख्या करता है। इसके अनुसार लगभग 7 करोड़ वर्ष पूर्व भारतीय प्लेट उत्तर-- - पूर्वी दिशा में स्थित यूरेशियन प्लेट की ओर गतिशील हुई। दो से तीन करोड़ वर्ष पूर्व ये भू-भाग अत्यधिक निकट आ गए, जिनसे टेथिस के अवसादों में वलन पड़ने लगा एवं हिमालय का उत्थान प्रारम्भ हो गया। सेनोजोइक महाकल्प के इयोसीन व ओलीगोसीन कल्प में वृहद हिमालय का निर्माण हुआ, मायोसीन कल्प में पोटवार क्षेत्र के अवसादों के वलन से लघु हिमालय बना। शिवालिक का निर्माण इन दोनों श्रेणियों के द्वारा लाए गए. अवसादों के वलन से प्लायोसीन कल्प में हुआ । क्वार्टरनरी अर्थात् नियोजोइक महाकल्प के प्लीस्टोसीन व होलोसीन कल्प में भी इसका निर्माण होता रहा है। हिमालय एक युवा पर्वत हैं, जिसका उत्थान अभी भी जारी है। हिमालय के क्षेत्र में आने वाले वाले भूकम्प, हिमालयी नदियों के निरन्तर होते मार्ग परिवर्तन एवं पीरपंजाल श्रेणी में 1500 से 1850 मीटर की ऊँचाई पर मिलने वाले झील निक्षेप करेवा हिमालय के उत्थान के अभी भी जारी रहने की ओर संकेत करते हैं।

    उत्तर भारत का विशाल मैदानी भाग

    • इनका निर्माण क्वार्टनरी या नियोजोइक महाकल्प के प्लीस्टोसीन एवं होलोसीन कल्प में हुआ है। यह भारत की नवीनतम भूगर्भिक संरचना है। टेथिस भू-सन्नति के निरन्तर संकरा व छिछला होने एवं हिमालयी व दक्षिणी भारतीय नदियों द्वारा लाए गए अवसादों के जमाव से यह मैदानी भाग निर्मित हुआ है। इसके पुराने जलोढ़ बांगर एवं नए जलोढ खादर कहलाते हैं। इस मैदानी भाग में प्राचीन वन प्रदेशों के दब जाने से कोयला और पेट्रोलियम के क्षेत्र मिलते हैं।

    वस्तुनिष्ठ प्रश्न एवं उत्तर

    1. भारत का सबसे प्राचीन भूखंड है-
    (a) प्रायद्वीपीय पठार
    (b) गंगा का मैदान
    (c) तटीय भाग
    (d) उत्तर का पर्वतीय भाग
    2. हिमालय के निर्माण में कौन-सा सिद्धांत सर्वमान्य है ?
    (a) भूमंडलीय गतिशीलता सिद्धांत
    (b) महाद्वीपीय विस्थापन सिद्धांत
    (c) प्लेट विवर्तनिक सिद्धांत
    (d) इनमें कोई नहीं
    3. इनमें कौन कथन सही है ?
    (a) भारत का दक्षिणी भाग गोंडवाना भूमि का एक खंड है।
    (b) दक्षिण भारत का अधिकतर चट्टानें रूपांतरित है।
    (c) दक्षिणी पठार की औसत ऊँचाई 400 से 900 मीटर तक है।
    (d) उपर्युक्त सभी कथन सही है
    4. इनमें कौन कथन सही नहीं है ?
    (a) भारत के तटीय भाग में सँकरे मैदान मिलते हैं।
    (b) भारत के उत्तरी भाग में जलोढ़ मिट्टी मिलती है।
    (c) हिमालय का पहाड़ी भाग भारत का सबसे पुराना भूखंड है।
    (d) मालवा पठार अरावली और विंध्याचल के बीच में है।
    5. विचार कीजिए-
    1. 250 मिलियन वर्ष पूर्व सभी महाद्वीप स्थलखंड के रू में एकत्रित जिसे पेंजिया कहा गया। 
    2. पेंजिया के चारों तरफ जलीय भाग को पेंथालासा कहा जाता है।
    3. पेंजिया शब्द का प्रयोग सर्वप्रथम अल्फ्रेड वेगनर ने किया।
    उपर्युक्त में कौन सही है / हैं-
    (a) 1 & 3
    (b) 1, 2, 3
    (c) 1 & 2
    (d) इनमें से कोई नहीं
    6. निम्नलिखित में से कौन-सा देश / महादेश अंगारालैण्ड या लॉरेंशिया का हिस्सा है ?
    (a) उत्तरी अमेरिका 
    (b) एशिया
    (c) यूरोप
    (d) सभी
    7. गोंडवानालैंड में विखंडन किस युग में प्रारंभ हुआ ?
    (a) जुरैसिक 
    (b) प्लीसटोसीन
    (c) टर्शियरी
    (d) NOT
    8. भारत का रूर प्रदेश निम्न में से किसे कहा जाता है ?
    (a) बुन्देल खंड पठार
    (b) छोटा नागपुर पठार
    (c) कुड़प्पा क्रम के चट्टान
    (d) मैकाल पठार
    9. प्रायद्वीपीय भारत कहा जाता है -
    (a) वैसा हिस्सा जो चारों तरफ जल से घीरा हो।
    (b) वैसा हिस्सा जो तीन तरफ जल से घीरा हो एवं एक तरफ स्थल से
    (c) वैसा हिस्सा जो दो तरफ जल एवं एक तरफ स्थल खंड से घीरा हो
    (d) इनमें से कोई नहीं
    10. निम्नलिखित में से किस चट्टान में जीवाश्म का अभाव पाया जाता है ?
    (a) आकियन क्रम
    (b) कुडप्पा
    (c) a & b
    (d) इनमें से कोई नहीं
    11. निम्नलिखित में से कौन-सा मरुस्थल है ?
    (a) सिंधु क्षेत्र 
    (b) गंगा क्षेत्र
    (c) असम क्षेत्र 
    (d) मध्य भारत क्षेत्र
    12. हिमालय की उत्पत्ति किस भू-सन्नति से हुई है ? 
    (a) टेथीज
    (b) इण्डोब्रह्मा
    (c) शिवालिक
    (d) गोदावरी
    13. निम्नलिखित में से कौन-सी चट्टान प्रणाली, भारत में नवीनतम है ?
    (a) विंध्यन
    (b) कुडप्पा
    (c) धारवाड़
    (d) गोंडवाना
    14. भारत के दक्कन के पठार पर बेसाल्ट - निर्मित लावा शैलों का निर्माण हुआ है- 
    (a) क्रिटेशियस युग में 
    (b) प्लीस्टोसीन युग में 
    (c) कार्बोनीफेरस युग में
    (d) मायोसीन युग में 
    15. लघु हिमालय स्थित है मध्य में-
    (a) ट्रांस हिमालय और महान हिमालय
    (b) शिवालिक और महान हिमालय
    (c) ट्रांस हिमालय और शिवालिक 
    (d) शिवालिक और बाह्य हिमालय
    16. निम्न में से कौन-सा कथन असत्य है ?
    (a) भौमिकीय दृष्टि से प्रायद्वीप क्षेत्र भारत का सबसे प्राचीन भाग है।
    (b) हिमालय विश्व में सबसे नवीन वलित (फोल्डेड ) को प्रदर्शित करते हैं।
    (c) भारत के पश्चिमी समुद्र तट का निर्माण नदियों की है। जमाव क्रिया द्वारा हुआ
    (d) भारत में गोंडवाना शिलाओं में कोयले का वृहत्तम भंडार है।
    17. गोंडवानालैंड के टूटने का क्रम प्रारंभ हुआ- 
    (a) परिमयन युग में 
    (b) जुरैसिक युग में
    (c) क्रिटेशियस युग में
    (d) ट्रियासिक युग में
    18. निम्नलिखित में भारत में कोयला उत्पन्न करने वाला भौमिकीय समूह है-
    (a) धारवाड़
    (b) विंध्यन
    (c) गोंडवाना
    (d) कुडप्पा
    19. भारत से उपबंध पुराचुंबकीय परिणामों के संकेत मिलते हैं कि भूतकाल में भारतीय स्थलपिंड सरका है-
    (a) उत्तर को
    (b) दक्षिण को
    (c) पूर्व को
    (d) पश्चिम को
    20. शिवालिक श्रेणी का निर्माण हुआ-
    (a) इयोजोइक में
    (b) पैलियोजोइक में
    (c) मसोजोइक में
    (d) सेनोजोइक में
    21. भारतीय उपमहाद्वीप मूलतः एक विशाल भूखंड का भाग था जिसे कहते हैं- 
    (a) जुरैसिक भूखंड
    (b) आर्यावर्त्त
    (c) इंडियाना
    (d) गोंडवानालैंड
    हमसे जुड़ें, हमें फॉलो करे ..
    • Telegram ग्रुप ज्वाइन करे – Click Here
    • Facebook पर फॉलो करे – Click Here
    • Facebook ग्रुप ज्वाइन करे – Click Here
    • Google News ज्वाइन करे – Click Here
    ]]>
    Tue, 09 Apr 2024 10:18:41 +0530 Jaankari Rakho
    General Competition | Geography | भूगोल : एक सामान्य परिचय https://m.jaankarirakho.com/950 https://m.jaankarirakho.com/950 General Competition | Geography | भूगोल : एक सामान्य परिचय
    • Geography शब्द का प्रयोग सर्वप्रथम ग्रीक विद्वान इरेटोस्थनीज ने किया था। जिस कारण उन्हें भूगोल का पिता कहा जाता है। 
    • कुछ विद्वान हिकेटियस को भूगोल का पिता मानते हैं क्योंकि उनकी पुस्तक पीरियड्स में पहली बार पृथ्वी का क्रमबद्ध विवरण प्रस्तुत किया गया है।
    • जर्मनी के भूगोलवेत्ता एलेक्जेंडर वॉन हम्बोल्ट को आधुनिक भूगोल का जनक माना जाता है। क्योंकि आधुनिक काल में इन्होंने ही भूगोल की दार्शनिक एवं प्रमाणिक व्याख्या की हैं।
    • Geography ग्रीक भाषा के दो शब्द Geo + Grapho से मिलकर बना है जिसमें Geo का अर्थ पृथ्वी तथा Grapho का अर्थ वर्णन होता है। अर्थात् संयुक्त रूप से Geography का शाब्दिक अर्थ पृथ्वी का वर्णन होता है।
    • परिभाषा - स्ट्रेबो नामक विद्वान के अनुसार पृथ्वी पर उपस्थित आकाशीय पिण्ड, सागर, स्थलखण्ड, झील, जलप्रपात, मानवीय क्रियाकलाप इत्यादि के बारे में जिस विषय में हम पढ़ते हैं, उसे ही भूगोल कहा जाता है।
    • सम्पूर्ण पृथ्वी के 71% भू-भाग पर जल का विस्तार है तो वही 29% -भाग पर स्थल का विस्तार हैं।
    • पृथ्वी को मुख्यतः सात महादेश और पाँच महासागर में बाँटा गया है।
    • महाद्वीप या महादेश - स्थलखण्ड के वृहत स्वरूप को महादेश कहते हैं। क्षेत्रफल के आधार पर विश्व का सबसे बड़ा महादेश एशिया है तो वही सबसे छोटा महादेश आस्ट्रेलिया हैं।
    • क्षेत्रफल के आधार पर महादेश का क्रम (घटते क्रम में): 1. एशिया, 2. अफ्रीका, 3 उत्तरी अमेरिका, 4. दक्षिणी अमेरिका, 5. अंटार्कटिका, 6. यूरोप, 7. ऑस्ट्रेलिया ।
    • अंटार्कटिका महादेश पर मानव निवास नहीं करता है, इसके अलावे अन्य सभी महादेशों पर मानव निवास करते हैं।
    • विश्व के सभी देश अंटार्कटिका महादेश पर अपना अनुसंधान संस्थान खोले हुए हैं। जिस कारण इस महादेश को विज्ञान को समर्पित महादेश भी कहा जाता है।
    • दक्षिण गंगोत्री, मैत्री और भारती, भारतीय अनुसंधान संस्थान हैं जो अंटार्कटिका में स्थित हैं।
    • महासागर - जलीय भाग के वृहत स्वरूप को महासागर कहा जाता है। महासागरों की कुल संख्या 5 है। जिसमें विश्व का सबसे बड़ा महासागर प्रशांत महासागर हैं तो वहीं सबसे छोटा महासागर आर्कटिक महासागर है।
    • 1. प्रशांत महासागर, 2. अटलांटिक महासागर, 3. हिन्द महासागर, 4. अंटार्कटिक महासागर, 5. आर्कटिक महासागर ।
    • भारत एशिया महादेश के दक्षिणी भाग में तथा हिंद महासागर के शीर्ष पर स्थित हैं। हिन्द महासागर एक ऐसा महासागर हैं जिसका नाम हमारे देश हिन्दुस्तान के नाम पर रखा गया हैं।
    • भारतीय संविधान अनुच्छेद-1 के तहत भारत का मात्र दो संवैधानिक नाम हैं- 1. इंडिया, 2. भारत ।
    • लेकिन भारत को और भी कई नाम जैसे - जम्बुद्वीप, आयावर्त्त, हिन्दुस्तान इत्यादि से जाना जाता है।

    भारत : एक परिचय

    अक्षांशीय विस्तार :
    भारत उत्तरी गोलार्द्ध में 8°4' उत्तरी अक्षांश रेखा से 37°6' उत्तरी अक्षांश रेखा के बीच में फैला हुआ है।

    देशांतरीय विस्तार :
    भारत पूर्वी देशान्तर में 68°7' पूर्वी देशान्तर रेखा से 97°25' पूर्वी देशान्तर में फैला हुआ है।

    • भारत पूर्वी देशान्तर में हैं इसलिए भारत का मानक समय 0° देशांतर रेखा से आगे है।
    • 82½° पूर्वी देशान्तर रेखा को भारत का मानक समय रेखा माना जाता है। क्योंकि 82½° पूर्वी देशांतर रेखा पर जो समय होता है वही समय पूरे भारतवर्ष का समय माना जाता है।
    • भारत का मानक समय 0° रेखा से 5:30 घंटा आगे है।
    • विश्व के सभी देश अपने समय का निर्धारण 0° देशांतर रेखा के आधार पर करता हैं।

    • कर्क रेखा : 23½° उत्तरी अक्षांश रेखा को कर्क रेखा कहा जाता है। यह भारत के लगभग मध्य से होकर गुजरती है और भारत को दो बराबर भाग उत्तरी भारत और दक्षिणी भारत में बाँटती है। कर्क रेखा भारत के आठ राज्यों से होकर गुजरती है जिसका पश्चिम से पूर्व में क्रम निम्न है-
      गुजरात, राजस्थान, मध्यप्रदेश, छत्तीसगढ़, झारखण्ड, प. बंगाल, मिजोरम।
      Trick : गरम (ग-गुजरात, र-राजस्थान, म- मध्यप्रदेश) छाझ (छ- छत्तीसगढ़, झ-झारखण्ड) पी (प. बंगाल) त्रृप्ति (त्रिपुरा) मिलेगी (मि-मिजोरम ) ।
    • कर्क रेखा के उत्तर में स्थित राज्य :
      हिमाचल प्रदेश, पंजाब, उत्तराखण्ड, हरियाणा, उत्तर प्रदेश, बिहार, सिक्किम, अरूणाचल प्रदेश, असम, मेघालय, नागालैंड, मणिपुर |
    • भारतीय मानक समय रेखा 822° पूर्वी देशांतर रेखा को भारतीय मानक समय रेखा कहा जाता है। जो भारत के पाँच राज्य उत्तर प्रदेश, मध्य प्रदेश, छत्तीसगढ़, ओडिशा और आंध्र प्रदेश से होकर गुजरती हैं।
      Trick : MOUCA (M-M.P., O-Odisha, U-U.P., C-Chhatisgarh, A-Andhra Pradesh)
    • Note :
      1. मध्य प्रदेश और छत्तीसगढ़ ऐसे राज्य हैं जिससे होकर कर्क रेखा और 82½° पूर्वी देशांतर रेखा दोनों गुजरती हैं।
      2. कर्क रेखा और 822° पूर्वी देशांतर रेखा एक-दूसरे को छत्तीगढ़ राज्य में प्रतिच्छेद करती है।
      3. भारत के पूर्वी भाग तथा पश्चिमी भाग में समय का अंतर दो घंटे का हो सकता है। गुजरात राज्य की तुलना में अरुणाचल प्रदेश का समय 2 घंटा आगे होगा।
      4. सम्पूर्ण भारत का अक्षांशीय विस्तार 6°4' उत्तरी अक्षांश रेखा से लेकर 37°6' उत्तरी अक्षांश रेखा तक हैं।
    • भारत के सबसे उत्तर में इंदिरा कॉल हैं जो लद्दाख में 37°6' उत्तरी अक्षांश रेखा पर स्थित हैं। वही भारत की सबसे दक्षिणी बिन्दु ग्रेट निकोबार में स्थित इंदिरा प्वाइंट हैं, जो 6°4' उत्तरी अक्षांश रेखा पर स्थित है।
    • भारत का सबसे पूर्वी बिंदु अरूणाचल प्रदेश में स्थित किबी हैं तथा पश्चिमी बिन्दु गुजरात में स्थित गुहार मोती हैं।
    • भारत की मुख्य भूमि का सबसे दक्षिणी विन्दु तमिलनाडु के कन्याकुमारी में स्थित कैंप कमोरिन है।
    • तमिलनाडु के कन्याकुमारी (कैप केमोरिन) में बंगाल की खाड़ी, अरब सागर और हिंद महासागर तीनों आपस में मिलती है।
    • भारत के दक्षिणी पूर्वी भाग में बंगाल की खाड़ी है। बंगाल की खाड़ी में ही अंडमान निकोबार द्वीप समूह स्थित है।
    • भारत के दक्षिणी पश्चिमी भाग में अरब सागर है। अरब सागर में लक्षद्वीप स्थित है।
    • भारत के मालाबार तट तथा लक्षद्वीप के बीच में जो सागरीय भाग है उसे लक्षद्वीप सागर कहा जाता है।
    • भारत के नौ राज्य समुद्री सीमा बनता है तथा 19 राज्य समुद्री सीमा नहीं बनाता है।
    • अरब सागर के साथ पाँच भारतीय राज्य सीमा बनाता है, जो निम्न है- 1. गुजरात, 2. महाराष्ट्र, 3. गोवा, 4. कर्नाटक, 5. केरल।
    • तमिलनाडु और श्रीलंका के बीच में मन्नार की खाड़ी स्थित है।
    • भारत की मुख्य भूमि की समुद्री सीमा की लंबाई 6100 km है।
    • भारत की सम्पूर्ण समुद्री सीमा (लक्षद्वीप और अंडमान निकोबार द्वीप समूह सहित ) की लंबाई लगभग 7516 km है।
    • भारत की स्थलीय सीमा की लंबाई लगभग 15,200 km है।
    • भारत के सम्पूर्ण सीमा (स्थलीय और जलीय) की लंबाई 22716 km है।
    • 'भारत सबसे लंबी स्थलीय सीमा बांग्लादेश के साथ बनाती है तथा सबसे स्थलीय सीमा अफगानिस्तान के साथ बनाती है ।
    • सबसे लंबी समुद्री सीमा गुजरात राज्य बनाता है तो वही सबसे छोटी समुद्री सीमा गोवा बनाता है।
    • पश्चिमी भारत की सबसे लंबी समुद्री सीमा गुजरात राज्य बनाता है तथा सबसे छोटी समुद्री सीमा गोवा राज्य बनाता है।
    • पूर्वी भारत की सबसे लंबी समुद्री सीमा आंध्र प्रदेश राज्य के साथ बनाता है तथा सबसे छोटी समुद्री सीमा पश्चिम बंगाल के साथ बनाता है।
    • दक्षिण भारत की सबसे लंबी समुद्री सीमा आंध्र प्रदेश राज्य बनाता है।
    • बांग्लादेश : भारत बांग्लादेश के साथ 4096 km की सीमा बनाता है। पाँच भारतीय राज्य बांग्लादेश के साथ सीमा बनाता है, जो निम्न हैं- 1. पश्चिम बंगाल, 2. असम, 3. मेघालय, 4. त्रिपुरा, 5. मिजोरम |
    • बांग्लादेश के साथ सबसे लंबी सीमा पश्चिम बंगाल बनाता है।
    • त्रिपुरा एक ऐसा राज्य है जो तीन तरफ बांग्लादेश से घिरा हुआ है।
    • चीन : भारत दूसरी सबसे लंबी स्थलीय सीमा चीन के साथ बनाता है। चीन के साथ भारतीय सीमा की लंबाई 3488 km है। चीन के साथ लद्दाख, हिमाचल प्रदेश, उत्तराखण्ड, सिक्किम और अरूणाचल प्रदेश सीमा बनाती है। भारत और चीन के बीच सीमा का निर्धारण मैकमोहन रेखा करती हैं। इस रेखा का निर्धारण 1914 ई० में हुआ है। यह रेखा मूलत: अरूणाचल प्रदेश और चीन के बीच सीमा बनाती है। अरूणाचल प्रदेश को पहले NEFA कहा जाता था। NEFA - North East Frontier Agency

    • तीन भारतीय राज्य प० बंगाल, सिक्किम और अरुणाचल प्रदेश तीन देशों के साथ सीमा बनाती है।
      1. पश्चिम बंगाल : यह बांग्लादेश, भूटान और नेपाल के साथ सीमा बनाती है।
      2. सिक्किम : यह नेपाल, भूटान और चीन के साथ सीमा बनाती है।
      3. अरूणाचल प्रदेश : यह भूटान, चीन और म्यांमार के साथ सीमा बनाती है ।
    • पाकिस्तान : भारत पाकिस्तान के साथ तीसरी सबसे लंबी स्थलीय सीमा बनाती है। भारत और पाकिस्तान की सीमा की लंबाई 3323 km है। 5 भारतीय राज्य और केन्द्रशासित प्रदेश पाकिस्तान के साथ सीमा बनाती है जो निम्न है- 1. लद्दाख, 2. जम्मू-कश्मीर, 3. पंजाब, 4. राजस्थान, 5. गुजरात । इसमें पाकिस्तान के साथ सबसे लंबी सीमा राजस्थान बनाती हैं।
    • रेडक्लिफ रेखा : भारत और पाकिस्तान के बीच सीमा का निर्धारण रेडक्लिफ रेखा करती है। यह रेखा 17 अगस्त, 1947 को खींचा गया था। यह एक अध्यारोपित रेखा है।
    • नेपाल : यह भारत के साथ चौथी सबसे लंबी सीमा बनाता है। भारत नेपाल सीमा की लंबाई 1751 km है । नेपाल 5 भारतीय राज्य के साथ सीमा बनाती है जो निम्न हैं- 1. उत्तराखण्ड, 2. उत्तर प्रदेश, 3. बिहार, 4. पश्चिम बंगाल, 5. सिक्किम ।
    • म्यांमार (बर्मा): भारत पाँचवीं सबसे लंबी सीमा म्यांमार के साथ बनाती है। इसकी लंबाई 1643 km है। इसके साथ 4 भारतीय राज्य सीमा बनाता है जो निम्न हैं- 1. अरूणाचल प्रदेश, 2. नागालैंड, 3. मणिपुर, 4. मिजोरम |
    • भूटान : भारत के उत्तर में भूटान हैं जो एक पर्वतीय देश है। भारत भूटान के साथ 699 km की सीमा बनाता है। 4 भारतीय राज्य भूटान से सीमा बनाता है जो निम्न हैं- 1. अरूणाचल प्रदेश, 2. पश्चिम बंगाल, 3. असम, 4. सिक्किम |
    • अफगानिस्तान : भारत सबसे छोटी सीमा (106 km) अफगानिस्तान के साथ बनाती है। भारत का एकमात्र केन्द्रशासित प्रदेश लद्दाख है जो अफगानिस्तान के साथ सीमा बनाती है।
    • भारत के भीतर पाँच राज्य ऐसे हैं जो न तो अंतर्राष्ट्रीय सीमा बनाता है न ही समुद्री सीमा बनाता है। वह पाँच राज्य निम्न हैं- 1. हरियाणा, 2. मध्य प्रदेश, 3. छत्तीसगढ़, 4. तेलंगाना, 5. झारखंड |
    • भारत में 16 ऐसे राज्य हैं जो अंतर्राष्ट्रीय सीमा बनाता है, जो निम्न हैं- 1. गुजरात, 2. राजस्थान, 3. पंजाब, 4. हिमाचल प्रदेश, 5. उत्तराखण्ड, 6. उत्तर प्रदेश, 7. बिहार, 8. प. बंगाल, 9. सिक्किम, 10. अरूणाचल प्रदेश, 11. असम, 12. मेघालय, 13. नागालैण्ड, 14. मणिपुर, 15. मिजोरम, 16. त्रिपुरा ।
    • भारत सात देशों के साथ जलीय सीमा बनाती है जो निम्न हैं- 1. पाकिस्तान, 2. बांग्लादेश, 3. म्यांमार, 4. श्रीलंका, 5. मालदीव, 6. थाइलैंड, 7. इंडोनेशिया ।
    • भारत तीन देशों के साथ जलीय और स्थलीय दोनों प्रकार की सीमा बनाता है जो निम्न हैं- 1. पाकिस्तान, 2. बांग्लादेश, 3. म्यांमार |
    • उत्तर प्रदेश सर्वाधिक राज्यों के साथ सीमा बनाने वाला राज्य है। जो आठ राज्य हैं जो निम्न हैं- 1. उत्तराखण्ड, 2. हिमाचल प्रदेश, 3. हरियाणा, 4. राजस्थान, 5. मध्य प्रदेश, 6. छत्तीसगढ़, 7. झारखण्ड, 8. बिहार ।
    • उत्तर प्रदेश सबसे लम्बी सीमा मध्य प्रदेश के साथ बनाता है तो वही सबसे छोटी सीमा हिमाचल प्रदेश के साथ बनाता है।
    • उत्तर प्रदेश एक केन्द्रशासित प्रदेश दिल्ली तथा एक देश नेपाल के साथ सीमा बनाती है।
    • दिल्ली उत्तर प्रदेश और हरियाणा के बॉर्डर पर स्थित एक केन्द्रशासित प्रदेश है।
    • सोनभद्र : यह एक ऐसा जिला है जो चार राज्य बिहार, झारखण्ड, छत्तीसगढ़ और मध्य प्रदेश के साथ सीमा बनाता है।
    • बिहार : इसके उत्तर में नेपाल, पूर्व में पश्चिम बंगाल, दक्षिण में झारखंड और पश्चिम में उत्तर प्रदेश है ।
    • झारखण्ड : यह पाँच राज्य के साथ सीमा बनाती है इसके उत्तर में बिहार, पूर्व में पश्चिम बंगाल, दक्षिण में ओडिशा, पश्चिम में छत्तीसगढ़ तथा उत्तर-पश्चिम में उत्तर प्रदेश स्थित है।
    • लद्दाख : लद्दाख के कुछ हिस्सा पर चीन का अधिकार है जिसे अक्साई चीन कहते हैं। वही लद्दाख के कुछ हिस्सा पर पाकिस्तान का हिस्सा है जिसे पाक अधिकृत कश्मीर (POK – Pak occupied Kashmir) कहते हैं।
    • LAC (Line of Actual Control) : यह रेखा लद्दाख में भारत और चीन के बीच खींची गई है।
    • LOC (Line of Control) : यह रेखा लद्दाख में भारत और पाकिस्तान के बीच खींची गई है।
    • जनसंख्या (Population) : 2011 की जनगणना के अनुसार भारत की कुल आबादी 121 करोड़ बतायी गयी हैं जो कि विश्व की कुल आबादी का 17.5% है। जनसंख्या के आधार पर भारत का विश्व में दूसरा स्थान है वही पहला स्थान चीन का है।
    • भारतीय राज्यों में जनसंख्या के आधार पर सबसे बड़ा राज्य उत्तर प्रदेश तो वही सबसे छोटा राज्य सिक्किम है।
    • केन्द्रशासित प्रदेशों में सबसे अधिक जनसंख्या वाला केन्द्रशासित प्रदेश दिल्ली हैं तो वही कम जनसंख्या वाला केन्द्रशासित प्रदेश लक्षद्वीप है।
    • जनसंख्या के आधार पर (घटते क्रम में देश) : 1. चीन, 2. भारत, 3. अमेरिका, 4. इंडोनेशिया, 5. ब्राजील |
    • जनसंख्या के आधार पर (घटते क्रम में राज्य) : 1. उत्तर प्रदेश, 2. महाराष्ट्र, 3. बिहार, 4. पश्चिम बंगाल |
    • द्वीप : वैसा भू-भाग जो चारों तरफ जल से घिरा हो, द्वीप कहलाता है।
    • विश्व का सबसे बड़ा द्वीप ग्रीनलैंड है। ग्रीनलैंड की राजधानी नुक है। ग्रीनलैंड भौगोलिक रूप से उत्तरी अमेरिका महादेश के अंतर्गत आता है। लेकिन इस पर राजनीतिक अधिकार- यूरोप महादेश का है। यहाँ पर शासन प्रशासन डेनमार्क के द्वारा-चलाया जाता है
    • प्रायद्वीप : वैसा भू-भाग जो तीन तरफ जल और एक तरफ स्थलखंड से घिरा हुआ हो, प्रायद्वीप कहलाता है। जैसे- हमारा देश भारत एक प्रायद्वीप है क्योंकि इसके पूर्वी भाग में बंगाल की खाड़ी, पश्चिमी भाग में अरब सागर और दक्षिणी भाग में हिंद महासागर है।
    • जलसंधि एवं चैनल :

    • 6° चैनल : यह ग्रेट निकोबार को सुमात्रा से अलग करता है।
    • 10° चैनल : यह अंडमान को निकोबार से अलग करता है।
    • डक्कन पास या डक्कन चैनल : यह दक्षिणी अंडमान को लिटिल अंडमान से अलग करता |
    • अंडमान निकोबार द्वीप समूह की राजधानी पोर्ट ब्लेयर दक्षिणी अंडमान में स्थित हैं।
    • कोको चैनल : यह कोको द्वीप को अंडमान से अलग करता है।
    • कोको द्वीप बंगाल की खाड़ी में स्थित हैं इस पर म्यांमार का अधिकार है।
    • पाक जलसंधि : यह भारत को श्रीलंका से अलग करता है।
    • 8° चैनल : यह मालदीव को मिनीकॉय से अलग करता है।
    • 9° चैनल : यह मिनीकॉय को लक्षद्वीप से अलग करता है।
    • 11° चैनल : यह अमिनी द्वीप को कनामोरे द्वीप से अलग करता है।
    • आधार रेखा : भारत का तटीय भाग टेढ़ी-मेढ़ी हैं, इस टेढ़ी-मेढ़ी रेखा को आपस में जो रेखा सीधी मिलाती है, उसे आधार- रेखा कहते हैं।

    • प्रादेशिक जल सीमा : आधार रेखा से प्रादेशिक जल सीमा की लम्बाई 12 नॉटिकल / समुद्री मील होता है।
    • संलग्न क्षेत्र मंडल : आधार रेखा से संलग्न क्षेत्र मंडल की लंबाई 24 नॉटिकल / समुद्री मील होता है।
    • अनन्य क्षेत्र मंडल : आधार रेखा से अनन्य क्षेत्रमंडल की लंबाई 200 समुद्री मील होता है।
    • एक नॉटिकल मील = 1852 m या 1.852 km 
    • Seven Sister : भारत के उत्तर - पूर्व में स्थित सात राज्य अरुणाचल प्रदेश, असम, मेघालय, नागालैंड, मणिपुर, मिजोरम, त्रिपुरा को सात बहनों वाली राज्य की संज्ञा दी जाती है।

    • चेरापूँजी खासी पहाड़ियों में स्थित है।
    • जुड़वाँ नगर : हैदराबाद- सिकंदराबाद, अहमदाबाद-गाँधीनगर, हुबली-धारवाड़, कटक-भुवनेश्वर ।
    • Delhi NCR (National Capital Reason) : Delhi NCR के अंतर्गत दिल्ली तथा उसके आस-पास के जिलों को शामिल किया गया है| Wikipedia के अनुसार 24 जिला को NCR में शामिल किया गया है, जिसमें उत्तर प्रदेश के 8, हरियाणा के 14 और राजस्थान के 2 जिला (भरतपुर, अलवर) को शामिल किया गया हैं।

    • Note : राजस्थान दिल्ली के साथ सीमा नहीं बनाता है। इसके बावजूद इसके 2 जिले NCR के अंतर्गत आता है।
    • पुडुचेरी : इस केन्द्रशासित प्रदेश में कुल चार जिले हैं जो निम्न हैं- 1. पुडुचेरी, 2. कराईकल, 3. यनम, 4. माहे ।
    • पुडुचेरी केन्द्रशासित प्रदेश का विस्तार भारत के तीन अलग-अलग राज्यों में है जो निम्न हैं- 1. आंध्र प्रदेश- यनम, 2. तमिलनाडु - पुडुचेरी और कराईकल 3. केरल - माहे।
    • पुडुचेरी केन्द्रशासित प्रदेश का विस्तार भारत के पूर्वी तट तथा पश्चिमी तट दोनों पर देखने को मिलता है । यनम पुडुचेरी और कराईकल पूर्वी तट पर स्थित हैं तो वही माहे पश्चिमी तट पर स्थित है।
    • पुडुचेरी केन्द्रशासित प्रदेश का संबंध बंगाल की खाड़ी तथा अरब सागर दोनों के साथ हैं । यनम पुडुचेरी और कराईकल बंगाल की खाड़ी के साथ सीमा बनाता है तो वही माहे अरब सागर के साथ सीमा बनाता है।
    • 1954 से पूर्व पुडुचेरी पर फ्रांसिसियों का अधिकार था ।
    • Note: भारत का सबसे प्राचीनतम भू-भाग प्रायद्वीपीय पठार है। प्रायद्वीपीय पठार - दक्षिणी भारत |
    • भारत उत्तर से दक्षिण में 3,214 km में फैला हुआ है, तो वही पश्चिम से पूरब में 2,933 km में फैला हुआ है।

    • भारत का विस्तार उष्ण तथा समशीतोष्ण दोनों कटिबंध में है। कर्क रेखा के दक्षिण में स्थित भारतीय राज्य उष्णकटिबंधीय में में है। है तो वही कर्क रेखा के उत्तर में स्थित भारतीय राज्य समशीतोष्ण कटिबंध में है।
    • कच्छ के रण को काठियावाड़ प्रायद्वीप से अलग कच्छ की खाड़ी करता है। गुजरात का कांडला बंदरगाह तथा ओखा बंदरगाह कच्छ की खाड़ी में स्थित है।
    • काठियावाड़ प्रायद्वीप को गुजरात की मुख्य भूमि से अलग खंभात की खाड़ी करता है।
    • दीव और दमन के बीच में खंभात की खाड़ी स्थित है।

    वस्तुनिष्ठ प्रश्न एवं उत्तर

    1. भारत की मुख्य भूमि कितनी डिग्री उत्तरी अक्षांश से आरंभ होती है ?
    (a) 4°4'
    (b) 22°30'
    (c) 8°4'
    (d) 12°4'
    2. कौन-सी अक्षांश रेखा भारत को दो बराबर भागों में बाँटती है ?
    (a) 23°30' उ०
    (b) 20°30′ उ०
    (c) 30°30 ' उ०
    (d) 33°30 '
    3. भारत का क्षेत्रफल कितना है ?
    (a) 42.6 लाख वर्ग किलोमीटर
    (b) 28.40 लाख वर्ग किलोमीटर
    (c) 56.2 लाख वर्ग किलोमीटर
    (d) 32.8 लाख वर्ग किलोमीटर
    4. इनमें किस राज्य में सूर्योदय पहले होता है ?
    (a) बिहार
    (b) गुजरात
    (c) अरुणाचल प्रदेश
    (d) तमिलनाडु
    5. मैकमहोन लाइन क्या है ?
    (a) भारत की मानक याम्योत्तर रेखा
    (b) भारत और चीन के बीच की अंतरराष्ट्रीय सीमरेखा
    (c) भारत को दो बराबर भागों में बाँटनेवाली अक्षांश रेखा
    (d) निकोबार द्वीप से गुजरनेवाली देशांतर रेखा के shan
    6. भारत का कौन पड़ोसी देश सबसे दक्षिण में स्थित है ?
    (a) मालदीव 
    (b) भूटान
    (c) श्रीलंका
    (d) बांग्लादेश
    7. वह कौन-सा स्थान है जिसकी स्थिति तोन समुद्रों के मिलन स्थल पर है ?
    (a) लक्षद्वीप
    (b) कोलकाता
    (c) कन्याकुमारी
    (d) इंदिरा प्वाइंट
    8. भारत विश्व में क्षेत्रफल की दृष्टि से कौन-सा स्थान रखता है ?
    (a) पहला
    (b) तीसरा
    (c) पाँचवाँ
    (d) सातवाँ
    9. कर्क रेखा किस राज्य से होकर नहीं गुजरती है ?
    (a) गुजरात
    (b) महाराष्ट्र
    (c) मध्य प्रदेश
    (d) झारखंड
    10. भारत के अक्षांशीय विस्तार में लगभग कितनी डिग्री का अंतर है ?
    (a) 20°
    (b) 30°
    (c) 40°
    (d) 50°
    11. भारत  के देशांतरीय विस्तार में लगभग कितनी डिग्री का अंतर है ?
    (a) 20° 
    (b) 30°
    (c) 40°
    (d) 50°
    12. भारत की स्थलीय सीमारेखा इसकी तटीय सीमारेखा से लगभग कितनी गुनी अधिक है ?
    (a) दुगुनी
    (b) तिगुनी
    (c) चौगुनी
    (d) पाँच गुनी
    13. इनमें कौन-सा पड़ोसी देश उत्तर प्रदेश को छूता है ?
    (a) अफगानिस्तान
    (b) पाकिस्तान
    (c) नेपाल
    (d) बांग्लादेश
    14. निम्न पड़ोसी देशों में भारत की सबसे लम्बी सीमा किस देश के साथ है ?
    (a) बांग्लादेश 
    (b) चीन
    (c) नेपाल
    (d) पाकिस्तान
    15. क्षेत्रफल के आधार पर भारत के निम्न राज्यों को अवरोही क्रम (descending order) में क्रमबद्ध कीजिए तथा नीचे दिए गए कूट की सहायता से सही उत्तर का चयन कीजिए : 
    1. मध्य प्रदेश
    2. महाराष्ट्र 
    3. राजस्थान
    4. उत्तर प्रदेश
    कूट :
    (a) 2, 3, 1 एवं 4 
    (b) 3, 1, 2 एवं 4
    (c) 3, 2, 1 एवं 4
    (d) 2, 4, 3 एवं 1
    16. निम्न में से किस राज्य का समुद्र तट ( Coast line) सबसे लम्बा है ?
    (a) आन्ध्र प्रदेश 
    (b) गुजरात
    (c) महाराष्ट्र
    (d) तमिलनाडु
    17. निम्न में से भारत का कौन-सा राज्य क्षेत्रफल में सबसे छोटा है ?
    (a) गोवा
    (b) नागालैंड
    (c) सिक्किम
    (d) त्रिपुरा
    18. निम्न देशों में भारत की सबसे छोटी अन्तर्राष्ट्रीय सीमा किस के साथ है ?
    (a) बांग्लादेश
    (b) चीन
    (c) नेपाल 
    (d) म्यांमार
    19. निम्न कथनों पर विचार कीजिए तथा नीचे दिए गए कूट की सहायता से सही उत्तर का चयन कीजिए-
    1. भारत के सूदूर दक्षिणी बिन्दु को इन्द्रा-प्वाइंट (Indira Point) करते हैं
    2. रझीरा - संकरी खाड़ी (Rajbeera Creek) भारत का सुदूर पश्चिमी छोर है ।
    3. भारत का सुदूर उत्तर बिन्दु (Northern Point) इन्द्रा - कोल (Indria Col) है।
    4. भारत का सुदूर पूर्व (Eastern Point) का नाम वालांग (Walang-Town) नगर है।
    कूट :
    (a) 1, 2 तथा 3 सही हैं ।
    (b) 1, 3 तथा 4 सही हैं।
    (c) 2, 3 तथा 4 सही हैं।
    (d) 1,2, 3 तथा 4 सही हैं।
    20. विश्व क्षेत्रफल में भारत का कितने प्रतिशत भाग है ?
    (a) 2.2
    (b) 2.4
    (c) 2.6 
    (d) 2.8
    21. निम्न में से कौन- -सा कथन सही नहीं है ?
    (a) क्षेत्रफल में राजस्थान भारत का सबसे बड़ा राज्य हैं।
    (b) क्षेत्रफल में मध्य प्रदेश भारत का दूसरा सबसे बड़ा राज्य है।
    (c) क्षेत्रफल में महाराष्ट्र भारत का तीसरा सबसे बड़ा राज्य है ।
    (d) क्षेत्रफल में कर्नाटक भारत का चौथा सबसे बड़ा राज्य है ।
    22. निम्न में से किस स्थान पर सूर्य की किरणें कभी भी लम्बवत् नहीं पड़ती ?
    (a) भोपाल (Bhopal)
    (b) जबलपुर (Jabalpur )
    (c) पालनपुर (Palanpur)
    (d) उज्जैन (Ujjain)
    23. निम्न में से कौन-सा ज्वालामुखी सक्रिय (Active) है ?
    (a) बैरन - द्वीप ( Barren Island)
    (b) नारकोंडम (Narcondam) द्वीप
    (c) चोबारे द्वीप (Choware Island)
    (d) सेन्टीनेल द्वीप ( Sentinel Island)
    24. निम्न में से कौन-सा स्थान श्रीलंका के सबसे निकट दूरी पर है ?
    (a) धनुषकोडी ( Dhanush Kodi)
    (b) कन्याकुमारी (Kanyakumari)
    (c) रामेश्वर (Rameshwaram)
    (d) तूतीकोरिन (Tuticorin)
    25. निम्न में से किस स्थान पर संध्या लाली सबसे कम समय के लिए होती है ?
    (a) बलुरू 
    (b) चेन्नई
    (c) कोयम्बटूर
    (d) मदुरई
    26. नाइन डिग्री चैनल किन-किन को अलग करता है ?
    (a) एंड्रॉट द्वीप को कावारती द्वीप से
    (b) मिनिकॉय द्वीप को मालदीव द्वीप से
    (c) अंडमान द्वीप को निकोबार द्वीप से
    (d) कवारत्ती द्वीप को मिनिकॉय द्वीप से
    27. भारत की स्थलीय सीमा कितने देशों के साथ जुड़ी है ? 
    (a) 6
    (b) 5
    (c) 8
    (d) 7
    हमसे जुड़ें, हमें फॉलो करे ..
    • Telegram ग्रुप ज्वाइन करे – Click Here
    • Facebook पर फॉलो करे – Click Here
    • Facebook ग्रुप ज्वाइन करे – Click Here
    • Google News ज्वाइन करे – Click Here
    ]]>
    Tue, 09 Apr 2024 07:43:55 +0530 Jaankari Rakho